​​​​Emergency Medical Technician; EMT-B Final Exam Test Bank Study Set Questions and Answers_ 2023.

Which of the following refers to a program or process for evaluating and improving the effectiveness of an EMS system?
A. System effectiveness management
B. Total quality system
C. Quality improvement
D. Process Improvement Plan (PIP)
C. Quality improvement

In 1966 the National Highway Safety Act charged which of the agencies with the development of emergency medical service standards?
A. U.S. Department of the Interior
B. U.S. Department of Health Services
C. U.S. Department of Transportation
D. U.S. Department of Homeland Security
C. U.S. Department of Transportation

Centralized coordination of emergency medical access, transportation, and care most refers to which of the following?
A. Emergency preparedness plan
B. Resource management
C. Central deployment
D. Tauma system
B. Resource management

Which of the following agencies is responsible for establishing EMS system assessment programs?
A. United States Health Services Agency (HSA)
B. Department of Health and Human Services (HHS)
C. National Transportation Safety Board (NTSB)
D. National Highway Traffic Safety Administration (NHTSA)
D. National Highway Traffic Safety Administration (NHTSA)

Which of the following BEST describes the purpose of the modern emergency medical services (EMS) system?
A. To have trained personnel respond quickly and provide emergency care on the scene, during transport, and at the hospital
B. To have trained personnel understand the limitations of their training and “do no harm,” while providing prompt transport to the hospital
C. To have trained personnel capable of assessing and caring for injured and ill patients on the scene
D. To have trained personnel knowledgeable in all aspects of prehospital care
A. To have trained personnel respond quickly and provide emergency care on the scene, during transport, and at the hospital

What is the minimum level of certification required of ambulance personnel responsible for direct patient care?
A. Emergency Medical Technician
B. Emergency Ambulance Driver
C. Advanced EMT
D. Emergency Medical Responder
A. Emergency Medical Technician

What BEST describes the level of EMS training that emphasizes activation of the EMS system and provides immediate care for life-threatening problems?
A. Cardiac care responder
B. Emergency Medical Responder
C. EMT
D. Emergency Medical Dispatcher
B. Emergency Medical Responder

What level of emergency medical training provides the most advanced pre-hospital care?
A. Emergency Medical Responder
B. Paramedic
C. Emergency Medical Technician
D. Advanced EMT
B. Paramedic

What is a common term used to describe the items needed for Standard Precautions or body substance isolation precautions?
A. Exposure-control plan
B. Harm-reduction strategy
C. Personal protective equipment
D. Infection-control plan
C. Personal protective equipment

Which of the following descriptions BEST defines the term pathogen?
A. A study of the origins of infection and disease
B. An organism that causes infection and disease
C. A medication with a harmful effect
D. An immunity developed after an exposure
B. An organism that causes infection and disease

An EMT should consider wearing protective eyewear when caring for which of these patients?
A. A patient with a difficult-to-visualize scalp laceration
B. A patient who was sprayed with mace by police during arrest
C. A patient who is actively coughing
D. A trauma patient who is covered in dirt, rocks, and other foreign debris
C. A patient who is actively coughing

Due to the high risk of an unstable work environment, EMTs responding to a motor vehicle crash may consider using __ in addition to standard PPE.
A. Face shields
B. Heavy- duty gloves
C. Level B hazmat suits
D. N-95 masks
B. Heavy- duty gloves

You are called to an assault on a homeless male. Dried blood from the trauma is visible around the patient’s mouth and nose. During assessment, you note the patient has a productive cough. The patient says, “Don’t worry, I’m not contagious. I’ve had this cough for a year.” Describe the best approach regarding respiratory isolation.
A. Mask yourself only
B. Use gloves for PPE, as that’s all that is needed
C. Isolate the patient with a non-rebreather mask
D. Mask yourself and the patient
D. Mask yourself and the patient

Which of the following is true regarding proper hand cleaning?
A. Soap and water should be used following any patient contact if the EMT does not wear gloves
B. In cases where soap and water and alcohol- based hand cleaners are not immediately available, bleach wipes should be used
C. If the EMT’s hands are visibly dirty, an alcohol-based hand cleaner is not sufficient
D. Following patient contact, the EMT should soak his hands in warm soapy water for at least 30 seconds and then shake them vigorously
C. If the EMT’s hands are visibly dirty, an alcohol-based hand cleaner is not sufficient

Which of the following pathogens can live in dried blood for days and should be a major concern for EMS providers even while cleaning contaminated equipment?
A. Hepatitis A
B. Tuberculosis
C. AIDS
D. Hepatitis B
D. Hepatitis B

Which of the following statements about HIV/AIDS is true?
A. HIV positive needle sticks have a 30 percent infection rate
B. Drugs are not available that slow HIV from progressing into AIDS
C. The HIV virus lives longer outside the body than hepatitis B
D. HIV positive needle sticks have less than a 0.05 percent infection rate
D. HIV positive needle sticks have less than a 0.05 percent infection rate

What is the name of the federal act that mandates a procedure by which emergency healthcare providers can find out if they have been exposed to potentially life-threatening diseases while on the job?
A. Occupational Exposure to Blood borne Pathogens Act
B. Ryan White CARE act
C. Communicable Disease Notification Act
D. EMS Personnel Health Protection Act of 1991
B. Ryan White CARE act

What are the minimum Standard Precautions an EMT should take when transporting a patient infected with Middle East Respiratory Syndrome (MERS)?
A. Gloves
B. Gloves, N-95 mask, and goggles
C. Gloves, N-95 mask, goggles, and gown
D. Gloves and N-95 mask
C. Gloves, N-95 mask, goggles, and gown

Which of the following terms is best defined as “a state of physical and/or psychological arousal to a stimulus?”
A. Homeostasis
B. Eustress
C. Distress
D. Stress
D. Stress

Which of the following is the form of stress that can cause immediate and long-term problems with an EMT’s health and well-being?
A. Eustress
B. Distress
C. Megastress
D. Hyperstress
A. Eustress

The term burnout is also known as a(n) __ stress reaction
A. Delayed
B. Cumulative
C. Acute
D. Post-traumatic
B. Cumulative

Post-traumatic stress disorder may be described as a(n).
A. Severe stress reaction
B. Acute stress reaction
C. Delayed stress reaction
D. Cumulative stress reaction
C. Delayed stress reaction

Which of the methods listed below is best described as moving a patient from the floor to a stretcher by having two or more rescuers kneel, curl the patient to their chest, stand, and then reverse the process to place the patient on the stretcher?
A. Draw-street method
B. Power lift
C. Direct carry
D. Direct ground lift
D. Direct ground lift

When placing all fingers and the palm in contact with the object being lifted, you are using which of the following?
A. Power grip
B. Vise grip
C. Lock grip
D. Power lift
A. Power grip

Which of the following factors should be considered before lifting any patient?
A. your physical limitations
B. The weight of the patient
C. Communications
D. All of the above
D. All of the above

Which of the following is the correct position of an EMT’s feet when lifting?
A. As wide apart as possible
B. Two feet apart
C. As close together as possible
D. Shoulder-width apart
D. Shoulder-width apart

Which of the following statements regarding body mechanics is true?
A. Position your feet close together
B. Use your legs to lift
C. Twist your torso while lifting
D. Use your back to lift
B. Use your legs to lift

Which of the following should you do when reaching for something?
A. Avoid reaching more than 15 to 20 inches in front of your body
B. Avoid twisting
C. Keep your back in a locked- in position
D. All of the above
D. All of the above

Which type of consent must be used by the EMT when seeking to treat a mentally competent adult?
A. Unconditional consent
B. Conscious consent
C. Expressed consent
D. Implied consent
C. Expressed consent

Which of the following refers to the set of regulations that defines the legal actions expected and limitations placed on the EMT?
A. Scope of practice
B. Professional standards
C. Legal standards of practice
D. Protocols and standing orders
A. Scope of practice

The administration of oxygen and the application of spinal immobilization is part of which of the following for the EMT?
A. Scope of practice
B. Duty to act
C. Standard of practice
D. Protocols and standing orders
A. Scope of practice

Which of the following refers to the care that would be expected to be provided by an EMT with similar training when caring for a patient in a similar situation?
A. Scope of practice
B. Professional standards
C. Standard of practice
D. Protocols and standing orders
C. Standard of practice

Your patient is a 40 year old known diabetic who was found unconscious at work by a coworker. What type of consent allows you to treat this patient?
A. Expressed consent
B. Consent for mentally incompetent adults
C. Implied consent
D. Consent for treatment for minor emergencies
C. Implied consent

Your patient is a 10 year old boy who suffered a possible fractured arm while rollerblading at a friend’s house. Which of the following is the BEST way to obtain consent for treatment?
A. Get consent from the patient’s 15 year old sister, who is at the scene
B. Act on implied consent
C. Call the patient’s mother at work
D. Allow the patient to consent as an emancipated minor
C. Call the patient’s mother at work

Your respond to a middle school for a 12 year old male patient who has been hit by a car. You get consent to treat the patient from the school principal. What concept allows the principal to speak for the parents?
A. Informed consent
B. Healthcare proxy
C. In loco parentis
D. Res ipsa loquitur
C. In loco parentis

The EMT’s obligation to provide care to a patient either as a formal or ethical responsibility is known as which of the following?
A. Standard of care
B. Scope of practice
C. Duty to act
D. Legal responsibility
C. Duty to act

You arrive on the scene of a 55 year old male patient. The patient’s wife called 911 because he is having chest pains. The patient is very angry with his wife for calling 911 because he states he only has heartburn and adamantly refuses any treatment or transport. After signing the patient refusal form, the patient collapses and goes into sudden cardiac arrest. The wife is crying uncontrollably and begging you to do something. What should you do?
A. Provide emergency care under implied consent
B. Explain to the wife that her husband signed a legal document refusing care, and if you intervened to help him now, you would be breaking the law
C. Provide emergency care for the patient only if his wife signs a document stating they will not sue the EMS service
D. Respect his legal right to not have any treatment, regardless of how much the wife begs you to help
A. Provide emergency care under implied consent

In which of the following situations should an EMT withhold resuscitative measures from a patient in cardiac arrest?
A. The patient’s caregiver presents a DNR order signed by the patient and his physician
B. Family members request that nothing be done
C. The EMT’s religious beliefs permit withholding resuscitation, and the caregiver presents documentation of the patient’s wishes
D. All of the above
A. The patient’s caregiver presents a DNR order signed by the patient and his physician

Your patient is a 45 year old man who is suffering from chest pain. Upon arrival, the patient is pale, sweaty, and seem short of breath. The patient is angry with his daughter for calling 911. He says that he had some spicy sausage for breakfast and has indigestion. Which of the following is an appropriate means of getting the patient the care he needs?
A. Call the patient’s neighbors and tell them that you have been called to the paitent’s hose but he is now refusing care
B. Tell the patient that his chest pain is most likely caused by his diet, have him take an antacid, and go to bed
C. Inform the patient that if he does not agree to treatment, you will have to take him against his will because he has a potentially life-threatening problem
D. Try to find out why the patient does not want to go to the hospital
D. Try to find out why the patient does not want to go to the hospital

You are on the scene with a 72 year old male patient with chest pain. The patient is complaining of shortness of breath but also refuses to go to the hospital, even after multiple attempts urging him to go. Which of the following should you do next?
A. Contact medical direction for orders to restrain the patient
B. Fully inform the patient about his situation and the implications of refusing care
C. Stay with the patient until he loses consciousness
D. Inform the patient that he is having a “heart attack” and must be taken to the hospital for evaluation
B. Fully inform the patient about his situation and the implications of refusing care

If the EMT is in doubt as to whether a patient in cardiac arrest should be resuscitated, which of the following is the best decision?
A. Have the patient’s family put their wishes in writing
B. Consult with the patient’s physician
C. Withhold resuscitative measures
D. Begin resuscitative measures
D. Begin resuscitative measures

Which of the following situations BEST illustrates the act of abandonment by the EMT?
A. An EMT who is off-duty sees a motor vehicle collision with probable injuries but does not stop to help
B. The EMT resuscitates a patient who has a DNR order signed by his physician
C. An EMT transports a patient to the emergency department, leaves the patient in the waiting room, but does not advise the ED staff
D. An EMT begins care of a patient, then turns the patient over to a paramedic
C. An EMT transports a patient to the emergency department, leaves the patient in the waiting room, but does not advise the ED staff

Two EMTs respond to the scene of a syncopal episode. They evaluate a 50 year old male patient who passed out and cut his upper lip. The EMTs examine the patient and his vital signs are normal. The EMTs talk the patient out of an expensive ambulance ride and suggest he will save money by driving himself to the urgent care clinic for stitches for his lip rather than waiting all night in the emergency department as a nonemergency patient. The patient signs the EMS refusal form. The patient decides to see his own doctor the next morning but dies at his house from sudden cardiac arrest an hour later. Which of the following statements is true?
A. The EMTs are not negligent because the patient signed the refusal form
B. The EMTs are negligent because there was proximate causation
C. The EMTs are negligent because the patient died
D. The EMTs are not negligent because the EMTs have no control over the patient’s medical condition
B. The EMTs are negligent because there was proximate causation

An off-duty EMT is driving down the road when she sees a major vehicle accident. There are no Emergency Medical Responders on the scene yet. There are only Good Samaritans. The EMT is late for a doctor’s appointment so she decides that she will not stop and help. The driver of the vehicle dies before help arrives. One of the Good Samaritans notices her EMT license plates and writes them down as she passes by. The Good Samaritan is angry that the EMT did not stop and help and tries to get the EMT fired for not helping. Which of the following statements in true?
A. The EMT is negligent because the patient died
B. The EMT is negligent for not stopping and helping
C. The EMT is not negligent because she did not have a duty to act
D. The EMT is not negligent because she had a doctor’s appointment
C. The EMT is not negligent because she did not have a duty to ac

Which of the following statements is true concerning protection by Good Samaritan laws?
A. An EMT is expected to act as a lay person when providing emergency care off-duty
B. The purpose of Good Samaritan laws is to protect people who are trying to help in an emergency
C. A Good Samaritan law allows an EMT to exceed the scope of practice, if necessary, when providing emergency care off-duty
D. An EMT cannot be sued for negligence if the state in which he works has a Good Samaritan law
B. The purpose of Good Samaritan laws is to protect people who are trying to help in an emergency

Which of the following requires training, polices, and procedures related to storing, accessing, and sharing patient information?
A. COBRA
B.EMTALA
C.HIPAA
D. HIAPA
C.HIPAA

While documenting a call, you add a false statement that was made about a local doctor. This could constitute which of the following?
A. HIPAA violation
B. Degradation of character
C. Slander
D. Libel
D. Libel

You respond to a bus accident with multiple patients. As you arrive at the scene, you are approached by a local politician who states that his wife has been involved. He tells you that if you will take care of her first, he will ensure you get the promotion you are up for. This becomes what type of decision for you as an EMT?
A. Medical
B. Ethical
C. Financial
D. Legal
B. Ethical

While treating a patient involved in a shooting at the patient’s vacation home, which of the following actions may interfere with the investigation of a crime scene?
A. Using the patient’s phone
B. Moving the patient
C. Using the bathroom
D. All of the above
D. All of the above

Which of the following is the EMT’s primary responsibility at a secured crime scene?
A. Taking notes that may be needed during court testimony
B. Preserving evidence
C. Providing patient care
D. Identifying any potential suspects encountered at the scene
C. Providing patient care

Which of the following information may be important to law enforcement officers investigating a crime scene?
A. Your experience as an EMT
B. Your opinion of what happened before you arrived on scene
C. How you gained access to the scene
D. What route you took when responding to the scene
C. How you gained access to the scene

You respond to the report of an unconscious female patient. You can acquire important medical information about the patient through which of the following?
A. Medical history identifier
B. Medical identification device
C. Driver’s license
D. On call medical director
B. Medical identification device

What is the distinction between anatomy and physiology?
A. Anatomy is the study of body components and systems, and physiology is the study of injuries and disease processes
B. The terms are synonymous
C. Anatomy is the study of body structures, and physiology is the study of body function
D. Anatomy is the study of physical body structures, whereas physiology is the study of emotions and behavior
C. Anatomy is the study of body structures, and physiology is the study of body function

With regard to medical terminology, a prefix is:
A. A modifier that indicates if a term is singular or plural
B. Added to the beginnings of roots or words to modify or qualify their meaning
C. The foundation of a word or term
D. The combination of any two or more whole words
B. Added to the beginnings of roots or words to modify or qualify their meaning

The abdominal quadrants include all of the following except the:
A. Right medial
B. Right upper
C. Left lower
D. Left upper
A. Right medial

The term lateral is best defined as:
A. Toward the middle of
B. To the back of
C. To the side
D. Under the arms
C. To the side

What is another term for the frontal aspect of the body?
A. Posterior
B. Caudal
C. Anterior
D. Dorsal
C. Anterior

The inside of a person’s thigh is also known as its _ aspect.
A. Medial
B. Axillary
C.Inferior
D. Lateral
A. Medial

You respond to a large concert venue where a number of spectators are reported to be severely intoxicated. You are directed to an area where several patients appear to be unconscious, lying face down on the ground. The position of these patients is describes as:
A. Posterior
B. Prone
C. Anterior
D. Supine
B. Prone

Which of the following BEST describes the anatomic position?
A. Standing up, facing forward, with arms raised above the head
B. Supine with arms crossed over the chest and knees slightly bent
C. Standing in profile with the hands on the hips
D. Standing, facing forward, with arms at the side, palms forward
D. Standing, facing forward, with arms at the side, palms forward

You and your newly hired EMT partner arrive on the scene of a bicycle collision at the local community park. One cyclist stands by and says that she has no injuries. The other is lying on his side on the bike path, guarding his ribs and holding the lower part of his left leg. Your partner kneels next to the man, introduces herself, and asks, “Can you ambulate?” The patient looks up, confused and in obvious pain. “Can you ambulate? You know… walk?” Your partner says, a little louder. After transporting the patient, you discuss the call with your partner and suggest that she should avoid using medical terms unnecessarily when taking with patients. She seems insulted and says, “Why?” “What would you say?”
A. Tell her that the point of communicating with patients and other providers is so there is a clear understanding; using medical terms when not necessary can cause confusion
B. You should say that you are an experience EMT and since she is new, she should just take your advice and apply it
C. Explain that using large words and medical terms can be seen as being egotistical, which can alienate both patients and other providers
D. Tell her that the general public isn’t smart enough to understand medical terminology and that all communication must be “dumbed down” when talking to patients
A. Tell her that the point of communicating with patients and other providers is so there is a clear understanding; using medical terms when not necessary can cause confusion

Why should an EMT avoid use of acronyms and abbreviations when communicating?
A. The medical acronyms and abbreviations used by prehospital care providers and hospital staff are different
B. They should only be avoided in verbal communications, where they can be misunderstood; they are expected in written patient care reports
C. There is a chance that they can lead to errors in continued care for the patient
D. Using acronyms and abbreviations is considered unprofessional
C. There is a chance that they can lead to errors in continued care for the patient

You and your EMT partner are responding to a medical aid call in rural West County area. The dispatcher advises that the caller is reporting the patient as having a history of “plegia.” Why would it be beneficial to have the dispatcher clarify a prefix for the word plegia?
A. A patient with plegia is potentially contagious, and the EMTs need to know what precautions are required
B. Without a clarifying prefix, it is difficult for the EMTs to effectively prepare for the type of patient they may encounter
C. Because a dysplegiac patient generally requires an EMT-Paramedic level of care, and it may change the level of response
D. There is actually no need to clarify the word
B. Without a clarifying prefix, it is difficult for the EMTs to effectively prepare for the type of patient they may encounter

A patient with bilateral femur fractures would have which of the following?
A. A femur fracture occurring with little to no trauma
B. A femur fracture in which the bone ends have punctured the muscle and skin of the thigh
C. Two fractures in the same femur
D. Fractures in both femurs
D. Fractures in both femurs

What word would be used to refer to a patient’s rapid breathing?
A. Dyspnea
B. Dyseffusion
C. Tachycardia
D. Tachypnea
D. Tachypnea

What is the primary reason for an EMT to use specific and proper medical terminology?
A. Medical communication needs to be exact and consistent
B. It will make patients and family members trust in the EMTs abilities more
C. People may think that an EMT is not intelligent or professional unless she uses medical terms
D. EMS providers can’t bill for services unless the correct terminology is used in all documentation
A. Medical communication needs to be exact and consistent

Which of the following are abdominal regions created by drawing two imaginary lines intersecting at the navel?
A. Abdominal sextants
B. Abdominal quadrants
C. Abdominal planes
D. Abdominal regions
B. Abdominal quadrants

To check the distal pulse of a patient with an injury to the forearm, the EMT would check fo a pulse in which location?
A. Throat
B. Upper arm
C. Wrist
D. Armpit
C. Wrist

Physiology is the study of:
A. Exercise and nutrition
B. Muscles and movement
C. Body structures
D. Body systems
D. Body systems

In relation to anatomy, the term topography means:
A. Key elements of the central nervous system
B. External landmarks of body structures
C. The study of the skin
D. Blood pressure fluctuations
B. External landmarks of body structures

Which of the following structures or tissues is NOT part of the musculoskeletal system?
A. Tendons
B. Acetabulum
C. Fat
D. Ligaments
C. Fat

Which of the following structures do NOT form part of the thorax?
A. Ischium
B. Ribs
C. 8th through 19th vertebrae
D. Sternum
A. Ischium

Which of the following BEST describes the medical condition of shock?
A. A state of inadequate tissue perfusion
B. An extreme emotional reaction to a stressful event
C. Delayed capillary refill
D. Hypotension
A. A state of inadequate tissue perfusion

What are the two most easily injured portions of the spine?
A. Cervical and lumbar
B. Sacral and coccygeal
C. Cervical and thoracic
D. Lumbar and sacral
A. Cervical and lumbar

Which of the following terms describes the heart muscle’s ability to generate its own electrical impulses?
A. Contractility
B. Automaticity
C. Conductivity
D. Irritability
B. Automaticity

Which of the following respiratory processes requires the active use of muscles?
A. Bifurcation
B. Inhalation
C. Exhalation
D. Gas exchange
B. Inhalation

What type of muscles control the size of bronchioles in the lungs?
A. Skeletal
B. Smooth
C. Striated
D. Voluntary
B. Smooth

A clot cannot form without which of the following blood components?
A. Plasma
B. White blood cells
C. Corpuscles
D. Platelets
D. Platelets

You are caring for a 47 year old male patient with classical chest pain suggestive of a myocardial infarction. Based on your understanding of the cardiovascular system, you know he could be suffering from damage, narrowing, or blockage of what arteries?
A. Femoral arteries
B. Coronary arteries
C. Brachial arteries
D. Pulmonary arteries
B. Coronary arteries

Which of the following types of blood vessels allow the exchange of substances directly between the blood and the cells of the body?
A. Veins
B. Capillaries
C. Arterioles
D. Alveoli
B. Capillaries

The study of how disease affects the functioning of the human body is called:
A. Psychology
B. Anatomy
C. Pathophysiology
D. Physiology
C. Pathophysiology

The volume of air that is moved in and out of the chest in a normal breath cycle is called:
A. Expiration
B. Inspiration
C. Vital capacity
D. Tidal volume
D. Tidal volume

Not all inspired air reaches the alveoli; 150 mL is stopped in the airway leading to the alveoli. This is called:
A. Tidal Volume
B. Vital capacity
C. Bronchial air
D. Dead air space
D. Dead air space

In normal exhalation, the diaphragm and intercostal muscles relax to contract the chest, which creates a positive pressure. This is what type of process?
A. Respiration
B. Inspiration
C. Active
D. Passive
D. Passive

Stroke volume depends on a series of factors: one is the force the myocardial muscle exerts to move the blood. This is known as:
A. Contractility
B. Automaticity
C. Preload
D. Afterload
A. Contractility

What substances, when dissolved, separate into charged particles?
A. Ions
B. Electrolytes
C. Cations
D. ATP
B. Electrolytes

The basic nutrient of the cell and the building block for energy is:
A. Protein
B. Water
C. Oxygen
D. Glucose
D. Glucose

Our blood transports oxygen from the lungs to the cells and returns with what byproduct of metabolism?
A. Carbon monoxide
B. Carbon dioxide
C. Lactic acid
D. Carbolic acid
B. Carbon dioxide

The seat of respiratory control is found in the:
A. Lungs
B. Nose
C. Medulla oblongata
D. Chest
C. Medulla oblongata

__ blood returns to the right side of the heart
A. Deoxygenated
B. Oxygenated
C. Anemic
D. Red
A. Deoxygenated

The high-pitched sound caused by an upper airway obstruction is known as:
A. Stridor
B. Gurgling
C. Rhonchi
D. Rales
A. Stridor

The trachea branches off at the _ and forms two mainstem bronchi
A. Carina
B. Bronchioles
C. Alveoli
D. Pleura
A. Carina

All of the following can result in airway obstructions, except:
A. Burns
B. Facial trauma
C. Infections
D. The tongue
D. The tongue

Perhaps the simplest way to determine if a patient has a patent airway is to:
A. Auscultate for breath sounds
B. Check for adequate chest rise
C. Say “hello”
D. Determine a respiratory rate
C. Say “hello”

What is the sound of the soft tissue of the upper airway creating impedance or partial obstruction to the flow of air?
A. Stridor
B. Gurgling
C. Hoarseness
D. Snoring
D. Snoring

Your patient is breathing 4 shallow breaths per minutes due to overdosing on his pain medication but he has a palpable radial pulse. He vomited prior to your arrival and is choking. You should:
A. Insert an oropharyngeal airway and ventilate
B. Move the patient to the ambulance and suction
C. Roll him over onto his side to clear his airway
D. Perform chest thrusts to clear the lungs
C. Roll him over onto his side to clear his airway

Which of the following patients should NOT have their airway opened using a head tilt chin lift maneuver?
A. A 25 year old man who is still unresponsive after a grand mal seizure
B. A homeless person of undetermined age found lying unresponsive in an alley with no bystanders
C. A 50 year old woman who choked on a piece of food while dining in a restaurant and was lowered to the floor by a waiter
D. A 35 year old diabetic woman who is in the driver’s seat of the car in her driveway, who becomes unresponsive while speaking to her husband
B. A homeless person of undetermined age found lying unresponsive in an alley with no bystanders

The jaw thrust maneuver is the only _ airway procedure for an unconscious patient with possible head, neck, or spine injury or unknown mechanism of injury
A. Recommended
B. Prohibited
C. Required
D. Forbidden
A. Recommended

Which of the following is a disadvantage of oropharyngeal airways (OPAs)?
A. They do not come in pediatric sizes
B. They required the use of water soluble lubricant
C. The cannot be used in a patient with a gag reflex
D. They cannot be used in patients with a suspected skull fracture
C. The cannot be used in a patient with a gag reflex

Which of the following is an advantage of using a nasopharyngeal airway (NPA)?
A. It eliminates the need for manual positioning of the patient’s head to keep the airway open
B. It may be tolerated by many patients with a gag reflex
C. It is ideal for patients with a suspected skull fracture
D. All of the above
B. It may be tolerated by many patients with a gag reflex

Which of the following structures is found in the lower airway?
A. Bronchi
B. Uvula
C. Tonsils
D. Pharynx
A. Bronchi

What is the danger that an altered mental status can pose to a patient’s breathing?
A. Depressed alveolar function
B. Hyperoxia
C. Bronchospasm
D. Loss of muscle tone and airway collapse
D. Loss of muscle tone and airway collapse

For life to be maintained, a balance of oxygen and carbon dioxide is needed. The condition when oxygen levels are low is called:
A. Hypoxia
B. Hypercarbia
C. Hypotension
D. Hyperventilation
A. Hypoxia

What are the signs of hypoxia?
A. Commonly seen as blue or gray skin, deterioration of patient’s mental status like confusion or restlessness
B. Shock caused from the lack of blood flowing to the vital organs like the brian and heart that is irreversible
C. Warm dry skin, with difficulty in breathing, and hypertension
D. Disease process that robs the patient of adequate breathing and perfusion
A. Commonly seen as blue or gray skin, deterioration of patient’s mental status like confusion or restlessness

What signs and symptoms would indicate inadequate breathing in a patient?
A. Increased effort to breathe, increased depth of respiration, pink dry skin, normal mental status
B. Increased effort to breathe, cyanosis, cool clammy skin, altered mental status
C. Decreased depth of respiration, decreased rate of breathing, hot clammy skin, normal mental status
D. Rapid breathing, pale skin, and a normal mental status
B. Increased effort to breathe, cyanosis, cool clammy skin, altered mental status

In assessing a patient’s breathing, what is your first question?
A. Is he alive or dead?
B. Is he breathing?
C. Is his breathing adequate or inadequate?
D. Is he big sick of little sick?
B. Is he breathing?

Why is inhalation described as an active process?
A. It uses oxygen to assist chest muscles to contract, creating a negative pressure
B. It requires chest muscles to relax and use energy to move, creating a positive pressure
C. It requires the diaphragm to relax and use energy to move, creating a positive pressure
D. It requires chest muscles to contract and use energy to move, creating a negative pressure
D. It requires chest muscles to contract and use energy to move, creating a negative pressure

The process of air moving in and out of the chest is called:
A. Tidal volume
B. Ventilation
C. Inhalation
D. Respiration
B. Ventilation

The normal stimulus to breathe is stimulated by the chemoreceptors that measure the change of what two gases?
A. High carbon monoxide and low oxygen
B. High hydrogen and low carbon dioxide
C. High carbon dioxide and low oxygen
D. Low hydrogen and high carbon monoxide
C. High carbon dioxide and low oxygen

Which of the following describes why fast respiration may decrease minute volume?
A. The lungs may not have the time to fill and exchange gas
B. It is due to the delay in the movement of the intercostal muscles and the pleural space
C. The rate causes turbulence in the trachea that increases the friction and decreases the amount of air movement
D. The rate does not decrease minute volume, it actually increases
A. The lungs may not have the time to fill and exchange gas

To calculate the minute volume, you need to multiply what two measurements?
A. Respiratory rate and bronchial dilation
B. Tidal volume and dead space air
C. Alveolar ventilation and respiratory rate
D. Tidal volume and respiratory rate
D. Tidal volume and respiratory rate

Which of the following is NOT determined in a scene size up ?
A. Mechanism of injury
B. Potential hazards to the EMS crew
C. Need for additional resources
D. Chief complaint
D. Chief complaint

At what point is the scene size up complete ?
A. Upon stabilization of the c spine
B. At the end of the call
C. When the number of patients has been determined
D. When crashed vehicles have been stabilized
B. At the end of the call

At which of the following points should you begin your scene size up?
A. When you arrive on the scene, but before exiting the ambulance
B. As you approach the scene in the ambulance
C. When the patient or family member opens the door to the residence
D. After exiting the ambulance, but before making patient contact
B. As you approach the scene in the ambulance

Which of the following situations requires additional action by the EMT during scene size up?
A. The sound of a barking and growling dog upon approaching the door to a residence
B. A bystander who is smoking a cigarette at the scene of an assault at a local park
C. A news media helicopter arrives and hovers overhead at the scene of a vehicle collision
D. A vehicle collision involving a tractor trailer that appears to be empty
A. The sound of a barking and growling dog upon approaching the door to a residence

You are called to a motor vehicle collision where the car is on fire. You should ensure safety by:
A. Borrowing turnout gear from the fire department
B. Putting your unit back in service and leaving the scene
C. Remaining a safe distance from the car until the fire is out
D. Using your fire extinguisher to put out the fire
C. Remaining a safe distance from the car until the fire is out

As you arrive at the scene of a house fire, a very upset man screams at you to help his young son, who is trapped under a piece of burning wood on the ground. Which of the following should you do first?
A. Perform an initial assessment on the patient
B. Size up the scene before acting
C. With the father’s help grab the boy by the arms and pull him from underneath the wood
D. Use a blanket to put out the fire on the piece of wood
B. Size up the scene before acting

At the scene of a vehicle collision in which there are no apparent hazards, which of the following guidelines should be followed for establishing a danger zone?
A. The danger zone should be 150 feet in all directions
B. The danger zone should be 15 feet in all directions
C. The danger zone should be 50 feet in all directions
D. There is no need to establish a danger zone when there are no apparent hazards
C. The danger zone should be 50 feet in all directions

When the heart contracts and forces blood into the arteries, the pressure created is known as the:
A. Diastolic blood pressure
B. Systolic blood pressure
C. Pulse pressure
D. Central venous pressure
B. Systolic blood pressure

What is the pressure remaining in the arteries after the pulse wave has passed through?
A. Resting blood pressure
B. Systolic blood pressure
C. Venous pressure
D. Diastolic blood pressure
D. Diastolic blood pressure

A patient with a pulse rate of 120 beats per minute is considered which of the following?
A. Normocardic
B. Tachycardic
C. Bradycardic
D. Dyscardic
B. Tachycardic

The abbreviation mmHg indicates that the blood pressure is measured by which of the following comparisons?
A. Millimeters of mercury
B. Millimeters of water
C. Minimum heart rate
D. Atmospheric pressure
A. Millimeters of mercury

The first set of vital sign measurements obtained are often referred to as which of the following?
A. Normal vital signs
B. Standard vital signs
C. Baseline vital signs
D. None of the above
C. Baseline vital signs

An oxygen saturation of 97% is considered which of the following?
A. Significant hypoxia
B. Hypoxia
C. Severe hypoxia
D. Normal
D. Normal

In a blood pressure reading of 120/80, the 120 is measuring what body process?
A. Systolic blood pressure; when the right ventricles contract and the blood is forced into the veins
B. Systolic blood pressure; when the left ventricles contract and the blood is forced into the veins
C. Systolic blood pressure; when the left ventricles contract and the blood is forced into the arteries
D. Diastolic blood pressure; when the left ventricles contract and the blood is forced into the arteries
C. Systolic blood pressure; when the left ventricles contract and the blood is forced into the arteries

Which of the following are the vital signs that need to be recorded?
A. Pulse, respiration, skin color, skin temperature and condition, pupils, blood pressure, and bowel sounds
B. Pulse, respiration, skin color, skin temperature, pupils, and blood pressure
C. Pulse, respiration, skin color, skin temperature and condition
D. Pulse, respiration, skin color, skin temperature and condition, pupils, and blood pressure
D. Pulse, respiration, skin color, skin temperature and condition, pupils, and blood pressure

Breathing sounds that should concern the EMT are:
A. Snoring, gurgling, wheezing, and crowing
B. Tachycardia, retractions, and diaphragmatic breathing
C. Snoring, gurgling, wheezing, crowing, and crowning
D. Retractions, and diaphragmatic breathing
A. Snoring, gurgling, wheezing, and crowing

Which of the following is NOT a consideration that should be used by the EMT in establishing the size of the danger zone?
A. Presence of hazardous materials
B. Wind direction
C. Fire
D. Amount of equipment needed
D. Amount of equipment needed

You are on the scene of a tanker truck versus passenger vehicle collision on a rural highway. The vehicles are just beyond a curve in the roadway and there is a distinct odor of diesel fuel. It is dark and there is little traffic. Which of the following should be used to alert oncoming traffic to the situation?
A. Flashing lights on the ambulance
B. Flares
C. Yellow crime scene tape
D. Reflective triangles
D. Reflective triangles

The patient was a driver in a lateral impact motor vehicle collision. During the assessment of his chest, the EMT notes a segment of the chest wall moving in the opposite direction form the rest of the chest. Which of the following BEST describes this finding?
A. Tension pneumothorax
B. Paradoxical movement
C. Flutter segment
D. Intercostal retractions
B. Paradoxical movement

When assessing a 14 year old male patient that has been involved in a bicycle accident, you notice that he has a small amount of blood coming form his left forearm. This observation is known as which of the following?
A. Clue
B. Indication
C. Symptom
D. Sign
D. Sign

What is the sound or feel of broken bones rubbing against each other called?
A. Osteomyelitis
B. Crepitation
C. Emesis
D. Decapitation
B. Crepitation

What term describes a permanent surgical opening in the neck through which a patient breathes?
A. Cricothyrotomy
B. Larygectomy
C. Stoma
D. Tracheostomy
C. Stoma

What term describes a surgical incision in the neck that is held open by a metal or plastic tube through which a patient can breathe or be placed on a ventilator?
A. Tracheopharygeal fistula
B. Tracheostomy
C. Stoma
D. Cricothyrotomy
B. Tracheostomy

When a patient describes how he feels, he is telling you which of the following?
A. His diagnosis
B. His signs
C. His symptoms
D. His syndrome
C. His symptoms

You are assessing a 76 year old male patient that has been involved in a fall from a standing position. You have completed the scene size up and primary assessment. What should you do next?
A. Focused history assessment
B. Reassessment
C. Ongoing assessment
D. Secondary assessment
D. Secondary assessment

You have a patient who is unresponsive on the floor. What is the best way to rule in or rule out trauma as a cause of the patient’s unresponsiveness?
A. Check the patient’s blood sugar to rule out hypoglycemia
B. Look for bystanders and ask them if they witnessed the incident
C. Examine the patient for signs of trauma
D. Look for a Glasgow Coma Scale score that is less than 8
B. Look for bystanders and ask them if they witnessed the incident

Which of the following techniques of physical examination must an EMT master?
A. Observation, palpation, and auscultation
B. Visualization, percussion, and auscultation
C. Percussion, inspection, and palpation
D. Auscultation, observation, and percussion
A. Observation, palpation, and auscultation

Which of the following methods should be used to have a patient rate the amount of pain he is having?
A. Use the memory aid DCAP
B. Use the memory aid AVPU
C. Have the patient rate the pain on a scale of 1 (least) to 10 (worst)
D. Ask the patient to state whether the pain is mild, moderate, severe, or unbearable
C. Have the patient rate the pain on a scale of 1 (least) to 10 (worst)

Which of the following BEST describes a base station?
A. A two way radio mounted in a vehicle
B. A two way radio at a fixed site
C. A two way radio that can be carried on a belt clip
D. A device used to receive and then amplify transmissions that must be carried over long distances
B. A two way radio at a fixed site

Which of the following BEST describes a mobile radio?
A. A two way radio mounted in a vehicle
B. A two way radio at a fixed site
C. A two way radio that can be carried on a belt clip
D. A device that receives and amplifies signals that must be sent over long distance
A. A two way radio mounted in a vehicle

Which of the following BEST describes a portable radio?
A. A two way radio that can be carried on a belt clip
B. A two way radio mounted in a vehicle
C. A two way radio at a fixed site
D. A device that receives and amplifies signals that must be sent over long distance
A. A two way radio that can be carried on a belt clip

Which of the following BEST describes a repeater?
A. A two way radio mounted in a vehicle
B. A two way radio at a fixed site
C. A two way radio that can be carried on a belt clip
D. A device that receives and amplifies signals that must be sent over long distance
D. A device that receives and amplifies signals that must be sent over long distance

The symptoms or circumstances for which a medication is given is called:
A. Side effects
B. Untoward effects
C. Indications
D. Contraindications
C. Indications

When you give patients nitroglycerin, they sometimes develop a headache. This would be called:
A. Contraindication
B. Indication
C. Side effect
D. Untoward effect
C. Side effect

Which of the following BEST describes a contraindication to a medication?
A. A reason why you should never give a medication to a patient
B. The way in which a drug causes its effects
C. An unintended action of the drug
D. A reason why you should give a medication to a patient
A. A reason why you should never give a medication to a patient

What condition must be present before you give oral glucose?
A. The patient, if conscious, must be able to swallow; in unconscious, you can apply the gel to a tongue depressor and place it between the cheek and gum or under the tongue
B. The patient must be conscious and able to swallow with an altered mental status and history of diabetes
C. The patient must be unconscious and have a history of diabetes
D. The patient must not have a history of diabetes
B. The patient must be conscious and able to swallow with an altered mental status and history of diabetes

Why should EMT’s study pharmacology?
A. As an EMT, you will be trusted to administer medications in emergency situations, although many of these may do nothing but give the patient false hope
B. As an EMT, you will be trusted to administer medications in emergency situations; many of these may be lifesaving, but there is potential to do harm
C. An EMT must know the manufacturer, sources, characteristics, and effects of every medication that has been prescribed to the patient
D. An EMT must know the sources, characteristics, and effects of each medication that the physician may prescribe
B. As an EMT, you will be trusted to administer medications in emergency situations; many of these may be lifesaving, but there is potential to do harm

Which of the following does NOT occur during inspiration?
A. Intercostal muscles contract
B. Diaphragm relaxes
C. Chest cavity increases in size
D. Diaphragm lowers
B. Diaphragm relaxes

Which of the following is true concerning expiration?
A. The ribs move upward and outward
B. The intercostal muscles contract to force air out of the lungs
C. The diaphragm moves upward
D. The chest cavity increases in size
C. The diaphragm moves upward

Which of the following respiratory rates is considered an abnormal respiratory rate for an adult?
A. 16 breaths/min
B. 12 breaths/min
C. 8 breaths/min
D. 20 breaths/min
C. 8 breaths/min

A patient who has shallow, slow, irregular gasping breaths is said to have __ respirations?
A. Agonal
B. Kussmauls
C. Central neurogenic
D. Cheyne-Stokes
A. Agonal

Which of the following respiratory rtes should be caused for alarm in a 2 month old child?
A. 16 breaths/min
B. 40 breaths/min
C. 28 breaths/min
D. 32 breaths/min
A. 16 breaths/min

Which of the following BEST defines inadequate breathing?
A. Breathing faster than normal
B. Breathing slower than normal
C. Wheezing noises when breathing
D. Breathing that is insufficient to sustain life
D. Breathing that is insufficient to sustain life

Which of the following may be seen just prior to respiratory arrest?
A. Accessory respirations
B. Agonal respirations
C. Very deep, rapid respirations
D. Breathing through the nose, not the mouth
B. Agonal respirations

Which of the following patients does NOT necessarily have inadequate breathinG?
A. Patient with an irregular respiratory rhythm
B. Patient with agonal respirations
C. Patient with cyanosis
D. Patient whose breath sounds cannot be heard
A. Patient with an irregular respiratory rhythm

While assessing the airway of a pediatric patient, you will notice that it is different than that of an adult. Which of the following is one of those differences?
A. The chest wall is softer, making it easier for the chest to expand
B. The tongue is smaller, taking up less room in the mouth and allowing larger objects to occlude the airway
C. The trachea is smaller, softer, and more flexible, allowing it to be more easily obstructed
D. The cricoid cartilage is less developed, reducing the possibility that it can be completely occluded
C. The trachea is smaller, softer, and more flexible, allowing it to be more easily obstructed

While caring for a 3 year old child, you should be concerned if his respiratory rate exceeds _ breaths per minutes
A. 16
B. 20
C. 30
D. 24
C. 30

Which of the following signs of inadequate breathing is more prominent in children than in adults?
A. See sawing of the chest and abdomen
B. Nasal flaring
C. Grunting respirations
D. All of the above
D. All of the above

Your patient is a 15 year old male with a history of multiple prior hospitalizations for asthma. Upon your arrival the patient responds only to painful stimuli and is making very weak respiratory effort. Which of the following should you do next?
A. Assist the patient with his inhaler
B. Contact medical control
C. Check the patient’s oxygen saturation level
D. Assist the patient’s ventilations with a bag valve mask device and supplemental oxygen
D. Assist the patient’s ventilations with a bag valve mask device and supplemental oxygen

Which of the following is a sign of adequate artificial ventilations in a pediatric patient?
A. You notice cyanosis developing around the mouth
B. Pulse rate increases
C. The breath is delivered easily
D. Pulse rate slows down
B. Pulse rate increases

When ventilating a child with inadequate respirations, which of the following is the maximum rate at which artificial respirations should be delivered?
A. 24 per minute
B. 20 per minute
C. 12 per minute
D. 15 per minute
B. 20 per minute

What is the best way to determine that you are getting adequate ventilation with a bag valve mask?
A. Push the pull amount of the bag into the patient
B. Ensure the pulse oximeter reads 95 to 100%
C. Look for chest rise and fall
D. Hyperventilate the patient until the oxygen saturation reaches 100%
C. Look for chest rise and fall

Which of the following patients with difficulty breathing should NOT receive supplemental oxygen?
A. A patient whose oxygen saturation level is 100% on room air
B. A patient with a chronic lung disease who may have a hypoxic drive
C. An infant whose eyes may be damaged by excessive oxygen administration
D. None of these patients should have oxygen withheld
D. None of these patients should have oxygen withheld

Which of the following is the general term used to refer to a problem with the heart?
A. Congestive heart failure
B. Myocardial infarction
C. Cardiac compromise
D. Cardiac dysrhythmia
C. Cardiac compromise

What is NOT part of the cardiovascular system?
A. Arteries
B. Hormones
C. Venules
D. Veins
B. Hormones

Which of the following BEST describes a fluttering sensation in the chest?
A. Palpitations
B. Pulseless electrical activity of the heart
C. Tachycardia
D. Dysrhythmia
A. Palpitations

Which of the following statements concerning heart attacks and cardiac arrest is NOT true?
A. Many patients may mistake their symptoms for other causes such as indigestion
B. Heart attacks present differently among women and men
C. The most common initial rhythm in sudden cardiac death is asystole
D. Some patients who have heart attacks live active and healthy lifestyles
C. The most common initial rhythm in sudden cardiac death is asystole

You are treating a patient with signs and symptoms of a myocardial infarction (MI). What is the most important drug you should administer?
A. Aspirin
B. Albuterol
C. Oxygen
D. Epinephrine auto- injector (Epi-Pen)
C. Oxygen

You are on scene with a 48 year old unresponsive male patient. Bystanders state he complained of chest pain and then suddenly collapsed. Vital signs are blood pressure 68/42, pulse 36, and respiratory rate 3. He is unresponsive to painful stimuli. After performing the primary assessment and treating all life threats, what is your next intervention?
A. Insert an oral airway
B. Apply the AED
C. Package the patient for rapid transport
D. Administer high-concentration oxygen at 15 lpm by bag valve mask
C. Package the patient for rapid transport

Assuming you protocol allows the administration of nitroglycerin when certain conditions exist, what is the maximum number of tablets to be administered in the pre hospital setting?
A. 3
B. 4
C. 1
D. 2
A. 3

Which of the following is the beneficial action of nitroglycerin in some cardiac emergencies?
A. It dilates only the coronary arteries
B. It relaxes blood vessels throughout the body
C. It increases the strength with which the ventricles contract
D. It slows down the heart
B. It relaxes blood vessels throughout the body

Which of the following statements regarding the administration of nitroglycerin tablets is true?
A. The patient may complain of a headache following administration
B. An increase in blood pressure should be expected
C. If the patient’s pulse rate changes following nitroglycerin administration, it indicates an allergic reaction
D. It takes 20 to 30 minutes for nitroglycerin to have an effect
A. The patient may complain of a headache following administration

Which of the following is the EMT’s role in caring for a patient with chest pain?
A. Make an interpretation of the patient’s cardiac rhythm
B. Determine the cause of the patient’s chest pain
C. Treat the patient as though he were having a heart attack
D. Provide automatic external defibrillation
C. Treat the patient as though he were having a heart attack

In which of the following situations would the administration of aspirin to a cardiac patient be prohibited?
A. The patient does not currently take aspirin
B. The patient has a diastolic blood pressure greater than 90 mmHg
C. The patient has a history of asthma
D. The patient feels dizzy
C. The patient has a history of asthma

Which of the following is the name of the condition in which fatty deposits form in the inner lining of the arteries?
A. Coronary artery disease
B. Aneurysm
C. Coronary thrombosis
D. Arteriosclerosis
A. Coronary artery disease

The medical term for fainting is:
A. Altered RAS status
B. Vertigo
C. Dehydration
D. Syncope
D. Syncope

Which of the following BEST describes status epilepticus?
A. Two or more seizures with tonic-clonic activity without an intervening period of consciousness
B. A period of drowsiness following tonic-clonic seizures
C. A seizure that occurs without a known cause
D. A seizure involving convulsions on only one side of the body
A. Two or more seizures with tonic-clonic activity without an intervening period of consciousness

Your patient is waking up from a seizure; it was the patient’s first seizure ever. When you ask what happened, the patient tells you that she had the smell of fresh mown grass just before she seized. This sensation is known as:
A. Aura
B. Tonic phase
C. Clonic phase
D. Postictal phase
A. Aura

The death of brain tissue due to deprivation of oxygen because of a blocked or ruptured artery in the brain is known as which of the following?
A. Transient ischemic attack
B. Seizure
C. Aphasia
D. Stroke
D. Stroke

A hormone called insulin is secreted by the:
A. Islets of Langerhans in the pancreas
B. Gallbladder found in the pancreas
C. Islets of Langerhans in the liver
D. None of the above
A. Islets of Langerhans in the pancreas

There are two types of seizures; if your patient is having a seizure that affects only one body part and does not cause her to lose consciousness, it is called a:
A. Postictal seizure
B. Partial seizure
C. Tonic- clonic seizure
D. Generalized seizure
B. Partial seizure

The condition in which there is an insufficient amount of sugar in the blood is called:
A. Hyperglycemia
B. Diabetic ketoacidosis
C. Diabetic coma
D. Hypoglycemia
D. Hypoglycemia

Your patient is a 25 year old female with a history of diabetes. She is confused, agitated, and verbally abusive to you, and she is very sweaty. Although she refuses to give a history of the present illness, you should suspect which of the following as the likely cause of the patient’s presentation?
A. A nondiabetic related problem, such as a head injury or mental illness
B. Failure to intake sufficient sugar
C. Failure to take her insulin
D. Excessive intake of foods high in sugar, such as soda or candy
B. Failure to intake sufficient sugar

For the reticular activating system (RAS) to work correctly, what three substances are needed?
A. Oxygen to perfuse brain tissue, insulin to nourish brain tissue, and water to keep the brain hydrated
B. Oxygen to perfuse the brain tissue, glucose to nourish brain tissue, and water to keep the brain hydrated
C. Oxygen to perfuse brain tissue, glucose to nourish brain tissue, and sodium to keep the brain hydrated
D. Oxygen to perfuse the brain tissue, insulin to nourish brain tissue, and sodium to keep he brain hydrated
B. Oxygen to perfuse the brain tissue, glucose to nourish brain tissue, and water to keep the brain hydrated

Normal consciousness is regulated by a series of neurologic circuits in the brain that comprise the reticular activating system (RAS). The RAS has simple requirements to function properly. Which one of the following items is NOT one of those requirements?
A. Sodium
B. Glucose
C. Water
D. Oxygen
A. Sodium

What is another name for a severe allergic reaction?
A. Psychosomatic reaction
B. Asthma
C. Dermatitis
D. Anaphylaxis
D. Anaphylaxis

Which of the following statements concerning severe allergic reactions is true?
A. The quicker the onset of symptoms, the greater the likelihood of a severe allergic reaction
B. A severe allergic reaction can be prevented by the use of an epinephrine auto- injector before exposure to the substance
C. A severe allergic reaction occurs only when the patient has never been exposed to the substance before
D. Allergies do not develop until a person is in his late teens to early 20s
A. The quicker the onset of symptoms, the greater the likelihood of a severe allergic reaction

Which of the following is NOT a sign or symptom of an allergic reaction involving the skin?
A. Flushing
B. Cyanosis
C. Itching
D. Hives
B. Cyanosis

The term poison is BEST described as any substance that can do which of the following?
A. Increase cellular activity
B. Harm the body
C. Deactivate nerve transmission
D. Depress the body’s respirations
B. Harm the body

Which of the following describes any substance produced by a living organism that is poisonous to human beings?
A. Toxin
B. Antibody
C. Antigen
D. Acid
A. Toxin

You are called to a farmhouse on a sunny spring afternoon. Upon arrival, a frantic mother tells you that her 3 year old daughter was playing in the yard, accidentally stepped on a hornet’s nest, and was stung repeatedly. The patient is alert, screaming and crying, and can follow her mother’s commands. Her arms and legs are swollen and show the marks of several stinger. Her body is covered in hives. Her vital signs are blood pressure 90/40, pulse 110, respiratory rate 24, and oxygen saturation 99%. Her lung sounds are clear and equal bilaterally. Her mother states she put “nox-a-sting” on the bites but the bites only seemed to get worse. What condition is the patient suffering from?
A. Moderate anaphylactic reaction
B. Allergic reaction from the hornet’s stings
C. Anaphylactic reaction from the “nox-a-sting” swabs
D. Minor anaphylactic reaction
B. Allergic reaction from the hornet’s stings

Which of the following is MOST likely to be a complaint of a patient suffering from anaphylaxis?
A. “My legs are numb and I can’t move them”
B. “My throat feels like it is closing”
C. “I am having severe muscle pains”
D. “I can’t remember what happened”
B. “My throat feels like it is closing”

Pain that originates in an organ, such as intestines, is called __ pain.
A. Parietal
B. Visceral
C. Acute
D. Referred
B. Visceral

The patient complaining of parietal abdominal pain would concern you the MOST is the one who:
A. Is lying on the floor very still and quiet with his knees drawn up to his chest
B. Is rolling about on the floor complaining of pain
C. Walks out to the ambulance informing you he has the worst “belly ache”
D. Is sitting upright in a chair, moaning in pain, and drinking antacid
A. Is lying on the floor very still and quiet with his knees drawn up to his chest

Your patient was diagnosed with cholecystitis (gallbladder inflammation) three days ago. The patient now presents with nausea, vomiting, and pain in the right shoulder. The pain in the shoulder can be classified as:
A. Somatic pain
B. Referred pain
C. Pancreatic pain
D. Visceral pain
B. Referred pain

Shock is the circulatory system’s failure to provide sufficient blood and oxygen to all the body’s tissues. Which of the answers is NOT a major type of shock?
A. Hypovolemic
B. Hypervolemic
C. Hemorrhagic
D. Cardiogenic
B. Hypervolemic

Which of the following vessels has the thickest muscular walls that allow constriction and dilation?
A. Capillaries
B. Arteries
C. Veins
D. Lymphatic vessels
B. Arteries

Which of the following types of vessels have valves to maintain one way blood flow?
A. Arterioles
B. Arteries
C. Veins
D. Capillaries
C. Veins

Which of the following BEST describes the function of blood?
A. It flows from the heart with the vital gases and nutrients to maintain lack of perfusion
B. It transports gases along with nutrients, aids in excretion, and provides protection and regulation
C. It clots, flows, transports, protects, and excretes on a daily basis
D. It is a life-giving liquid that supports all the body’s functions to maintain hypoperfusion
B. It transports gases along with nutrients, aids in excretion, and provides protection and regulation

Which of the following is the outermost layer of the skin?
A. Adipose tissue
B. Dermis
C. Epidermis
D. Fascia
C. Epidermis

You are examining a 48 year old patient who has been burned. You decide to use the rule of palms to measure the extent of the burn. What does this mean?
A.You can only use the rule of palm for children, so you would use the rule of nines instead
B. The palm of the patient’s hand equals 9% of the body’s surface area
C. The palm of your hand represents 1% of body’s surface area
D. The palm of the patient’s hand equals about 1% of the body’s surface area
D. The palm of the patient’s hand equals about 1% of the body’s surface area

Which of the following is a description of the rule of nines for an adult?
A. The rule of nines assigns 9% to the head and neck, each upper extremity, the chest, the abdomen, the upper back, and the lower back and buttocks; 8% to the front of each lower extremity; and 10% for the back of each lower extremity, then 1% to the genital region
B.The rule of nines assigns 9% to the head and neck; 6% to each upper extremity; 12% to the chest and abdomen; 9% to the upper back, the lower back and buttocks, the front of each lower extremity, and the back of each lower extremity, then 1% to the genital region
C. The rule of nines assigns 9% to the head and neck, each upper extremity, the chest, the abdomen, the upper back, the lower back, and the buttocks, the front of each lower extremity, an the back of each lower extremity, then 1% to the genital region
D. The rule of nines assigns 9% to the head and neck, as well as each upper extremity; 10% to the chest, abdomen, an the upper back; 8% to the lower back and buttocks, the front of each lower extremity, and the back of each lower extremity, then 1% to the genital region
C. The rule of nines assigns 9% to the head and neck, each upper extremity, the chest, the abdomen, the upper back, the lower back, and the buttocks, the front of each lower extremity, an the back of each lower extremity, then 1% to the genital region

Which of the following is NOT a major function of the skin?
A. Protection from the environment
B. Temperature regulation
C. Regulates the pH balance of the body
D. Excretion of wastes
C. Regulates the pH balance of the body

Which of the following layers of the skin is the MOST important in insulating the body against heat loss?
A. Subdural layer
B. Parietal layer
C. Subcutaneous layer
D. Epidermis
C. Subcutaneous layer

Which of the following BEST describes the term crowning
A. Presenting part of the baby being visible at the vaginal opening
B. Delivery of the head during a breech birth, completing delivery
C. Discharge of bloody mucus
D. Complete dilation of the cervix
A. Presenting part of the baby being visible at the vaginal opening

Which of the following describe a breech presentation?
A. The infant presents buttocks first
B. The infant presents with both feet first
C. The infant presents face first
D. Both A and B
D. Both A and B

Which of the following BEST describes placenta previa?
A. The placenta prematurely separates from the uterine wall
B. The pregnancy is lost before the 20th week of gestation
C. The placenta is implanted over the opening of the cervix
D. The umbilical cord is the presenting part
C. The placenta is implanted over the opening of the cervix

The condition in which the placenta separates from the uterine wall is known as which of the following?
A. Ectopic pregnancy
B. Preeclampsia
C. Placenta previa
D. Abruptio placentae
D. Abruptio placentae

You are assessing a pregnant woman whose chief complaint is vaginal bleeding. She is 8 months pregnant and has moderate, bright red bleeding. She says that her doctor was concerned about the location of the placenta. This condition is known as:
A. Unstable placenta
B. Placenta disruption
C. Placenta previa
D. Abruptio placenta
C. Placenta previa

The anterior fontanelle should be closed between the ages of _ and _.
A. 6 months; 10 months
B. 12 months; 18 months
C. 3 months; 6 months
D. 24 months; 36 months
B. 12 months; 18 months

You are at a friend’s birthday party with people of all ages. There is an 11 month old boy, just learning to stand up, who has fallen. He was holding onto the edge of a table and he toppled over. He is crying hard and his mother is trying to console him. She asks if you would mind checking him out, and you agree to take a look at him. You notice that his anterior fontanelle is bulging. This is most likely caused by:
A. Fluid loss from dehydration
B. An infection causing intracranial pressure
C. The bones of the head not fusing properly
D. Pressure built up because of his crying
D. Pressure built up because of his crying

In general, a child is considered an adolescent when he reaches the age of _ years.
A. 9
B. 7
C. 4
D. 12
D. 12

Which of the following should be considered by the EMT as a result of the proportionally larger size of a small child’s head.?
A. Different injury patterns than in adults
B. Differences in positioning to maintain an open airway
C. Adaptations in positioning for cervical spine immobilization
D. All of the above
D. All of the above

The maxillae form which of the following structures?
A. Upper jaw
B. Lower jaw
C. Forehead
D. Cheek bones
A. Upper jaw

What is another name for the zygomatic bone?
A. Maxillae
B. Mandible
C. Temporal
D. Malar
D. Malar

The bony bumps you feel along the center of a person’s back are known as which of the following?
A. Foremen magnum
B. Transverse processes
C. Spinous process
D. Vertebrae
C. Spinous process

Which of the following is defined as area of the body surface that is innervated by a single spinal nerve?
A. Spinous process
B. Malar
C. Transverse process
D. Dermatome
D. Dermatome

Messages from the body to the brain are carried by which of the following types of nerves?
A. Sensory
B. Cranial
C. Skeletal
D. Motor
A. Sensory

Which of the following systems includes the pairs of nerves that enter and exit the spinal cord between each pair of vertebrae?
A. Peripheral nervous system
B. Autonomic nervous sytem
C. Central nervous system
D. All of the above
A. Peripheral nervous system

Which of the following is a function of the autonomic nervous system?
A. Constriction of blood vessels
B. Running or walking
C. Speaking
D. Soling complex math problems
A. Constriction of blood vessels

To which of the following sections of the spine are the ribs attached?
A. Lumbar
B. Sacral
C. Thoracic
D. Cervical
C. Thoracic

How many cervical vertebrae are there?
A. 7
B. 4
C. 5
D. 12
A. 7

Which of the following injuries is considered an indirect brain injury?
A. Depressed skull fracture with cerebral penetration by bone fragments
B. Concussion
C. Cerebral laceration
D. Gunshot wound to the head
B. Concussion

Which of the following is classified as an open head injury?
A. Laceration with a skull fracture
B. Contusion without a skull fracture
C. Laceration without a skull fracture
D. Both A and B
A. Laceration with a skull fracture

C. National Highway Transportation Safety Administration (NHTSA).
Most EMT training programs are based on standards developed by the:
A. American Red Cross (ARC).
B. American Heart Association (AHA).
C. National Highway Transportation Safety Administration (NHTSA).
D. National Institutes of Health (NIH).

D. being pleasant, cooperative, and sincere, and a good listener.
An EMT can inspire patient confidence and cooperation by:
A. transporting the patient from the scene to a hospital.
B. providing patient care without regard for her own personal safety.
C. telling the patient that everything will be all right.
D. being pleasant, cooperative, and sincere, and a good listener.

C. negligence.
If an on-duty EMT fails to provide the standard of care and if this failure causes harm or injury to the patient, the EMT may be accused of:
A. assault.
B. abandonment.
C. negligence.
D. breach of promise.

D. informed
You are treating a conscious and mentally competent adult patient who wants to refuse your care and transport to the hospital. This refusal must be _ and documented.
A. implied
B. actual
C. involuntary
D. informed

B. implied
The EMT is authorized to treat and transport an unconscious patient because of the legal consideration known as _ consent.
A. applied
B. implied
C. triage
D. immunity

D. meaningless.
A child falls of a trampoline at an elementary school and twists her ankle. Because the parents are not present, the child’s consent is
A. not needed.
B. actual.
C. implied.
D. meaningless.

C. Good Samaritan laws
In some states, _ help protect the off-duty EMT from lawsuits when stopping at the scene of a collision to offer assistance.
A. professional associations
B. blanket insurance policies
C. Good Samaritan laws
D. abandonment laws

A. entering hazmat scenes with SCBA.
Each of the following is the responsiblity of an EMT at a hazardous-materials (hazmat) incident except:
A. entering hazmat scenes with SCBA.
B. protecting yourself and others.
C. recognizing potential problems.
D. notifying the hazardous-materials response team.

B. Standard Precautions.
The form of infection control that assumes that all body fluids should be considered potentially infectious is:
A. infectious disease.
B. Standard Precautions.
C. immunity.
D. universal precautions.

D. the distance the patient needs to be carried.
When planning to lift a patient, all of the following are important considerations for the EMT except:
A. the weight of the patient.
B. one’s physical characteristics.
C. communicating with one’s partner.
D. the distance the patient needs to be carried.

C. use the leg muscles to do the lift.
When lifting an injured patient, the EMT should:
A. keep the back loose and knees locked.
B. twist or attempt to make moves other than the lift.
C. use the leg muscles to do the lift.
D. try not to talk to her or his partner.

C. use a stair chair whenever possible.
You are treating a 45-year-old male who twisted his ankle in the upstairs bathroom. To carry this patient on the stairs, you should:
A. keep the stretcher as level as possible.
B. use a long backboard at all times.
C. use a stair chair whenever possible.
D. do all of these.

C. keep arms straight when pulling.
Ways an EMT can avoid a potential back injury include all of the following except:
A. push, rather than pull, a load.
B. keep the back locked in while lifting.
C. keep arms straight when pulling.
D. push or pull from a kneeling position if the weight is below waist level.

D. the patient is unconscious.
An emergency move is required in each of the following situations except when:
A. the scene is hazardous.
B. care of life-threatening conditions requires repositioning.
C. other patients who have life threats must be reached.
D. the patient is unconscious.

A. A spinal injury may be aggravated.
Which of the following is the greatest danger to the patient in an emergency move?
A. A spinal injury may be aggravated.
B. Bleeding may increase after movement.
C. The airway may become obstructed.
D. There is no danger associated with an emergency move.

B. extremity lift.
A method of lifting and carrying a patient in which one EMT slips hands under the patient’s armpits and grasps the wrists while another EMT grasps the patient’s knees is called the:
A. direct ground lift.
B. extremity lift.
C. draw-sheet method.
D. direct carry method.

B. modified draw-sheet method.
Your patient is a medical patient with a suspected drug overdose. He is lying supine on your stretcher, and you have been maintaining his airway. When moving him from the ambulance stretcher to the hospital stretcher, you will probably use the:
A. crade carry.
B. modified draw-sheet method.
C. direct ground lift.
D. extremity lift.

A. opposite sides
To load the wheeled ambulance stretcher into the ambulance, the two EMTs should position themselves on _ of the stretcher.
A. opposite sides
B. opposide ends
C. the same side
D. one end and one side

B. extremity lift.
You are treating a 28-year-old conscious diabetic who has an altered level of consciousness. She is lying on the floor at the moment. To move her from the floor to a stair chair, use the:
A. indirect carry.
B. extremity lift.
C. slide transfer.
D. chair lift.

A. do not protect the patient’s neck and spine.
Drags are used only in emergencies because they:
A. do not protect the patient’s neck and spine.
B. require excessive energy from the EMT.
C. may injure the EMT’s back.
D. provide full immobilization.

B. an even number of
To maintain balance when lifting a patient-carrying device, it is best to use _ rescuers to carry the device.
A. three
B. an even number of
C. an odd number of
D. bystanders and

B. make an urgent move.
You are treating the driver of a vehicle involved in a collision. He is a 22-year-old male who requires immediate airway and bleeding control. You are unable to provide this treatment in the vehicle. You should:
A. check the patient’s vital signs.
B. make an urgent move.
C. remove the patient on a short backboard.
D. do all of these.

D. Piggyback
Your patient is an unconscious adult female stroke patient. Which carry is considered very difficult to use with an unconscious person?
A. Cradle
B. Three-rescuer
C. Shoulder
D. Piggyback

C. firefighter’s
The _ carry must be performed in one unbroken sweep.
A. pack strap
B. front piggyback
C. firefighter’s
D. four-rescue

C. flexible
A canvas or rubberized stretcher that can be used to move a patient through a narrow hallway or restricted are is called a _ stretcher.
A. basket
B. portable
C. flexible
D. wheeled-ambulance

B. clothes drag.
A patient with obvious spinal injuries is found on the floor of a burning building. The EMT rates the situation hazardous but not yet dire. Alone and without special equipment, the EMT should use the:
A. cradle carry.
B. clothes drag.
C. firefighter’s carry.
D. pack-strap method.

C. wheeled ambulance
The patient-carrying device of choice for the 20-year-old male who is dizzy by not injured is the _ stretcher.
A. portable ambulance
B. wire basket
C. wheeled ambulance
D. slat

D. All of these are reasons to serve as an advocate.
If you are an EMT with a service that does not provide the appropriate personal protective equipment, why should you serve as an advocate for this equipment?
A. Your crew members could be injured unneccessarily.
B. You could be seriously injured.
C. An injured EMT is of little help to the patient.
D. All of these are reasons to serve as an advocate.

A. retreat to a safe area.
During an EMT call, a lethal threat is made by the 24-year-old intoxicated male. The EMT should first:
A. retreat to a safe area.
B. radio for assistance.
C. reevaluate the situation.
D. remedy the situation.

B. Eustress
Of the different types of stress, which is a positive form that helps the EMT work under pressure and respond effectively?
A. Eumulative stress
B. Eustress
C. Distress
D. Critical incident stress

C. enter the neighborhood.
When responding to a violent situation, observation beings when you:
A. enter the scene.
B. exit the ambulance.
C. enter the neighborhood.
D. arrive at the patient’s side.

B. carry a portable radio.
To ensure crew safety, one member of the crew should always:
A. remain in the ambulance.
B. carry a portable radio.
C. wear a bulletproof vest.
D. carry a canister of pepper gas.

B. quickly control the bleeding; then have the dog locked in another room.
While you are treating a patient with a severely bleeding forearm, the patient’s pet dog appears. The patient states, “He won’t hurt you. He’s very friendly.” Your best course of action would be to:
A. have your partner observe the dog closely while you treat the patient.
B. quickly control the bleeding; then have the dog locked in another room.
C. ignore the dog because the patient assures you it is friendly and will not harm you.
D. do all of these.

C. treat life-threatening problems and transport.
If a patient refuses care and then becomes unconscious, it is best for the EMT to:
A. refuse to treat or transport the patient.
B. ask a family member for permission to treat.
C. treat life-threatening problems and transport.
D. contact medical direction for advice.

B. the patient’s expressed wishes may be followed.
An advantage of the advance directive is that:
A. the patient is not involved in making a secision about her treatment.
B. the patient’s expressed wishes may be followed.
C. no matter what the family says, CPR is not given.
D. it protects the EMT from charges of negligence.

C. not a reason to withhold medical care.
In most cases, the oral wishes of the patient’s family to withhold care are:
A. all that is needed to stop CPR from being initiated.
B. all that is needed to stop CPR once it is initiated.
C. not a reason to withhold medical care.
D. not sufficient unless they are given in writing.

D. do all of these.
Some EMTs participate in activities that attract legal actions, while most EMTs are rarely involved in legal entanglements. You can prevent most lawsuits if you:
A. provide care within the scope of your practice.
B. properly document your care.
C. are courteous and respectful to all your patients.
D. do all of these.

D. health insurance costs.
The negligent EMT may be required to pay for all of the following except the patient’s:
A. lost wages.
B. medical expenses.
C. pain and suffering
D. health insurance costs.

D. It regulates body temperature.
Which of the following is not a function of the musculoskeletal system?
A. It gives the body shape.
B. It protects the internal organs.
C. It provides for body movement.
D. It regulates body temperature.

C. manubrium.
The superior portion of the sternum is called the:
A. xiphoid process.
B. sternal body.
C. manubrium.
D. clavicle.

B. medial malleolus.
A young girl fell while ice skating and injured the protrusion on the inside of the ankle. The medical term for this location is the:
A. acromion.
B. medial malleolus.
C. lateral malleolus.
D. calcaneus.

B. automaticity
The heart muscle has property called _. This means that the heart has the ability to generate and conduct electrical impulses on its own.
A. contractibility
B. automaticity
C. involuntary contraction
D. conductibility

A. autonomic
A division of the peripheral nervous system that controls involuntary motor functions is called the _ nervous system.
A. autonomic
B. central
C. sensory
D. motor

B. the observer.
When in the anatomical position, a person is facing:
A. away from you.
B. the observer.
C. face down.
D. face up.

A. forward.
In the anatomical position, the person’s palms will be facing:
A. forward.
B. backward.
C. upward.
D. downward.

D. plantar.
An anatomical term that is occasionally used to refer to the sole of the foot is:
A. calcaneus.
B. ventral.
C. dorsal.
D. plantar.

C. zygomatic
The bones of the cheek are called the _ bones.
A. orbit
B. maxillae
C. zygomatic
D. mandible

D. superior
The heart is _ to the stomach.
A. distal
B. medial
C. proximal
D. superior

B. proximal; distal
When comparing body structure positions, the knees are said to be _ to the toes, and the toes are _ to the knees.
A. inferior; superior
B. proximal; distal
C. distal; dorsal
D. anterior; posterior

C. supine
A patient found lying on her back is in the _ position.
A. anatomical
B. prone
C. supine
D. lateral recumbent

C. four
To assist in describing the location of abdominal organs, we divide the abdomen into _ parts.
A. two
B. three
C. four
D. five

A. thorax.
Your 18-year-old male patient has severe burns of the entire front (anterior) surface of the torso. The torso of the body is composed of the abdomen, pelvis, and:
A. thorax.
B. upper arms and legs.
C. extremities.
D. head.

A. thoracic
The heart is located in the center of the _ cavity.
A. thoracic
B. cranial
C. pelvic
D. cardiac

C. diaphragm.
The structure that divides the chest cavity from the abdominal cavity is the:
A. meninges.
B. duodenum.
C. diaphragm.
D. spinal column.

C. patella.
The anatomical name for the kneecap is the:
A. ilium.
B. malleolus.
C. patella.
D. phalange.

C. top, back, and sides of the skull.
The cranium consists of the:
A. facial bones.
B. mandible and maxillae.
C. top, back, and sides of the skull.
D. zygomatic bones.

A. acromion process.
The highest point in the shoulder is the:
A. acromion process.
B. humerus.
C. metatarsal.
D. clavicle.

C. Good Samaritan
At the scene of a collision, an off-duty EMT provides care to the patient, acting in good faith and to the best of her abilities. In many states, this EMT is protected from care-related lawsuits by _ laws.
A. applied consent
B. total immunity
C. Good Samaritan
D. jeopardy

B. consider consent for care to be implied and begin care.
When confronted with an unconscious minor without parents or a legal guardian present, the EMT should:
A. seek a physician’s approval before beginning care.
B. consider consent for care to be implied and begin care.
C. ask the child for consent and begin care.
D. consider consent to be applied and begin care.

C. The EMT had a local duty.
The legal concept of negligence requires that three circumstances must me demonstrated. Which of the following is not one of the three circumstances?
A. The EMT had a duty to act.
B. The EMT committed a breach of duty.
C. The EMT had a local duty.
D. The breach of duty caused harm.

B. prone
A person lying on his stomach with his face down is in the _ position.
A. supine
B. prone
C. coma
D. recovery

C. denial, anger, bargaining, depression, acceptance.
As an EMT, you have been assigned to take a terminally ill patient back and forth to chemotherapy multiple times a week for the next few weeks. You realize that the patient has been going through emotional stages in the following order:
A. depression, bargaining, denial, acceptance, anger.
B. acceptance, rage, depression, acceptance, bargaining.
C. denial, anger, bargaining, depression, acceptance.
D. bargaining, acceptance, denial, anger, depression.

B. National Registry of Emergency Medical Technicians
In 1970, the _ was founded to establish professional standards for EMS personnel.
A. American Medical Association
B. National Registry of Emergency Medical Technicians
C. National Highway Traffic Safety Administration
D. U.S. Department of Transportation

D. ambulance.
Safe, reliable transportation is a critical component of an EMS system. Most patients can be transported effectively by:
A. airplane.
B. helicopter.
C. rescue vehicle.
D. ambulance.

B. EMT-Intermediate
An _ is a national-level EMT who has been trained to start IVs, perform advanced airway techniques, and administer some medicines beyond the EMT.
A. EMT-First Responder
B. EMT-Intermediate
C. EMT-Critical Care
D. EMT-Paramedic

B. quality improvement.
A continuous self-review with the purpose of identifying and correcting aspects of the EMS system that require improvement is called:
A. standing orders.
B. quality improvement.
C. protocols.
D. medical direction.

B. Medical Director.
A physician who assumes the ultimate responsibility for the patient-care aspects of the EMS system is called the:
A. Designated Agent.
B. Medical Director.
C. Off-line Director.
D. Primary Care Physician.

A. patients who refuse care.
Situations that are higher risks of a lawsuit against an EMS agency are:
A. patients who refuse care.
B. on-scene deaths.
C. cardiac arrest cases.
D. pedestrians struck by cars.

D. scope of practice.
The legal extent or limits of the EMT’s job are formally defined by the:
A. patient.
B. DOT curriculum.
C. state.
D. scope of practice.

A. Decisiveness
Which is not generally considered a sign or symptom of stress?
A. Decisiveness
B. Guilt
C. Loss of interest in work
D. Difficulty sleeping

D. motor vehicle collisions.
All of the following are types of calls that have a high potential for causing excessive stress except:
A. calls involving infants and children.
B. patients with severe injuries.
C. cases of abuse and neglect.
D. motor vehicle collisions.

B. increased consumption of fatty foods.
Lifestyle changes that can help the EMT deal with stress include all of the following except:
A. exercise to burn off tension.
B. increased consumption of fatty foods.
C. decreased consumption of caffeine.
D. decreased consumption of alcohol.

A. requesting a change of shift or location.
Changes in your professional life to reduce and prevent stress can include:
A. requesting a change of shift or location.
B. taking on another part-time position.
C. working additional overtime shifts.
D. requesting a busier location.

D. protect the body from the environment, bacteria, and other organisms.
One of the functions of the integumentary system is to:
A. regulate the diameter of the blood vessels in the circulation.
B. eliminate excess oxygen into the atmosphere.
C. allow environmental water to carefully enter the body.
D. protect the body from the environment, bacteria, and other organisms.

C. normal and to be expected.
Stress after a major EMS incident is:
A. unusual and expected.
B. a sign of weakness.
C. normal and to be expected.
D. part of the grieving process.

B. depression.
Retreating to a world of one’s own after hearing one is going to die is a result of the stage of grief called:
A. bargaining.
B. depression.
C. denial.
D. anxiety.

B. anaerobic metabolism.
When a patient’s lower extremities are trapped under a farm tractor, the blood supply is diminished to the cells in the legs. This injury can result in:
A. no lactic acids being produced.
B. anaerobic metabolism.
C. no carbon dioxide being produced.
D. none of these.

C. bloodborne
A disease that is spread by exposure to an open wound or sore of an infected individual is caused by a(n) _ pathogen.
A. universal
B. airborne
C. bloodborne
D. infectious

D. hepatits.
An infection that causes inflammation of the liver is called:
A. meningitis.
B. tuberculosis.
C. typhoid.
D. hepatitis.

C. airborne.
A disease spread by inhaling or absorbing droplets from the air through the eyes, nose, or mouth is considered:
A. bloodborne.
B. noncommunicable.
C. airborne.
D. viral.

D. hepatitis B virus.
The communicable disease that kills the most health workers every year in the United States is:
A. tuberculosis.
B. HIV/AIDS.
C. meningitis.
D. hepatitis B virus.

B. productive cough has TB.
Always assume that any patient with a:
A. cold has a bloodborne disease.
B. productive cough has TB.
C. fever has typhoid.
D. rash has measles.

D. It is an airborne pathogen.
Which of the following is not true about the human immunodeficiency virus (HIV)?
A. It attacks the immune system.
B. It doesn’t survive well outside the human body.
C. It can be introduced through puncture wounds.
D. It is an airborne pathogen.

C. 30 percent.
Your patient has hepatitis B. You are accidentally stuck with a needle that has some of this patient’s infected blood on it. Your chance of contracting the disease is about:
A. 10 percent.
B. 20 percent.
C. 30 percent.
D. 40 percent.

A. 0.5 percent.
Your patient has HIV. You are accidentally stuck with a needle that has some infected blood on it. Your chance of contracting the disease is about:
A. 0.5 percent.
B. 5 percent.
C. 10 percent.
D. 15 percent.

C. HEPA or N-95 respirator.
If you think your patient has TB, you should wear the usual personal protective equipment plus a:
A. surgeon’s mask.
B. gown.
C. HEPA or N-95 respirator.
D. Tyvek suit.

A. pocket mask with a one-way valve.
Instead of providing mouth-to-mouth ventilations on the nonbreathing patient, the EMT, when acting alone, should use a(n):
A. pocket mask with a one-way valve.
B. one-way valve.
C. bag-valve mask.
D. endotracheal tube.

C. Hand washing after each patient contact
Which method of infection control reduces exposure to yourself, your crew, and your next patient?
A. Wearing a HEPA or N-95 respirator
B. Taking universal precautions
C. Hand washing after each patient contact
D. None of these

B. the Ryan White CARE Act.
An act that establishes procedures through which emergency response workers can find out if they have been exposed to life-threatening infectious diseases is called:
A. OSHA 1910.1030.
B. the Ryan White CARE Act.
C. AIDS Protection Act.
D. OSHA 1910.120.

A. OSHA 1910.1030.
Each emergency response employer must develop a plan that identifies and documents job classifications and tasks in which there is the possibility for expoure to potentially infectious body fluids. This is required by:
A. OSHA 1910.1030.
B. the Ryan White CARE Act.
C. the AIDS Protection Act.
D. OSHA 1910.120.

D. hepatitis B vaccination.
Every employer of EMTs must provide free of charge:
A. a yearly physical examination.
B. a life insurance policy.
C. universal health insurance.
D. hepatitis B vaccination.

D. all of these.
Engineering controls that prevent the spread of bloodborne diseases include:
A. pocket masks.
B. needle containers.
C. disposable airway equipment.
D. all of these.

C. HEPA or N-95 respirator
Which of the following is not required by the OSHA bloodborne pathogen standard?
A. Postexposure evaluation and follow-up
B. Personal protective equipment
C. HEPA or N-95 respirator
D. Housekeeping controls and labeling

B. Daycare centers
Which of the following is not considered a high-risk area for TB?
A. Correctional facilities
B. Daycare centers
C. Homeless shelters
D. Nursing homes

D. turn off your lights and siren.
As you near an emergency scene, you should:
A. sound your siren to broadcast your arrival.
B. go straight to the front door.
C. secure the scene as quickly as possible.
D. turn off your lights and siren.

A. notify the police immediately.
If anyone at the scene is in possession of a weapon, the EMT should:
A. notify the police immediately.
B. ask the person to give it to you.
C. ignore the person with the weapon.
D. advise the person to leave the scene.

A. respiratory failure.
The reduction of breathing to the point where oxygen intake is not sufficient to support life is called:
A. respiratory failure.
B. anoxic metabolism.
C. respiratory arrest.
D. respiratory support.

C. blue or gray skin coloration.
Adequate signs of breathing include all of the following except:
A. equal expansion of both sides of the chest.
B. air moving in and out of the nose.
C. blue or gray skin coloration.
D. present and equal breath sounds.

B. nasal flaring.
The widening of the nostrils of the nose with respirations is called:
A. hyperventilating.
B. nasal flaring.
C. nasal gurgling.
D. wheezing.

B. cyanosis.
The condition in which a patient’s skin or lips are blue or gray is called:
A. stridor.
B. cyanosis.
C. pallor.
D. anemia.

C. shortness of breath.
If a patient is unable to speak in full sentences, this could be a sign of:
A. complete airway blockage.
B. snoring.
C. shortness of breath.
D. respiratory arrest.

B. inserting an endotracheal tube immediately.
The procedures by which life-threatening respiratory problems are initially treated by the EMT include all of the following except:
A. opening and maintaining the airway.
B. inserting an endotracheal tube immediately.
C. providing supplemental oxygen to the breathing patient.
D. ensuring a clear airway with frequent suctioning.

A. the tongue.
You are assessing the airway of an unconscious male patient. You recall that most airway problems are caused by:
A. the tongue.
B. asthma.
C. shock.
D. the epiglottis.

D. Jaw-thrust
You are treating a patient who fell down a flight of metal stairs. Which maneuver is most appropriate for an unconscious patient found lying at the bottom of a stairwell?
A. Head-tilt, chin-lift
B. Head-tilt, neck-lift
C. Jaw-pull lift
D. Jaw-thrust

B. one-rescuer bag-valve mask.
You are managing a 34-year-old male who you suspect has had a narcotic overdose. His respirations are very slow and shallow, and you will need to assist them. When choosing a means of ventilating a patient, your last choice would be:
A. flow-restricted, oxygen-powered ventilation device.
B. one-rescuer bag-valve mask.
C. two-rescuer bag-valve mask.
D. mouth-to-mask with high-flow supplemental oxygen.

C. rate of ventilation is too fast or too slow.
Artificial ventilation may be inadequate if the:
A. chest rises with each ventilation.
B. heart rate returns to normal.
C. rate of ventilation is too fast or too slow.
D. skin becomes warm and dry.

A. 15/22 mm.
The standard respiratory fitting on a bag-valve mask that ensures a proper fit with other respiratory equipment is:
A. 15/22 mm.
B. 10/14 mm.
C. 5/20 mm.
D. 20/26 mm.

D. a pop-off valve.
A bag-valve mask should have or be all of the following except:
A. a self-refilling shell.
B. a clear face mask.
C. easily cleared and sterilized.
D. a pop-off valve.

C. 15
The proper oxygen flow rate when ventilating a patient with a BVM is _ liters per minute.
A. 5
B. 10
C. 15
D. 20

B. sufficient to achieve visible chest rise.
According to the American Heart Association guidelines, when ventilating a patient with a bag mask that has supplementary oxygen, the volume administered should be:
A. 400 milliliters.
B. sufficient to achieve visible chest rise.
C. 800 milliliters.
D. as much as possible during the 1-second time frame.

C. clear any mucus or secretions obstructing the stoma.
The first step in providing artificial ventilation of a stoma breather is to:
A. leave the head and neck in a neutral position.
B. ventilate at the appropriate rate for the patient’s age.
C. clear any mucus or secretions obstructing the stoma.
D. establish a seal using a pediatric-sized mask.

A. an audible alarm when ventilation is activated.
A flow-restricted, oxygen-powered ventilation device should have all of the following features except:
A. an audible alarm when ventilation is activated.
B. a trigger that enables the rescuer to use both hands.
C. a peak flow rate of up to 40 liters per minute.
D. a rugged design and construction.

B. nasopharyngeal airway.
The two most common airway adjuncts for the EMT to use are the oropharyngeal airway and the:
A. nasal cannula.
B. nasopharyngeal airway.
C. endotracheal tube.
D. Yankauer.

D. all unconscious patients with no gag reflex.
An oropharyngeal airway should be inserted in:
A. all patients with inadequate breathing.
B. trauma patients with a gag reflex.
C. medical patients with a gag reflex.
D. all unconscious patients with no gag reflex.

C. never suction for longer than 15 seconds.
When suctioning a 19-year-old patient who you suspect has bleeding into his throat, you should:
A. suction on the way in and the way out.
B. avoid using eyewear or a mask.
C. never suction for longer than 15 seconds.
D. hypoventilate prior to suctioning.

C. shock.
The emergency situation in which there is a failure of the cardiovascular system to provide sufficient blood to all the vital tissues is called:
A. respiratory arrest.
B. respiratory failure.
C. shock.
D. cardiac arrest.

C. hypoxia.
An insufficiency in the supply of oxygen to the body’s tissues is called:
A. anoxia.
B. no-oxia.
C. hypoxia.
D. cyanosis.

A. 200
Before the oxygen cylinder’s pressure gauge reads a minimum of _ psi, you must switch to a fresh cylinder.
A. 200
B. 400
C. 800
D. 1,000

C. store reserve cylinders in a warm, humid room.
When handling oxygen cylinders, the EMT should do all of the following except:
A. have the cylinders hydrostatically tested every 5 years.
B. ensure that valve seat inserts and gaskets are in good condition.
C. store reserve cylinders in a warm, humid room.
D. use medical-grade oxygen in all cylinders.

A. nonrebreather mask.
The best way to deliver high-concentration oxygen to a breathing patient is to use a:
A. nonrebreather mask.
B. partial rebreather mask.
C. bag-valve mask.
D. nasal cannula.

B. 24; 44
A nasal cannula provides between _ percent and _ percent oxygen concentrations.
A. 10; 21
B. 24; 44
C. 36; 58
D. 72; 96

B. leave them in unless they are loose.
You are assessing a 54-year-old woman who is unconscious and has a noisy upper airway. If she has dentures, during airway procedures the EMT should:
A. remove them right away.
B. leave them in unless they are loose.
C. remove the teeth one at a time.
D. hold them in place with a free hand.

A. the mouth and nose are smaller and more easily obstructed.
When managing the airway of a child, an airway consideration you should remember is that:
A. the mouth and nose are smaller and more easily obstructed.
B. the chest wall is firmer in a child.
C. the trachea is wider and less easily obstructed.
D. all of these are airway considerations in a child.

A. respiratory arrest.
You are dealing with a patient who is in severe respiratory distress from a life-threatening asthma attack. If breathing stops completely, the patient is in:
A. respiratory arrest.
B. ventilatory reduction.
C. artificial ventilation.
D. respiratory failure.

C. Flow-restricted, oxygen-powered ventilation device
Which ventilation device is contraindicated in infants and children?
A. Bag-valve mask
B. Pediatric pocket mask
C. Flow-restricted, oxygen powered ventilation device
D. Nonrebreather mask.

A. flowmeter.
A device that allows the control of oxygen in liters per minute is called a:
A. flowmeter.
B. G tank.
C. humidifier.
D. reservoir.

B. constant flow selector valve.
A type of flowmeter that has no gauge and allows for the adjustment of flow in liters per minute and in stepped increments is called a:
A. Bourdon gauge flowmeter.
B. constant flow selector valve.
C. humidifier.
D. pressure compensated flowmeter.

A. Lack of humidity can dry out the patient’s mucous membranes.
Why do some EMS systems use humidified oxygen?
A. Lack of humidity can dry out the patient’s mucous membranes.
B. It provides a reservoir for the oxygen.
C. It limits the risk of infection.
D. It is helpful when transporting patients short distances.

C. hypoxic drive.
A patient in the end stage of a respiratory disease may have switched over to:
A. hyperventilation syndrome.
B. hyperbaric therapy.
C. hypoxic drive.
D. carbon dioxide drive.

D. Chronic pulmonary disease
You are talking with a patient and he states that has has had COPD for the past 10 years. What is COPD?
A. A type of shock
B. A type of ventilation
C. A mechanism of breathing
D. Chronic pulmonary disease

D. withdrawn; rigid
When using an air mattress, the patient is placed on the device and the air is _ by a pump. The mattress will then form a _ and conforming surface around the patient.

C. digestive
The body system that is responsible for the breakdown of food into absorbable forms is called the _ system.
A. urinary
B. nervous
C. digestive
D. integumentary

C. post-traumatic stress disorder.
A stress reaction that involves either physical or psychological behavior manifested days or weeks after an incident is called a(n):
A. cumulative stress disorder.
B. burnout.
C. post-traumatic stress disorder.
D. acute stress reaction.

B. Health Insurance Portability and Accountability Act.
An agency privacy officer is required by the:
A. Ryan White Law.
B. Health Insurance Portability and Accountability Act.
C. National Fire Protection Association Standards.
D. Privacy Control Act of 2002.

D. 6,000
When an adult patient is breathing at a respiratory rate of 12 times a minute, you would expect that the minute volume would be approximately _ mL per minute.
A. 3,000
B. 4,500
C. 5,000
D. 6,000

C. dehydration.
Your patient is a 45-year-old female who has been vomiting and has had diarrhea for the past week. There is a danger that she may have a condition called:
A. extremity edema.
B. an excess of body fluid.
C. dehydration.
D. all of these.

A. Yes, he is probably very short of breath.
You question an elderly man sitting on a bench in the park. He has a respiratory complaint, and he speaks in short, two or three word sentences. Is this significant?
A. Yes, he is probably very short of breath.
B. No, elderly patients always talk slowly.
C. No, he is probably always like that.
D. Yes, he probably has a complete airway obstruction.

B. Because children in this age group are primarily nasal breathers.
Why can nasal congestion be a major problem in the first few months of life?
A. Because the liver is so large in patients in this age group.
B. Because children in this age group are primarily nasal breathers.
C. Because it is an indication of life-threatening airway compromise.
D. Because children in this age group breathe with their diaphragm.

B. sucking
When the mother strokes the infant’s lips and the baby starts sucking, this is a nervous system reflex known as the _ reflex.
A. Moro
B. sucking
C. palmar
D. rooting

C. self-destructive behaviors.
The adolescent years are the beginning of:
A. better decision making skills.
B. nasal breathing.
C. self-destructive behaviors.
D. all of these.

D. Young adult
You are treating a patient who was involved in a serious accident. Based on the leading causes of death, what age group is this patient most likely to be in?
A. Infant
B. School age
C. Adolescent
D. Young adult

D. Adolescent
Serious conflicts occur in families as the issues of control and independence collide with children in which age group?
A. Toddler
B. Preschool
C. School age
D. Adolescent

B. 16.
Girls are usually finished growing by the age of:
A. 14.
B. 16.
C. 18.
D. 20.

B. Yes, she is probably short of breath.
You are examining an elderly woman sitting on a park bench. She is sitting forward with her elbows outward and speaking in short, choppy sentences. Is this significant in her history?
A. No, she is probably always like that.
B. Yes, she is probably short of breath.
C. No, elderly patients like to sit forward like that.
D. Yes, she probably has a spine injury.

B. head-tilt, chin-lift
You have been called to a restaurant for a patient who collapsed at the dinner table. According to a family members, she did not strike her head as she slid out of the chair when she went unconscious. To open her airway, the EMT should use a _ maneuver.
A. modified jaw-thrust
B. head-tilt, chin-lift
C. head-tilt, neck-lift
D. modified chin-lift

C. measured from the patient’s nostril to the earlobe.
A nasopharyngeal airway should be:
A. turned 180 degrees with the tip facing the roof of the mouth.
B. inserted with the bevel on the lateral side of the nostril.
C. measured from the patient’s nostril to the earlobe.
D. inserted in the left nostril when possible.

C. inadequate breathing and is a precursor to respiratory arrest.
You are treating a patient who initially had a chief complaint of severe difficulty breathing. You have concerns that this may be leading to respiratory failure, which is:
A. cause by electrocution in young adults.
B. another term used to describe respiratory arrest.
C. inadequate breathing and is a precursor to respiratory arrest.
D. the complete cessation of expiration.

D. air that can be felt at the nose or mouth on exhalation.
You are treating a patient who initially had a chief complaint of severe difficulty breathing. You have concerns that this may be leading to respiratory failure. The patient could have any of the following signs except:
A. absent or minimal chest movement.
B. noises such as wheezing, gurgling, stridor, or crowing.
C. diminished or absent breath sounds.
D. air that can be felt at the nose or mouth on exhalation.

B.12 to 20 per minute.
You are treating a patient who your partner states has a normal breathing rate for an adult. The patient’s breathing rate is most likely:
A. 6 to 10 per minute.
B. 12 to 20 per minute.
C. 15 to 30 per minute.
D. 25 to 50 per minute.

D. all of these are correct.
You are treating a patient who initially had a chief complaint of severe difficulty breathing. You have concerns that this may be leading to respiratory failure. As you assess and manage the patient, you decide she is either not breathing or her breathing is inadequate. Thus, it will be necessary to provide:
A. artificial ventilation.
B. positive pressure ventilation.
C. assistance with a bag-valve mask device.
D. all of these are correct.

D. blood pressure.
During the primary assessment, all of the following are assessed except:
A. airway.
B. mental status.
C. circulation.
D. blood pressure.

B. Evaluating extremity mobility
Which is not a component of the primary assessment?
A. Determining patient priority
B. Evaluating extremity mobility
C. Forming a general impression
D. Assessing breathing

B. holding pieces of a toy in his or her hand.
When forming a general impression of a child’s condition, the environment can provide clues. An example would be a child:
A. complaining of nausea or vomiting.
B. holding pieces of a toy in his or her hand.
C. with a blood pressure of 120/80.
D. with a rapid heartbeat.

C. blood pressure.
When forming a general impression, the EMT takes into consideration all of the following except the patient’s:
A. position.
B. age and sex.
C. blood pressure.
D. sounds.

B. assessing mental status.
After forming a general impression, the next step of the primary assessment is:
A. assessing breathing.
B. assessing mental status.
C. determining the priority of the patient.
D. assessing the airway.

C. verbal.
The “V” in AVPU stands for:
A. virtual.
B. visible.
C. verbal.
D. vertex.

B. painful.
The “P” in AVPU stands for:
A. pulse.
B. painful.
C. paralysis.
D. parasthesia.

D. U.
The lowest and the most serious mental status is:
A. A.
B. V.
C. P.
D. U.

B. administer high-concentration oxygen.
If a patient’s level of responsiveness is lower than “Alert,” you should:
A. apply a cervical collar.
B. administer a high-concentration oxygen.
C. place him or her in a recovery position.
D. bandage all bleeding wounds.

C. has an open airway.
If a patient is talking or crying, assume that the patient:
A. is in little pain.
B. has no life-threatening bleeding.
C. has an open airway.
D. has an “A” mental status.

C. assist ventilations with a bag-valve mask.
If the patient is not alert and her breathing rate is slower than 8 breaths per minute, the EMT should:
A. insert an oral airway.
B. place the patient in a prone position.
C. assist ventilations with a bag-valve mask.
D. apply the PASG.

B. poor circulation.
Pale and clammy skin indicates:
A. nervous system damage.
B. poor circulation.
C. extremely high temperature.
D. an airway problem.

A. high
A patient who gives a poor general impression should be considered a _ priority.
A. high
B. low
C. delayed
D. stable

B. Nausea and vomiting
Which one of the following is not an example of a high-priority condition?
A. Difficulty breathing
B. Nausea and vomiting
C. A severed artery
D. A mental status of “U”

A. has a medical problem or a trauma problem.
The primary assessment takes different forms, which depend on the age of the patient and whether he or she:
A. has a medical problem or a trauma problem.
B. is alert or conscious.
C. has a history of diabetes.
D. has a history of head injury.

C. capillary refill test.
When evaluating circulation in infants, the EMT should use the:
A. femoral pulse.
B. distal pulse check.
C. capillary refill test.
D. radial pulse.

B. shouting and flicking his feet.
The mental status of an unconscious infant is typically checked by:
A. splashing cold water on him.
B. shouting a flicking his feet.
C. perorming a pin prick on the feet.
D. applying a sternal rub.

B. A fever with a rash
Which is not a high-priority condition?
A. Shock
B. A fever with a rash
C. A complicated childbirth
D. Severe pain anywhere

A. manual stabilization of the head for a trauma patient.
The difference between the general impression steps for a medical patient and those for a trauma patient is:
A. manual stabilization of the head for a trauma patient.
B. no SAMPLE is taken for a trauma patient.
C. the medical patient is treated faster.
D. the AVPU is not used for a medical patient.

C. a pertinent negative.
The EMT documents that the chest pain patient denies shortness of breath. This is most appropriately termed:
A. an informative objective.
B. advisory information.
C. a pertinent negative.
D. run data information.

D. state one’s personal opinion of the patient’s condition.
When writing the narrative section of a prehospital care report (PCR), the EMT should do all of the following except:
A. use medical terminology correctly.
B. avoid nonstandard abbreviations.
C. write legibly and use correct spelling.
D. state one’s personal opinion of the patient’s condition.

B. If it is not written down, you didn’t do it.
Which statement best describes an important concept of documentation?
A. Always fill out a continuation sheet.
B. If it is not written down, you didn’t do it.
C. It should include plenty of medical terms.
D. Document everything you see.

A. document with a refusal-of-care form.
If the patient does not wish to go to the hospital, the EMT should:
A. document with a refusal-of-care form.
B. take the patient to the hospital anyway.
C. end the assessment immediately.
D. call for another crew to subdue him.

B. omission.
An important part of the assessment or care was not performed. This is called:
A. submission.
B. omission.
C. inhibition.
D. commission.

C. falsification.
An EMT makes up a set of vitals for inclusion on the prehospital care report (PCR). This is called:
A. assault.
B. libel.
C. falsification.
D. battery.

C. one that describes observable or measurable information.
An objective statement is:
A. one that is made from a particular point of view.
B. one that is made by the patient or a family member.
C. one that describes observable or measurable information.
D. one’s own assessment of what is wrong with the patient.

A. objective information.
The EMT writes “The patient is alert and oriented” on the prehospital care report (PCR). This is an example of:
A. objective information.
B. a statement beyond his or her training.
C. subjective information.
D. nonfactual information.

D. use quotes around the statement.
When you are documenting exactly what a patient told you on the prehospital care report (PCR), you should:
A. paraphrase for brevity.
B. summarize the key points.
C. use medical terminology.
D. use quotes around the statement.

A. not relevant.
Writing a statement such as “The patient’s daughter was rude to us” on a prehospital care report (PCR) is:
A. not relevant.
B. objective information.
C. run data information.
D. an example of what should be included in quotes.

A. ensure an open airway.
The next step in the primary assessment after establishing unresponsiveness is to:
A. ensure an open airway.
B. check for adequate breathing.
C. check for circulation.
D. look for profuse bleeding.

A. an EMT’s clinical judgement.
The term “sixth sense” is used to describe:
A. an EMT’s clinical judgement.
B. the six steps in a primary assessment.
C. a patient’s sensory functions.
D. the ability to hear.

B. Skin temperature
Which one of the following is a true vital sign?
A. Nausea
B. Skin temperature
C. Level of responsiveness
D. Age

B. 60 to 100
The normal pulse rate for an adult at rest is between _ beats per minute.
A. 50 to 70
B. 60 to 100
C. 65 to 95
D. 80 to 100

D. radial
During the determination of vital signs, the initial pulse rate for patients 1 year of age and older is normally taken at the _ pulse.
A. carotid
B. femoral
C. pedal
D. radial

D. thready.
When the force of the pulse is weak and thin, it is described as:
A. shallow.
B. full.
C. partial.
D. thready.

B. 12 to 20
Normal at-rest respiration rates for adults vary from _ breaths per minute.
A. 5 to 10
B. 12 to 20
C. 15 to 30
D. 20 to 32

D. crowing.
A noisy, harsh sound heard during inhalation that indicates a partial airway obstruction is:
A. gurgling.
B. snorting.
C. stertorous respirations.
D. crowing.

C. heart contracts.
Systolic blood pressure indicates the arterial pressure created as the:
A. artery contracts.
B. artery relaxes.
C. heart contracts.
D. heart relaxes.

B. heart refills.
Diastolic blood pressure indicates the arterial pressure created as the:
A. heart contracts.
B. heart refills.
C. artery contracts.
D. artery relaxes.

B. auscultation.
The techniqur of measuring blood pressure with a sphygmomanometer and a stethoscope is called:
A. palpation.
B. auscultation.
C. oscillation.
D. priaprism.

A. not as accurate as the auscultation method.
Determining blood pressure by palpation is:
A. not as accurate as the auscultation method.
B. used when there is no noise around a patient.
C. documented as the “palp/diastolic.”
D. used whenever the patient is hypertensive.

A. signs.
Information that you can see, hear, feel, and smell is called:
A. signs.
B. sensations.
C. symptoms.
D. assessments.

B. patient’s pertinent past history.
When the EMT asks the patient “Have you recently had any surgery or injuries?” he is inquiring about the:
A. patient’s medications.
B. patient’s pertinent past history.
C. events leading up to the illness.
D. allergies that the patient may have.

A. patient’s medications.
When the EMT asks the patient “Are you on birth control pills?” she is inquiring about the:
A. patient’s medications.
B. patient’s pertinent past history.
C. events leading up to the illness.
D. allergies that the patient may have.

C. SAMPLE.
An acronym used to remember what questions to ask about the patient’s present problem and past history is:
A. AVPU.
B. DCAP-BTLS.
C. SAMPLE.
D. PEARL.

D. avoid asking the patient’s age.
When conducting a patient interview on an adult patient, the EMT should do all of the following except:
A. position oneself close to the patient.
B. identify oneself and reassure the patient.
C. gently touch the patient’s shoulder or rest a hand over the patient’s hand.
D. avoid asking the patient’s age.

C. 60 to 80
Normal diastolic blood pressures range from _ mmHg.
A. 50 to 80
B. 50 to 90
C. 60 to 80
D. 60 to 90

C. 136
Using the formula presented in the text, a 36-year-old man would have an estimated systolic blood pressure of _ mmHg.
A. 116
B. 126
C. 136
D. 146

B. 116
Using the formula presented in the text, a 26-year-old woman would have an estimated systolic blood pressure of _ mmHg.
A. 106
B. 116
C. 126
D. 136

A. It may give a false reading.
Why should the stethoscope not be placed under the cuff and then inflated when evaluating the blood pressure?
A. It may give a false reading.
B. It can dent the stethoscope.
C. The patient will not like the feeling.
D. It will prevent pumping the cuff to the proper pressure.

C. brachial
When taking a patient’s blood pressure, the stethoscope is placed over the _ artery.
A. carotid
B. radial
C. brachial
D. femoral

C. mechanism of injury (MOI).
The best indication of potential injury is the:
A. type of accident.
B. type of injury.
C. mechanism of injury (MOI).
D. position of the patient.

B. 50
When there are no apparent hazards at the scene of a collision, the danger zone should extend _ feet in all directions from the wreckage.
A. 25
B. 50
C. 75
D. 100

C. head
In the up-and-over injury pattern, the patient is most likely to sustain _ injuries.
A. knee
B. hip
C. head
D. leg

C. index of suspicion.
A keen awareness that there may be injuries based on the mechanism of injury (MOI) is called a(n):
A. sixth sense.
B. general impression.
C. index of suspicion.
D. kinesthetic sense.

C. discover and treat life-threatening conditions.
The purpose of the primary assessment is to:
A. take the patient’s vital signs.
B. gather information about the collision.
C. discover and treat life-threatening conditions.
D. obtain the patient’s history.

C. reconsider the mechanism of injury (MOI).
The first step in the focused history and physical exam for ANY trauma patient is to:
A. make a status decision.
B. check for an open airway.
C. reconsider the mechanism of injury (MOI).
D. treat life-threatening conditions.

C. abrasions.
The “A” in DCAP-BTLS stands for:
A. allergies.
B. alert.
C. abrasions.
D. arrhythmia.

D. breathing rate.
All of the following are clues that help an EMT determine the need for cervical immobilization except:
A. mechanism of injury (MOI).
B. level of responsiveness.
C. location of injuries.
D. breathing rate.

B. It evaluates areas of the body where the greatest threats to the patient may be.
Which of the following is true of the rapid trauma assessment?
A. It begins with examination of posterior body and ends with examination of the head.
B. It evaluates areas of the body where the greatest threats to the patient may be.
C. It is performed on a trauma patient who has no significant mechanism of injury.
D. It includes careful examination of the face, eyes, ears, nose, and mouth.

A. A trauma patient with a significant mechanism of injury
On which type of patient is the secondary assessment most often performed?
A. A trauma patient with a significant mechanism of injury
B. A trauma patient with no significant mechanism of injury
C. A responsive medical patient
D. An unresponsive medical patient

A. ask the OPQRST questions.
To obtain a history of a patient’s present illness:
A. ask the OPQRST questions.
B. conduct the subjective interview.
C. ask the SAMPLE questions.
D. use the look, listen, feel, and smell method.

C. provokes.
The “P” in OPQRST stands for:
A. punctures.
B. penetrations.
C. provokes.
D. pulses.

A. 3, 1, 2, 4
The steps in the focused history and physical exam of the unresponsive medical patient include the following:

  1. Obtain baseline vital signs.
  2. Gather the history of the present illness from bystanders and family.
  3. Conduct a rapid physical exam.
  4. Gather a SAMPLE history from bystanders and family.

Which is the correct order in which these steps should be performed?
A. 3, 1, 2, 4
B. 2, 3, 4, 1
C. 3, 2, 4, 1
D. 1, 2, 3, 4

C. 15
For a stable patient, the EMT should perform the reassessment every _ minutes.
A. 5
B. 10
C. 15
D. 20

A. repeat the primary assessment.
During the reassessment, whenever you believe there may have been a change in the patient’s condition, you should immediately:
A. repeat the primary assessment.
B. transport the patient immediately.
C. document trends in vital signs.
D. repeat the rapid trauma assessment.

D. shock.
Cool, clammy skin in a middle-aged male most likely indicates:
A. high fever.
B. exposure to cold.
C. mild fever.
D. shock.

D. your own safety.
As an EMT, your overriding concern at all times is:
A. the patient’s safety.
B. the safety of patients and bystanders.
C. the patient’s life.
D. your own safety.

A. ears and nose for blood or clear fluids.
During the secondary assessment of the head of a trauma patient, inspect the:
A. ears and nose for blood or clear fluids.
B. inner surface of the eyelids.
C. skin color of the cheeks.
D. mouth for blood and clear fluids.

C. 8
Infants and young children under the age of _ are abdominal breathers.
A. 2
B. 4
C. 8
D. 12

B. devices used to transmit radio signals over long distances.
Repeaters are:
A. phones that transmit radio signals over airwaves instead of wires.
B. devices used to transmit radio signals over long distances.
C. patients who consistently call 911.
D. a type of cellular phone used in EMS.

C. Directly facing a patient
Which of the following is an appropriate interpersonal communication technique?
A. Standing above a patient
B. Avoiding eye contact
C. Directly facing a patient
D. Standing with arms crossed

B. Unit identification and level of provider
What is the first item given to the hospital in your medical radio report to the receiving facility?
A. The patient’s age and sex
B. Unit identification and level of provider
C. Estimated time of arrival
D. Emergency medical care given

D. Patient’s attitude
Which of the following is not considered an essential component of the verbal report to the receiving facility?
A. Patient’s chief complaint
B. Additional treatment given en route
C. Additional vital signs taken en route
D. Patient’s attitude

C. The EMT paints a picture of the patient’s problem in words.
Which of the following is true of the medical radio report?
A. The EMT uses codes to communicate patient information.
B. The EMT makes sure to give his or her patient diagnosis.
C. The EMT paints a picture of the patient’s problem in words.
D. The EMT speaks rapidly to limit transmission time.

D. avoid questioning the physician about the order.
To avoid misunderstanding and miscommunication when speaking with medical direction, you should do all of the following except:
A. give information clearly and accurately.
B. repeat back the order you are given word for word.
C. ask the physician to repeat the order if it is unclear.
D. avoid questioning the physician about the order.

A. oral glucose.
Medications that are carried on the ambulance and that EMTs can administer include activated charcoal, oxygen, and:
A. oral glucose.
B. nitroglycerin.
C. epinephrine.
D. all of these.

D. side effect.
Any action of a drug other than the desired action is called a(n):
A. contraindication.
B. indication.
C. reflex.
D. side effect.

D. injected under the skin.
When a drug is administered subcutaneously, the drug is:
A. dissolved under the tongue.
B. injected into a vein.
C. rubbed into a muscle.
D. injected under the skin.

D. They depend more on the diaphragm for respiration than adults do.
Which of the following is true of the structure of infants’ and children’s airways?
A. The trachea is more rigid and less flexible than an adult’s.
B. The tongue is proportionately smaller than an adult’s.
C. The cricoid cartilage is more rigid than and adult’s.
D. They depend more on the diaphragm for respiration than adults do.

A. 1
The means of providing artificial ventilation are:

  1. Pocket face mask with supplemental oxygen
  2. Two-person bag-valve mask with supplemental oxygen
  3. Flow-restricted, oxygen-powered ventilator
  4. One-person bag-valve mask with supplemental oxygen.

Which of these is the best method to use?
A. 1
B. 2
C. 3
D. 4

B. 10-12
The adequate rate of artificial ventilations for a nonbreathing adult patient is _ breaths per minute.
A. 6-8
B. 10-12
C. 14-16
D. 18-20

C. 12-20
The adequate rate of artificial ventilations for a nonbreathing infant or child patient is _ breaths per minute.
A. 8-10
B. 6-8
C. 12-20
D. 22-25

A. Snoring or gurgling
Which of the following respiratory sounds made by an unresponsive adult most likely indicates a serious airway problem requiring immediate intervention?
A. Snoring or gurgling
B. Slight wheezing
C. Sniffling
D. Whistling or grunting

D. blue, cool, and clammy.
The skin of a patient with inadequate breathing will most likely be:
A. pale, cool, and dry.
B. red, hot, and clammy.
C. yellow, warm, and dry.
D. blue, cool, and clammy.

C. sitting-up
If a patient is experiencing breathing difficulty but is breathing adequately, it is usually best to place him in the _ position.
A. tripod
B. supine
C. sitting-up
D. recovery

C. Angina pectoris
Which one fo the following is a name for adult chest pain due to a decreased blood supply to the heart muscle?
A. Stroke
B. Arrhythmia
C. Congestive heart failure
D. Angina pectoris

C. coronary
Most heart attacks are caused by the narrowing or occlusion of a _ artery.
A. cephalic
B. brachial
C. coronary
D. carotid

D. Acute myocardial infarction
Which of the following is the condition in which a portion of the myocardium dies because of oxygen starvation?
A. Angina pectoris
B. Mechanical pump failure
C. Cardiogenic shock
D. Acute myocardial infarction

B. arrhythmia.
An irregular heart rhythm is called:
A. mechanical pump failure.
B. arrythmia.
C. cardiogenic shock.
D. congestive heart failure.

A. Bradycardia
Which term applies to a pulse slower than 60 beats per minute?
A. Bradycardia
B. Ventricular fibrillation
C. Tachycardia
D. Atrial fibrillation

C. tachycardia.
An at-rest heart beating faster than 100 beats per minute is referred to as:
A. bradycardia.
B. ventricular fibrillation.
C. tachycardia.
D. atrial fibrillation.

A. Congestive heart failure
Which of the following is the name for a condition caused by excessive fluid build-up in the lungs because of the inadequate pumping of the heart?
A. Congestive heart failure
B. Acute heart failure
C. Acute myocardial infarction
D. Chronic myocardial infarction

D. position of comfort.
The 52-year-old male conscious patient with a possible heart attack is best transported in the:
A. recovery position.
B. medical coma position.
C. traumatic coma position.
D. position of comfort.

A. hypoglycemia.
A diabetic found with a weak, rapid pulse and cold, clammy skin who complains of hunger pangs suffers from:
A. hypoglycemia.
B. cardiogenic shock.
C. hyperglycemia.
D. ulcers.

B. Hyperglycemia
Which of the following conditions frequently results in an acetone smell of the patient’s breath?
A. Stroke
B. Hyperglycemia
C. Ulcers
D. Hypoglycemia

A. oral glucose.
A conscious hypoglycemic patient who is able to swallow is frequently administered:
A. oral glucose.
B. insulin.
C. nitroglycerin.
D. epinephrine.

C. treat her like any other patient with altered mental status.
You are treating a 25-year-old female diabetic patient. If you cannot administer glucose gel because she is not alert enough to swallow, you should:
A. wait until her mental status improves.
B. contact medical direction immediately.
C. treat her like any other patient with altered mental status.
D. place her in a position of comfort.

C. forms antibodies.
The first time a person is exposed to an allergen, the immune system:
A. reacts violently.
B. shuts down.
C. forms antibodies.
D. ignores the allergen.

C. respiratory distress.
To be considered a severe allergic reaction, a patient must have signs and symptoms of shock and/or:
A. a history of allergies.
B. massive swelling.
C. respiratory distress.
D. increased blood pressure.

B. Treat for shock and transport immediately.
A 21-year-old male patient has no history of allergies and is having his first allergic reaction. What action should you take?
A. Consult with medical direction.
B. Treat for shock and transport immediately.
C. Administer epinephrine via auto-injector.
D. Attempt to determine the cause immediately.

C. inhaled
Carbon monoxide, chlorine, and ammonia are examples of _ poisons.
A. ingested
B. injected
C. inhaled
D. absorbed

A. different poisons act on the body at different rates.
It is important for the EMT to determine when the ingestion of a poison occurred because:
A. different poisons act on the body at different rates.
B. those who ingest poison in the evening tend to vomit frequently.
C. dilution of the poison is never effective after 10 minutes.
D. the antidote is more effective once the poison reaches the stomach.

B. administering high-concentration oxygen.
The principal prehospital treatment of a patient who has inhaled poisonous gas is:
A. administering activated charcoal.
B. administering high-concentration oxygen.
C. rapidly administering an antidote.
D. irrigating the respiratory tract with water.

B. depressed vital signs.
Drinking alcohol along with taking other drugs frequently results in:
A. uncontrolled shivering.
B. depressed vital signs.
C. extreme agitation.
D. all of the above.

D. all of these.
Activated charcoal is contraindicated for patients who have ingested:
A. alkalis.
B. gasoline.
C. acids.
D. all of these.

C. involve young children.
Most cases of poisoning:
A. are intentional in nature.
B. involve elderly patients.
C. involve young children.
D. lead to disability or death.

C. ingested
Poisons that are swallowed are _ poisons.
A. absorbed
B. inhaled
C. ingested
D. injected

B. Continuous ambulatory peritoneal dialysis
What is the name for a gravity exchange process for peritoneal dialysis in which a bag of dialysis fluid is raised above the level of an abdominal catheter to fill the abdominal cavity and lowered below the level of the abdominal catheter to drain the fluid out?
A. Continuous cycler-assisted peritoneal dialysis
B. Continuous ambulatory peritoneal dialysis
C. Abdominal dialysis
D. Primary fluid exchange process

C. Peritonitis
You are called to the home of a dialysis patient who has extreme abdominal pain and a fever. She said that she spent the whole day at the clinic and was not feeling well since she got home. What could be her most likely problem today?
A. Diabetes
B. An ulcer
C. Peritonitis
D. Appendicitis

D. It is a vibration felt on palpation, which typically occurs within an arterial-venous fistula.
You are called to the home of a dialysis patient who has extreme abdominal pain and a fever. She said that she spent the whole day at the clinic and was not feeling well since she got home. She tells you that she was told she has a thrill in the arm used for dialysis. What does that mean?
A. There is a hairline fracture in the bone in that extremity.
B. She is missing a number of vessels in the extremity.
C. It is a very tight bandage applied after the procedure to minimize blood clots.
D. It is a vibration felt on palpation, which typically occurs within an arterial-venous fistula.

B. severe pain after falling and striking the head.
Common presentations of patients experiencing psychiatric emergencies include each of the following except:
A. panic or anxiety.
B. severe pain after falling and striking the head.
C. suicidal or self-destructive behavior.
D. unusual speech patterns (ex: too rapid or pressured-sounding speech).

A. plenty of physical contact.
Emergency care of a patient having a behavioral or psychiatric emergency includes each of the following except:
A. plenty of physical contact.
B. encouraging the patient to discuss what is troubling him.
C. not lying to the patient.
D. being prepared to spend time talking to the patient.

A. to defend against an attack by an emotionally disturbed patient.
In a limited number of situations, you may have to utilize force. The use of force by an EMT is allowed:
A. to defend against an attack by an emotionally disturubed patient.
B. whenever the patient refuses to be transported.
C. if the patient refuses to follow your care plan.
D. if the patient has taken any drugs.

B. retroperitoneal space.
The organs found outside the peritoneum and between the abdomen and the back are in the:
A. posterior pelvic compartment.
B. retroperitoneal space.
C. periumbilical region.
D. posterior buttocks.

C. upper left
The spleen is located in the _ quadrant.
A. upper right
B. lower right
C. upper left
D. lower left

D. An abdominal aortic aneurysm
Your 60-year-old male patient tells you that he has a tearing sensation going into the middle of his back. He states that he has no back problems and has a history of hypertension. What is possible wrong with him?
A. A kidney stone
B. Acute pulmonary edema
C. Acute appendicitis
D. An abdominal aortic aneurysm

B. He has a bleeding ulcer.
Your patient has a history of alcoholism and is complaining of stomach pains and feeling weak and dizzy. He states that he vomited twice and has had very dark, foul-smelling diarrhea all day. What is the likely cause of today’s problem?
A. He also has diabetes and needs insulin.
B. He has a bleeding ulcer.
C. He is having a stroke.
D. His spleen is malfunctioning.

D. ensure your own safety.
Your first step when called to care for any attempted suicide victim is to:
A. gain access to the patient.
B. wait for police assistance.
C. survey for behavioral changes.
D. ensure your own safety.

C. Seek advice from medical direction.
You are on the scene of a patient whom the police have been talking with for quite some time. They were originally called to the private residence because a neighbor though she had heard shots fired. Apparently no one was injured, but there are weapons in the home. You are unable to perform normal assessment and care procedures because the patient is aggressive and hostile. What action should you take?
A. Restrain the patient immediately.
B. Ask a family member to assist you.
C. Seek advice from medical direction.
D. Call the patient’s physician.

B. To defend against attack by an emotionally disturbed patient
When is the EMT allowed to use reasonable force?
A. Only when the police are on the scene
B. To defend against attack by an emotionally disturbed patient
C. Whenever a patient refuses treatment
D. Whenever alcohol abuse is suspected

A. administration of intravenous fluids.
The limited number of interventions an EMT can provide include all of the following except:
A. administration of intravenous fluids.
B. application of an AED.
C. application of oxygen.
D. insertion of oral or nasal airways.

C. asses the patient as usual.
When a 42-year-old patient presents with more than one condition or with a familiar condition but under unusual circumstances, the EMT should:
A. call medical direction right away.
B. call for an ALS unit right away.
C. assess the patient as usual.
D. assign a higher priority to the patient.

A. “I’m not familiar with that disease. Could you tell me about it?”
When a 45-year-old male patient tells you that he has a disease with which you are not familiar, it is best to respond by saying:
A. “I’m not familiar with that disease. Could you tell me about it?”
B. “That doesn’t concern me. My job is to treat emergencies.”
C. “Interesting disease. I just studied it during EMS training.”
D. “I’m not a physician. Just tell me what’s wrong today.”

C. consult with medical direction for advice.
When a patient has two or more medical conditions that are presenting symptoms at the same time, it is good to:
A. treat them all at once.
B. treat them one at a time.
C. consult with medical direction for advice.
D. just manage the ABCs and forget the rest.

D. any combination of these.
The 59-year-old female patient with slurred speech may have had:
A. a stroke.
B. an overdose.
C. a seizure.
D. any combination of these.

C. seizure.
The 46-year-old male with chest pain may have any of the following conditions except:
A. angina.
B. acute myocardial infarction.
C. seizure.
D. rib fracture.

A. internal bleeding.
A 45-year-old female patient who is vomiting coffee-ground-like material and complaining of abdominal pain probably has:
A. internal bleeding.
B. a seizure history.
C. gallbladder problems.
D. ingested a strong acid.

D. any of these.
A wheeze is a common breathing sound found in a patient having:
A. an asthma attack.
B. an allergic reaction.
C. bronchospasm.
D. any one of these.

B. abdominal pain.
There is no specific EMT intervention for the patient with:
A. chest pain.
B. abdominal pain.
C. a fractured leg.
D. a serious laceration.

C. hyperthermia.
Each one of the following is an example of patent problems that has no specific EMT intervention except:
A. headache.
B. sickle-cell crisis.
C. hyperthermia.
D. post-surgical complications.

D. all of these are correct.
The term used to describe the conclusion that an EMT makes about a patient’s condition, after assessing the patient, is called the:
A. presumptive diagnosis.
B. EMT diagnosis.
C. EMS diagnosis.
D. all of these are correct.

C. critical thinking.
The analytical process that assists the EMT in reaching a field diagnosis is referred to as:
A. active assessment.
B. passive assessment.
C. critical thinking.
D. detailed assessment.

B. administer many lab tests.
The basic approach that clinicians use to arrive at a diagnosis includes each of the following except:
A. gather information.
B. administer many lab tests.
C. consider the possibilities.
D. reach a conclusion.

C. differential diagnosis.
You are on the scene of a 60-year-old female complaining of difficulty breathing and a fever. You draw up a list of conditions that may be the cause of her problems today. This is referred to as the:
A. admission diagnosis.
B. presenting problem.
C. differential diagnosis.
D. assessment finding.

A. red flag.
The signs or symptoms that suggest the possibility of a particular problem that is very serious are referred to as a(n):
A. red flag.
B. black triage tag.
C. unstable situation.
D. none of these.

D. all of these.
When a highly experienced physician comes to a diagnosis, he or she most likely used:
A. heuristics.
B. pattern recognition.
C. shortcuts.
D. all of these.

A. narrowing down a long list.
The traditional approach to diagnosis involves:
A. narrowing down a long list.
B. jumping to conclusions.
C. taking lots of shortcuts.
D. eliminating similar conditions.

D. underconfidence.
Each of the following are considered common heuristics or biases except:
A. illusory correlation.
B. availability.
C. representativeness.
D. underconfidenc.

B. confirmation.
When an EMT is specifically looking for evidence that supports the diagnosis he or she already has in mind, he or she is committing a(n) _ bias.
A. anchoring
B. confirmation
C. satisfying
D. illusionary

B. representativeness
When a patient does not fit the “classic pattern,” such as a cardiac patient without crushing substernal chest pain radiating down the left arm, you have to be careful not to make a(n) _ error or bias.
A. confirmation
B. representativeness
C. overconfidence
D. availability

D. availability.
You once treated a 50-year-old female with a heat stroke. The next time you have a female patient and you are in a warm environment, you may be more likely to think of this as the diagnosis as opposed to more common problems, such as dehydration. This bias is referred to as:
A. overconfidence.
B. illusory correlation.
C. confirmation.
D. availability.

B. illusory correlation.
You should be skeptical about one condition being the actual cause of another condition that a patient presents with. Drawing conclusions about the cause of a diagnosis can lead to a(n):
A. anchoring adjustment.
B. illusory correlation.
C. search-satisfying bias.
D. availability bias.

A. missing out on the secondary diagnosis.
You are treating a patient who was found on the floor in the basement of his son’s house. It seems evident that he has a fractured hip as he lies on the floor in pain. If you stop the search for a diagnosis as soon as you come up with the cause of today’s problem, this can lead to:
A. missing out on the secondary diagnosis.
B. overconfidence and misdiagnosis.
C. overestimating the frequency of the problem.
D. all of these.

B. Remove the CPAP and ventilate the patient with a BVM device.
You are treating a respiratory patient who is conscious, alert, and in severe distress. He took an Albuterol treatment prior to your arrival on the scene. At this point, you have been administering CPAP for about 10 minutes. You notice that his mental status is rapidly diminishing. What should you do next?
A. Administer another bronchodilator treatment per medical control.
B. Remove the CPAP and ventilate the patient with a BVM device.
C. Lower the level of pressure in the CPAP device considerably.
D. Turn up the pressure of the oxygen going into the CPAP device.

C. A pulmonary embolism
You receive a high-priority call to meet an incoming airplane on the tarmac at the regional airport. A 60-year-old female patient with a history of deep vein thrombosis and 35 years of smoking a pack a day of cigarettes is acutely short of breath. This shortness of breath began about 20 minutes ago, and the EMTs are administering oxygen by nonrebreather mask at this time. The duration of the flight was approximately 6 hours. What do you suspect is the patient’s prehospital diagnosis?
A. An acute exacerbation of her COPD
B. The development of epiglottits
C. A pulmonary embolism
D. Acute pulmonary edema

A. the daughter has hypotension.
You arrive on the scene of a family who is very anxious because the father, a 45-year-old male, and the 17-year-old daughter were piling wood when they found a beehive. They both have numerous stings, but the daughter is experiencing signs of an anaphylactic reaction while the father is experiencing a simple allergic reaction. You will most likely find that:
A. the daughter has hypotension.
B. the father has hypertension.
C. the daughter has hives.
D. the father has swelling of the airway structures.

B. She may have attempted suicide by mixing chemicals to produce hydrogen sulfide.
You respond to a call for a woman who was described as sleeping in her car. The police have arrived, and a car window needed to be broken to gain access to the 35-year-old female because the doors were locked and she is unconscious. As you enter the vehicle, there is a strong smell of rotten eggs. What should you suspect happened?
A. The exhaust fumes were not well vented on the vehicle, causing carbon dioxide to leak.
B. She may have attempted suicide by mixing chemicals to produce hydrogen sulfide.
C. There was a cyanide leak into the vehicle.
D. She may have taken a double dose of a strong sleeping pill.

B. white blood cells
Each of the components of the blood has specialized functions. The _ are the cells that are critical in response to infection and the mediators of the body’s immune response system.
A. red blood cells
B. white blood cells
C. platelets
D. plasma

A. destruction of the pancreas.
The complications of sickle cell disease can include each of the following except:
A. destruction of the pancreas,
B. acute chest syndrome.
C. priapism.
D. stroke.

B. perfusion.
The circulation of blood throughout the body, filling the capillaries and supplying the cells and tissues with oxygen and nutrients, is called:
A. physiology.
B. perfusion.
C. bleeding.
D. metabolism.

B. Suction the airway.
You find a 19-year-old male trauma patient lying across the front seat of his sports car after a head-on collision. He is making gurgling sounds as he breathes. What should you do?
A. Ventilate him.
B. Suction the airway.
C. Hyperextend the neck,
D. Apply high-concentration oxygen.

D. long backboard
You find a 19-year-old male trauma patient lying across the front seat of his sports car after a head-on collision. Sometimes a _ can act as a full-body splint when this critical trauma patient must be immobilized quickly.
A. wheeled stretcher
B. short KED
C. warm blanket
D. long backboard

D. do all of these.
You find a 19-year-old male trauma patient lying across the front seat of his sports car after a head-on collision. On further examination, you note that the patient has two fractured femurs, a crushed pelvis, and a possible abdominal injury. You should:
A. not apply traction splints.
B. consider the PASG if your protocol allows.
C. set up an ALS intercept en route to the hospital.
D. do all of these.

C. bandage all the lacerations.
When you must minimize the scene care of a multiple-trauma patient, you can perform any of the following except:
A. suction the airway.
B. ventilate with a BVM.
C. bandage all the lacerations.
D. immobilize the cervical spin.

C. scene safety.
Even when you are trying to cut scene time for a multiple-trauma patient, the one thing you should not cut out is:
A. applying a traction splint if needed.
B. the secondary exam.
C. scene safety.
D. immobilization.

A. signs and symptoms exhibited.
You are treating a 17-year-old male patient who fell from two stories and is in severe pain. Internal bleeding is not visible, so you must base the severity of blood loss on:
A. signs and symptoms exhibited.
B. what the patient tells you.
C. the size of the contusion.
D. advice from medical direction.

D. the patient’s history of diabetes.
You are treating a 17-year-old male patient who fell from two stories and is in severe pain. For this patient, the factors on which severity of bleeding depends include all of the following except:
A. the rate of bleeding and the amount of blood loss.
B. the patient’s age and weight.
C. the ability of the patient’s body to respond and defend against blood loss.
D. the patient’s history of diabetes.

B. capillary.
Blood that oozes and is dark red is most likely from a(n):
A. vein.
B. capillary.
C. artery.
D. lymphatic vessel.

C. apply a tourniquet to the lower arm.
You are treating a 22-year-old female who slashed her wrist in an effort to end her life. The wound is spurting bright red blood, and she has decided to let you treat her. After trying direct pressure and elevation to control the bleeding to control the bleeding, the next step would be to:
A. apply a tourniquet to the leg.
B. apply an air splint to the arm.
C. apply a tourniquet to the lower arm.
D. press on the femoral artery.

D. apply a tourniquet.
You are treating a 22-year-old female who slashed her wrist in an effort to end her life. The wound is spurting bright red blood, and she has decided to let you treat her. When other methods have failed to control bleeding, you should:
A. apply the PASG.
B. apply an air splint.
C. press harder on the pressure point.
D. apply a tourniquet.

C. vomiting bile.
Signs of external bleeding include all of the following except:
A. painful, swollen, or deformed extremities.
B. a tender, rigid abdomen.
C. vomiting bile.
D. bruising.

C. fall from a height.
Examples of penetrating trauma include all of the following except:
A. handgun bullet wound.
B. carving-knife wound.
C. fall from a height.
D. screwdriver stab wound.

C. gunshot wounds.
Examples of blunt trauma include all of the following except:
A. blast injuries.
B. auto-pedestrian collisions.
C. gunshot wounds.
D. falls.

A. the same as those of shock.
You are treating a 49-year-old male patient who was assaulted. You suspect he may have some internal bleeding. The signs and symptoms of internal bleeding are:
A. the same as those of shock.
B. identical to those of external bleeding.
C. usually not present in elderly patients.
D. easy to stabilize in the field.

B. Hydrophobic
Which of the following is not considered a type of shock?
A. Hypovolemic
B. Hydrophobic
C. Cardiogenic
D. Neurogenic

C. decompensated phase of shock.
The point at which the body can no longer compensate for low blood volume is referred to as the:
A. compensated phase of shock.
B. anaphylactic phase of shock.
C. decompensated phase of shock.
D. terminal phase of shock.

C. Brain and spinal cord
What are the major components of the central nervous system?
A. Cranial nerves
B. Peripheral nerves
C. Brain and spinal cord
D. Vessels and support tissue

D. Voluntary
Which of the following is not a division of the nervous system?
A. Central
B. Peripheral
C. Autonomic
D. Voluntary

D. forehead, top, back, and upper sides
The skull is made up of the cranium and the facial bones. The cranium consists of the _ areas of the skull.
A. temporal, mandible, and maxilla
B. frontal, parietal, and distal
C. anterior, forehead, and lateral
D. forehead, top, back, and upper sides

A. vertebrae.
The bones forming the face include all of the following except:
A. vertebrae.
B. zygomatic.
C. mandible.
D. maxillae.

A. cerebrospinal fluid.
The brain and spinal cord are bathed in:
A. cerebrospinal fluid.
B. lymphatic fluid.
C. synovial fluid.
D. mucous secretion.

B. vertebrae.
The spine is divided into sections called:
A. joints.
B. vertebrae.
C. coccygeal.
D. compartments.

B. 5
The lumbar area of the spine includes _ vertebrae.
A. 4
B. 5
C. 7
D. 12

D. cholera.
A bacterial disease that can be used in an act of biological terrorism to inflict serious illness, severe diarrhea, dehydration, and electrolyte imbalance in its victims is:
A. plague.
B. anthrax.
C. ricin.
D. cholera.

B. a concussion.
A 22-year-old female struck her head and states that she feels groggy and has a headache. She most likely has:
A. an abrasion.
B. a concussion.
C. a contrecoup.
D. none of these.

D. attention deficit.
A 50-year-old male driver who was involved in an automobile collision states that he did not lose consciousness. However, a bystander says that he ran to the car after the collision and tried to talk to the driver, but said that the driver “just sat there staring off into space for a few minutes.” This patient may have any of the following except:
A. contusion.
B. concussion.
C. coup injury.
D. attention deficit.

B. amnesia.
A patient has some memory loss after a head injury. This is referred to as:
A. bruising.
B. amnesia.
C. verbally responsive.
D. “seeing stars.”

C. coup
A bruising of the brain occurs on the side of the injury. This is referred to as a(n) _ injury.
A. contrecoup
B. hematoma
C. coup
D. epidural

D. any of these.
A collection of blood within the skull or brain is called:
A. a subdural hematoma.
B. an epidural hematoma.
C. an intracerebral bleed.
D. any of these.

D. decreased respiration leading to increased cellular perfusion.
All of the following make a head injury worse except:
A. limited room for expansion inside the skull.
B. increased pressure in the skull.
C. increased carbon dioxide level in the brain.
D. decreased respiration leading to increased cellular perfusion.

B. assessing equality of strength.
An assessment strategy used to check an extremity for injury or paralysis in a conscious patient is:
A. checking for a proximal pulse.
B. assessing equality of strength.
C. checking for foot wave.
D. confirming sensitivity.

B. incubation period.
The time between exposure and the appearance of symptoms is referred to as the:
A. toxicity period.
B. incubation period.
C. transmissibility time frame.
D. infectivity window.

D. unequal.
If a patient has a brain injury with a skull fracture, the patient’s pupils tend to be:
A. equal.
B. dilated.
C. constricted.
D. unequal.

D. Increased blood pressure, decreased pulse
Which of the following describes the blood pressure and pulse for a patient with a traumatic brain injury?
A. Decreased blood pressure, increased pulse
B. Decreased blood pressure, decreased pulse
C. Increased blood pressure, increased pulse
D. Increased blood pressure, decreased pulse

A. deep lacerations or severe bruises to the scalp or forehead.
Consider the possibility of a cranial fracture whenever you note:
A. deep lacerations or severe bruises to the scalp or forehead.
B. a deformed femur.
C. facial deformity.
D. blood or other fluids in the airway.

B. 33 irregularly
The spinal column is made up of _ shaped bones.
A. 33 regularly
B. 33 irregularly
C. 12 regularly
D. 12 irregularly

B. cervical
Any blunt trauma above the clavicles may damage the _ vertebrae.
A. lumbar
B. cervical
C. sacral
D. thoracic

A. a persistent erection of the penis.
Priapism is:
A. a persistent erection of the penis.
B. spasms of the hands and feet.
C. apparent only in unconscious patients.
D. uncontrolled muscle twitches of the thighs.

B. stretched out above his head.
You are assessing a 22-year-old male who you suspect sustained a spine injury when he fell of the roof. Often with a cervical-spine injury, the patient in a supine position may have his arms:
A. straight out at his side.
B. stretched out above his head.
C. down at his side.
D. across the chest.

C. abuse or neglect.
The EMT should consider that any injury of an elderly person could be a sign of:
A. a severe fall.
B. Alzheimer’s disease.
C. abuse or neglect.
D. depression.

C. decreased ability to clear foreign substances from the lungs.
Geriatric patients usually have decreased elasticity of the lungs and decreased activity of cilia that result in:
A. increased risk of heart attack.
B. diminished activity and tolerance of physical stress.
C. decreased ability to clear foreign substances from the lungs.
D. decreased energy and tolerance of hot and cold.

D. thin, dry, wrinkled skin.
Your patient is a 70-year-old male who was found confused and walking on the side of the highway. The loss of skin elasticity and shrinking of sweat glands in this geriatric patient can cause:
A. diminished activity and tolerance of physical stress.
B. decreased ability to clear foreign substances from the lungs.
C. decreased energy and tolerance of hot and cold.
D. thin, dry, wrinkled skin.

A. decreased energy and tolerance of heat or cold.
Diminished function of the thyroid gland is one of the effects of aging. This results in:
A. thin, dry, wrinkled skin.
B. decreased energy and tolerance of heat and cold.
C. depression and loss of social support.
D. decreased strength.

C. abnormal heart rhythm.
An elderly patient falls, which may indicate a more serious problem, such as:
A. viral pneumonia.
B. acute alcoholism.
C. abnormal heart rhythm.
D. severe depression.

C. pulled out if easily done.
An unstable object impaled in the cheek wall should be:
A. stabilized from the outside.
B. stabilized from the inside.
C. pulled out if easily done.
D. stabilized from the inside and the outside.

A. supine
If possible, an 18-year-old male patient with facial fractures should be transported on a long spine board in the _ position.
A. supine
B. Trendelenburg
C. head-elevated
D. prone

D. femoral artery.
If direct pressure and a hemostatic dressing does not control bleeding from the lower leg injury is most-likely involving to the:
A. carotid artery.
B. brachial artery.
C. subclavian vein.
D. femoral artery.

A. picking embedded particles out of the cut.
Care for an open wound includes all of the following except:
A. picking embedded particles out of the cut.
B. controlling bleeding.
C. bandaging the dressing in place.
D. cleaning the wound surface.

B. pinching the nostrils together.
Your patient is a 55-year-old woman who called EMS because she has a nosebleed that has not stopped all afternoon. The best method for control of nasal bleeding is:
A. packing the nose with cotton.
B. pinching the nostrils together.
C. packing the nose with gauze.
D. applying pressure to the facial artery.

C. treating for shock.
The first step in caring for possible internal bleeding, after ensuring respiration and circulation and controlling life-threatening external bleeding, is:
A. administering liquids by mouth to the patient.
B. applying a bulky dressing.
C. treating for shock.
D. placing the patient in a sitting position.

D. laceration.
A razor blade cut is an example of a(n):
A. incision.
B. abrasion.
C. contusion.
D. laceration.

D. avulsion.
The injury in which flaps of skin and tissue are torn loose or pulled off completely is called a(n):
A. laceration.
B. amputation.
C. incision.
D. avulsion.

C. stabilize the object.
A 22-year-old female patient has an object impaled in the forearm. After the control of profuse bleeding, you should:
A. remove the object.
B. place a pressure dressing over the site.
C. stabilize the object.
D. apply firm pressure to a pressure point.

D. Pale, cool, clammy skin
Which of the following is a sign of shock?
A. High blood pressure
B. Constricted pupils
C. Slowed pulse rate
D. Pale, cool, clammy skin

A. stabilized with gauze and protected with a disposable cup.
An object impaled in the eye should be:
A. stabilized with gauze and protected with a disposable cup.
B. removed carefully and a pressure dressing applied.
C. shielded with a cup taped over the orbit.
D. removed and a dressing applied with minimum pressure.

C. covered with folded 4 X 4s.
A lacerated eyelid or injury to the eyeball should be:
A. flushed with water.
B. covered with a cold pack.
C. covered with folded 4 X 4s.
D. covered with dark patches.

A. life threatening.
All open wounds to the chest should be considered:
A. life threatening.
B. potentially infectious.
C. a low priority.
D. an indication for PASG.

C. apply a rigid collar.
Before moving a 35-year-old supine male patient with possible spinal injuries onto a long spine board, you should always:
A. align the teeth and tape the jaw in place.
B. apply a short spine board.
C. apply a rigid collar.
D. secure a KED to the patient.

A. direct pressure or pinching.
The initial effort to control bleeding from a severed neck artery should include:
A. direct pressure or pinching.
B. applying tape.
C. pressure points.
D. occlusive dressing.

B. paralysis of extremities.
The most reliable sign of spinal-cord injury in a conscious 30-year-old female patient is:
A. pain without movement.
B. paralysis of extremities.
C. pain with movement.
D. tenderness along the spine.

D. do not replace the organ but cover with a moistened dressing.
When caring for an open abdominal wound with evisceration:
A. replace the organ but cover with occlusive material.
B. replace the organ but cover with a bulky dressing.
C. do not replace the organ but cover with occlusive material.
D. do not replace the organ but cover with a moistened dressing.

C. face up with the knees bent.
A 35-year-old female patient who is in acute abdominal distress but denies any vomiting should be transported to the ED:
A. in the Trendelenburg position with legs straight.
B. in the coma position.
C. face up with the knees bent.
D. in the recumbent position with one knee bent.

C. splint it in the position found.
When a joint is locked into position, the EMT should:
A. pull traction on the extremity.
B. straighten it.
C. splint it in the position found.
D. skip splinting and transport immediately.

A. circulation to the extremity.
A splint properly applied to an extremity should prevent all of the following except:
A. circulation to the extremity.
B. an open bone injury.
C. motion of bone fragments.
D. damage to muscle and blood vessels.

B. rigid splint or sling and swathe.
A fracture to the humerus shaft is best cared for by immobilizing with a(n):
A. wrist sling.
B. rigid splint or sling and swathe.
C. air-inflated splint.
D. sling and swathe.

D. sling and swathe.
A fracture to the proximal end of the humerus is best cared for by immobilizing with a(n):
A. wrist sling and swathe.
B. padded board, sling, and swathe.
C. air-inflated splint.
D. sling and swathe.

B. keep the arm in its found position and apply a short, padded board splint.
Your 40-year-old female patient fell backward onto her left elbow and heard a loud snap. She has a distal pulse and sensation. The best way to immobilize a fractured elbow when the arm is found in the bent position and there is a distal pulse is to:
A. straighten the arm and apply an air-inflated splint.
B. keep the arm in its found position and apply a short, padded board splint.
C. keep the arm in its found position and apply an air-inflated splint.
D. straighten the arm and apply a wire-ladder splint.

B. align to anatomical position under gentle traction.
If a severe deformity exists or distal circulation is compromised when you are splinting you should:
A. push protruding bones back into place.
B. align to anatomical position under gentle traction.
C. immediately move the patient to a stretcher.
D. immobilize in the position found.

C. place the broken hand in its position of function.
The first step in immobilizing a fractured wrist with distal pulse is to:
A. correct angulation of the wrist.
B. secure a padded board splint to the wrist.
C. place the broken hand in its position of function.
D. tape the wrist.

A. legs bound together with wide cravats.
A 40-year-old female patient with a fractured pelvis should be immobilized on an orthopedic stretcher or long spine board with:
A. legs bound together with wide cravats.
B. long padded boards secured down each side of the body.
C. stabilizing sandbags placed between the legs.
D. straps securing the torso and pelvis.

D. traction
A fractured femur is best immobilized with a(n) _ splint.
A. long padded
B. air-inflated
C. rigid vacuum
D. traction

A. assess distal circulation and sensory and motor function.
Before immobilizing a fractured knee:
A. assess distal circulation and sensory and motor function.
B. straighten the angulation.
C. apply firm traction.
D. flex the leg at the knee.

C. pillow splint
The best method for immobilizing a suspected ankle fracture is a(n) _ splint.
A. traction
B. padded board
C. pillow splint
D. air-inflated

B. ligaments are torn.
A sprain is an injury in which:
A. tendons are torn.
B. ligaments are torn.
C. cartilage is crushed.
D. muscle spasm.

C. Definitive care is provided in the prehospital setting.
Which of the following is not true of open extremity injuries?
A. The skin has been broken or torn.
B. There is increased likelihood of infection.
C. Definitive care is provided in the prehospital setting.
D. Such injuries require surgery at the hospital.

D. tendons.
Muscle is attached to bone by:
A. cartilage.
B. ligaments.
C. smooth muscles.
D. tendons.

B. epidermis and dermis.
A partial thickness burn involves the:
A. epidermis.
B. epidermis and dermis.
C. epidermis, dermis, and subcutaneous layers.
D. epidermis, dermis, subcutaneous layers, and muscles.

A. 13.5 percent
The entire back of a 50-year-old female’s right arm and her entire chest have been burned. What percentage of surface burn would you report?
A. 13.5 percent
B. 18 percent
C. 27 percent
D. 36 percent

C. superficial burns covering more than 50 percent of the body surface.
A moderate burn involves:
A. partial and full thickness burns of the face and hands.
B. full thickness burns over less than 2 percent of the body surface.
C. superficial burns covering more than 50 percent of the body surface.
D. superficial burns covering less than 20 percent of the body surface.

A. swelling and blistering.
Partial thickness burns cause:
A. swelling and blistering.
B. nerve damage.
C. slight swelling.
D. scarring.

D. brush away the lime.
A 40-year-old male patient is suffering from chemical burns to the skin caused by dry lime. After putting on PPE your next step should be to:
A. wash the area with running water.
B. remove the lime with phenol.
C. remove the lime with alcohol.
D. brush away the lime.

D. 20
Acid burns to the eyes should be flooded with water for at least _ minute(s).
A. 1
B. 5
C. 10
D. 20

A. rule of nines.
A method for estimating the extent of a burn is the:
A. rule of nines.
B. rule of percentages.
C. rule of degree.
D. burn assessment rule.

D. 16
If a woman is having her first baby, the first stage of labor will usually last an average of _ hours.
A. 4
B. 8
C. 12
D. 16

B. 2 to 3
During the most active stage of labor, the uterus usually contracts every _ minutes.
A. 1 to 2
B. 2 to 3
C. 5 to 9
D. 10 to 15

C. puncture it with a finger.
If the amniotic sac does not break during delivery, the EMT should:
A. do nothing; it will break after birth.
B. quickly remove it with sterile scissors.
C. puncture it with a finger.
D. transport the patient immediately.

B. support the baby’s head.
To assist the mother in delivering the baby, gently:
A. pull at the baby’s shoulders.
B. support the baby’s head.
C. rotate the baby to the left or right.
D. push your gloved hand into the vagina.

B. Suction the baby’s mouth and nose.
After the delivery, which of the following should be done first?
A. Clamp and cut the cord.
B. Suction the baby’s mouth and nose.
C. Lay the baby on his back.
D. Lift the baby by the feet and slap the buttocks.

C. vigorously rub the baby’s back.
Following delivery, if spontaneous respiration does not begin after suctioning the baby’s mouth and nose, the EMT should first:
A. begin mouth-to-mouth-and-nose resuscitation.
B. apply mechanical resuscitation with 100 percent oxygen.
C. vigorously rub the baby’s back.
D. transport immediately, administering 100 percent oxygen.

D. pull the patient out the side windows.
You are on the scene of a car crash. To assist with patient access, the EMT should initially do each of the following except:
A. try opening each car door.
B. roll down windows.
C. ask the patient to unlock the doors.
D. pull the patient out the side windows.

A. Requirements for speed of removal
What dictates the specific technique used for spinal immobilization of a patient at a collision extrication?
A. Requirements for speed of removal
B. How many rescuers are available for removal
C. Whether an airbag has deployed
D. None of these

D. 4
Which step in the phases of the rescue process is out of order?

  1. Sizing up the situation
  2. Gaining access
  3. Disentangling the patient
  4. Stabilizing the vehicle
    A. 1
    B. 2
    C. 3
    D. 4

D. number of ambulances in the region.
Considerations for size-up of a collision include all of the following except:
A. potential hazards.
B. need for additional EMS units.
C. number of patients involved.
D. number of ambulances in the region.

C. wear highly visible clothing.
To minimize injuries at a collision, the EMT should:
A. use a limited number of tools.
B. deactivate the safety guards on tools.
C. wear highly visible clothing.
D. wait until the police arrive on the scene.

B. is often over 1,200 degrees Fahrenheit.
The vehicle’s catalytic converter can be a source of ignition at a collision because it:
A. often emits sparks.
B. is often over 1,200 degrees Fahrenheit.
C. may leak gasoline.
D. pumps high-pressure fumes.

A. opening the trunk and cutting the battery cable.
The three-step process of disentanglement described in the text includes all of the following except:
A. opening the trunk and cutting the battery cable.
B. creating exits by displacing the doors and roof posts.
C. disentangling occupants by displacing the front end.
D. gaining access by disposing of the roof.

B. It it must be disrupted, disconnect the ground cable from the battery.
Which of the following is true about dealing with a collision vehicle’s electrical system?
A. It should be disabled by cutting a battery cable.
B. If it must be disrupted, the ground cable from the battery.
C. Cutting the battery cable can assist in rescue operations.
D. Disconnecting the ground cable produces a spark that can ignite battery gases.

D. margin of safety.
When positioning flares, use a formula that includes the stopping distance for the posted speed plus the:
A. angle of the road.
B. radius of the danger zone.
C. reaction distance.
D. margin of safety.

C. displace doors and roof posts.
Once the vehicle has been stabilized, the next part of an extrication procedure for patient rescue is to:
A. dispose of the vehicle’s roof.
B. displace the front end of the vehicle.
C. displace doors and roof posts.
D. do none of these.

A. How many persons are injured
The Emergency Medical Dispatcher (EMD) has told you the exact location of a collision, how many and what kinds of vehicles are involved, and any known hazards. What other information would you like the EMD to provide concerning this collision?
A. How many persons are injured
B. The direction designator
C. How to contact the person who reported the collision
D. The nature of the emergency

C. What’s the patient’s name?
Which of the following is an unnecessary question for the Emergency Medical Dispatcher to ask when receiving a call for help?
A. How old is the patient?
B. What’s the patient’s gender?
C. What’s the patient’s name?
D. Is the patient conscious?

A. the continuous sound of a siren could worsen a patient’s condition.
When operating a siren, be aware that:
A. the continuous sound of a siren could worsen the patient’s condition.
B. all motorists will hear and honor your signal.
C. the siren must be continuously used when carrying injured patients.
D. it is necessary to pull up close to vehicles and sound the siren.

D. danger zones.
All of the following are factors that affect an ambulance’s response to a scene except:
A. the day of the week.
B. the time of day.
C. detours.
D. danger zones.

B. stop the ambulance.
If a patient develops cardiac arrest during transport, have the operator of the ambulance:
A. adjust the ambulance’s speed.
B. stop the ambulance.
C. contact the hospital emergency department.
D. assist you with CPR.

C. check the see what is to be done with the patient.
When transferring a nonemergency patient to the emergency department personnel, you should:
A. wait for emergency staff to call for the patient.
B. off-load, wheel to the designated area, and stay with the patient.
C. check to see what is to be done with the patient.
D. off-load, wheel to the designated area, and leave the patient.

C. obtain your release.
The last step when transferring a patient is to:
A. wait to help the emergency department staff.
B. transfer the patient’s valuables.
C. obtain your release.
D. transfer patient information.

C. prepare the prehospital care report.
At the hospital, as soon as you are free from patient-care activities, you should:
A. notify dispatch you are back in service.
B. quickly clean the vehicle’s patient compartment.
C. prepare the prehospital care report.
D. check on patient status with the emergency department.

D. Unopened gauze bandages
Which of the following is not a biohazard?
A. Suction catheters
B. Contaminated dressings
C. Blood-soaked linen
D. Unopened gauze bandages

D. Completeness and operability of equipment is ensured.
Which of the following is not a benefit of an EMS/hospital equipment exchange program?
A. Patients are not subjected to injury-aggravating movement.
B. Delay of the crew at the hospital is prevented.
C. Ambulances can return to quarters fully equipped.
D. Completeness and operability of equipment is ensured.

D. law.
Vigorous cleaning of the ambulance while parked at the hospital is prevented or restricted by all of the following limitations except:
A. time.
B. equipment.
C. space.
D. law.

C. 10
You have just assisted the mother in delivering a healthy newborn. The first clamp placed on the umbilical cord should be about _ inches from the baby.
A. 2
B. 5
C. 10
D. 12

A. clamp the cord again.
If bleeding continues from the umbilical cord after clamping and cutting, the EMT should:
A. clamp the cord again.
B. unclamp the cord and tie.
C. apply a sterile dressing.
D. transport the baby immediately.

A. 20 minutes
Which of the following is the maximum amount of time the EMT should wait for the placenta to be delivered before transporting the mother and infant?
A. 20 minutes
B. 45 minutes
C. 1 hour
D. 2 hours

A. 500 cc
Delivery of the placenta is usually accompanied by the loss of no more than _ of blood.
A. 500 cc
B. 600 cc
C. 800 cc
D. 1,000 cc

B. position a sanitary napkin.
The first step to control vaginal bleeding after birth is to:
A. apply a pressure dressing.
B. position a sanitary napkin.
C. massage the uterus.
D. pack the vagina with sterile gauze.

B. perform under stress.
Along with physical and mental fitness, ambulance operators should be able to:
A. move other vehicles off the roadway.
B. perform under stress.
C. always be up for a run.
D. justify feelings of superiority.

D. absolute right of way.
When driving an ambulance, you should realize that there are no state laws that grant the:
A. use of controlled additional speed.
B. passage through traffic signals.
C. privilege of special parking at the scene.
D. absolute right of way.

B. gently push up on the baby’s head or buttocks to keep pressure off the cord.
If, during birth, the umbilical cord presents first, you should:
A. gently push the cord back into the vagina.
B. gently push up on the baby’s head or buttocks to keep pressure off the cord.
C. gently push on the cervix.
D. clamp and cut the cord.

C. transport immediately and provide oxygen.
If an arm presentation without a prolapsed cord is noted, the EMT should:
A. reach up the vagina and turn the baby.
B. do nothing; the delivery will be normal.
C. transport immediately and provide oxygen.
D. insert a gloved hand and push back the vaginal wall.

C. 37th
A baby is considered premature if it weighs less than 5 pounds or is born before the _ week of pregnancy.
A. 35th
B. 36th
C. 37th
D. 38th

D. Lack of airway passage swelling
Which of the following signs is not an indication that a child has an airway disease?
A. Wheezing
B. Breathing effort on exhalation
C. Rapid breathing
D. Lack of airway passage swelling

C. fontanelle.
The soft spot on top of an infant’s head is called a:
A. contusion.
B. depression site.
C. fontanelle.
D. suture.

B. conserve patient body heat.
After calming a snake-bite victim and treating for shock, you locate the fang marks. Next, you should:
A. immobilize the affected extremity.
B. conserve patient body heat.
C. cleanse the wound site.
D. apply a tourniquet to the extremity.

A. administering oxygen by nonrebreather.
A 49-year-old male patient who was installing a new roof on a very hot day complains of severe muscle cramps in his legs and feels faint. You should move the patient to a cool place and begin care by:
A. administering oxygen by nonrebreather.
B. transporting the patient immediately.
C. administering oxygen by nasal cannula.
D. giving the patient water.

D. provide resuscitation and transport to the hospital.
In cases of sudden infant death syndrome, the EMT should:
A. hunt for evidence of child abuse.
B. hold off on care and simply transport the body.
C. declare the patient dead.
D. provide resuscitation and transport to the hospital.

C. Injuries to the center of the back and upper arms
Which of the following might lead you to consider child abuse?
A. Multiple skinned knees
B. A burn from chewing an electric cord
C. Injuries to the center of the back and upper arms
D. Both ankles are sprained

D. affected area feels frozen, but only on the surface.
Your patient was outdoors on a very cold, windy day chopping wood. You notice that he did not have gloves on or a hat. You suspect he may have frostbite. With frostbite, the:
A. underlying tissues feel frozen to the touch.
B. skin is commonly mottled and grayish blue.
C. skin blisters and swells.
D. affected area feels frozen, but only on the surface.

A. shivering.
The initial sign of hypothermia is:
A. shivering.
B. skin is red colored.
C. skin is blistered and smells.
D. white, waxy skin.

A. unconsciousness and absence of discernible vital signs.
Extreme hypothermia is characterized by:
A. unconsciousness and absence of discernible vital signs.
B. shivering, numbness, and drowsiness.
C. flaccid muscles.
D. rapid breathing.

C. dehydration.
Common causes of shock in children include infections, trauma, blood loss, and:
A. meningitis.
B. heart failure.
C. dehydration.
D. lack of insulin.

C. dehydration.
A sunken fontanelle may indicate:
A. elevated intracranial pressure.
B. hypertension.
C. dehydration.
D. fever.

A. blow-by
Flowing ozygen over the face of a small child so it will be inhaled is referred to as the _ technique.
A. blow-bye
B. rebreather
C. supplemental
D. flow-by

B. a seizure.
A complication of a rapidly rising temperature in a child is often:
A. hyperactivity.
B. a seizure.
C. vomiting.
D. ringing in the ears.

D. croup.
A group of viral illnesses that results in inflammation of the larynx, trachea, and bronchi is called:
A. meningitis.
B. anaphylaxis.
C. asthma.
D. croup.

D. periosteum.
The strong, white, fibrous material covering the bones is called the:
A. shell.
B. perineum.
C. marrow.
D. periosteum.

C. dislocation.
The coming apart of a joint is referred to as a:
A. fracture.
B. sprain.
C. dislocation.
D. strain.

A. strain.
Overstretching or overexertion of a muscle is called a:
A. strain.
B. sprain.
C. dislocation.
D. fracture.

B. hot
In a heat emergency, the patient with _, dry skin requires rapid cooling and immediate transport.
A. pale
B. hot
C. cool
D. warm

A. blotchy and grayish yellow.
As frostbite progresses and expsure continues, the skin turns from white and waxy toL
A. blotchy and grayish yellow.
B. pale and pinkish red.
C. deep blue at the nose and cheeks.
D. cherry red, except for the extremities.

C. cover the frostbitten area, handle it as gently as possible, and transport the patient.
To treat a patient with deep frostbite:
A. immerse the limb in 105 degree (F) water and transport the patient.
B. rub snow on the drozen area or apply cold packs if available.
C. cover the frostbitten area, handle it as gentle as possible, and transport the patient.
D. protect the frostbitten area by keeping it cold and transport the patient.

C. no joint or lower leg injury.
Your patient fell onto his right leg. The indications for a traction splint are a painful, swollen, deformed mid-thigh with:
A. an open fracture of the lower leg.
B. extensive blood loss and shock.
C. no joint or lower leg injury.
D. either ankle or knee involvement.

C. Hazardous Materials Technician.
The level of training established by OSHA for those who actually plug, patch, or stop the release of a hazardous material is called:
A. First Responder Awareness.
B. First Responder Operations.
C. Hazardous Materials Technician.
D. Paramedic.

C. upwind/same level.
At a hazardous materials incident site, the “safe zone” should be located:
A. downwind/downhill.
B. downwind/same level.
C. upwind/same level.
D. upwind/downhill.

B. a Material Safety Data Sheet.
A resource that must be maintained at the work site by the employer and that must be available to all employees working with hazardous materials is called:
A. NFPA 704.
B. a Material Safety Data Sheet.
C. the Emergency Response Guidebook.
D. a shipping manifest.

B. monitoring and rehabilitating the hazmat team members.
The responsibilities of EMS personnel at a hazmat incident include caring for the injured and:
A. staging personnel and equipment in the warm zone.
B. monitoring and rehabilitating the hazmat team members.
C. decontaminating those exiting the hot zone.
D. notifying medical direction about the incident.

D. has treatable life-threatening illness or injuries.
You are assigning triage tags to muliple patients at a bus crash. Categorizing a patient as Priority 1 at an MCI means that the patient:
A. is serious but does not have life-threatening injuries or illness.
B. has minor musculoskeletal or soft-tissue injuries.
C. is dead or fatally injured.
D. has treatable life-threatening illness or injuries.

A. staging
The MCI supervisor responsible for communicating with the treatment areas to determine the number and priority of the patients in each respective treatment area is called the _ supervisor.
A. staging
B. triage
C. treatment
D. transportation

C. yellow
A Priority 2 patient would be color-coded as _ to identify his or her treatment priority at an MCI.
A. black
B. green
C. yellow
D. red

C. 3.
At an MCI, the patients who are assessed as having minor injuries should be categorized as Priority:
A. 1.
B. 2.
C. 3.
D. 4.

D. Laryngopharynx
Upon insertion of an oropharyngeal airway (OPA), the distal tip of the device should be positioned in what anatomical space?
A. Oropharynx
B. Nasopharynx
C. Hyperpharynx
D. Laryngopharynx

C. Bronchioles
The administration of a beta(2) specific drug to a patient with respiratory distress is done to effect a change to what portion of the respiratory tree?
A. Alveoli
B. Vocal cords
C. Bronchioles
D. Mainstem bronchi

A. “I . . . . . . can’t . . . . . b-breathe.”
Which of the following statements from the patient would best indicate severe difficulty in breathing?
A. “I . . . . . . can’t . . . . . b-breathe.”
B. “I am struggling to catch my breath . . . . real bad.”
C. “I feel like I’m breathing through a straw.”
D. “I can’t breathe regular cause my chest is hurting.”

D. a 68-year-old male seizing with sonorous sounds heard with each breath.
Which of the following patients is most likely about to lose total airway patency, thereby requiring immediate intervention by the EMT?
A. A 98-year-old female with brain cancer who is confused.
B. A 52-year-old male who is intoxicated and has recurrent vomiting.
C. A 29-year-old female in active labor who is complaining of severe pain.
D. A 68-year-old male seizing with sonorous sounds hear with each breath.

C. Oropharyngeal suctioning
Upon assessment of the patient’s airway, you note significant gurgling with each breath. What would be the next appropriate airway intervention the EMT should perform to eliminate this finding?
A. Head tilt-chin lift
B. Modified jaw thrust
C. Oropharyngeal suctioning
D. Insertion of a nasopharyngeal airway

D. application of a head tilt-chin lift
You are managing a female patient who has been in an auto accident and is now unresponsive. As you manage her airway, which of the following interventions would you likely not employ?
A. Insertion of an OPA
B. Upper airway suctioning
C. A modified jaw-thrust maneuver
D. Application of a head tilt-chin lift

B. You should measure the OPA from the center of the mouth to the angle of the jaw.
When sizing an OPA for insertion in a 48-year-old male, which sizing technique is most appropriate?
A. You should measure the OPA from the tip of the nose to the angle of the jaw.
B. You should measure the OPA from the center of the mouth to the angle of the jaw.
C. The OPA should fit between the corner of the mouth and the angle of the jaw.
D. The OPA should be large enough that the lips do not seal shut when properly inserted.

D. For as long as needed to remove vomitus
During your initial airway management, your patient needs suctioning for prolonged vomiting. How long should you provide this oral suctioning?
A. For 5 to 10 seconds
B. No greater than 10 seconds
C. Less than 25 seconds initially
D. For as long as needed to remove vomitus

A. Inhalation
The mechanical process of respiration (ventilation) occurs in two phases. Which phase sees a negative pressure inside the thorax in order to make the ventilation process work?
A. Inhalation
B. Exhalation
C. Pause before inhalation
D. Pause before exhalation

C. Alveolar space ventilation
A patient with severe asthma is having trouble breathing and adequately ventilating. What portion of normal tidal volume is this patient most likely experiencing the greatest disturbance in?
A. Base respiratory rate
B. Dead space ventilation
C. Alveolar space ventilation
D. Overall ventilation per minute

B. Absent breath sounds in the bases of the lungs
A patient with a change in his ventilation status secondary to a severe chest wall injury will likely display what specific early finding indicative of poor alveolar ventilation?
A. Altered mental status
B. Absent breath sounds in the bases of the lungs
C. Poor pulse oximetry and a change in the heart rate
D. Dropping systolic blood pressure and narrowing pulse pressure

A. Ventilation
A patient is found with a respiratory rate of 8 per minute, absent basal breath sounds, a pulse ox reading of 83 percent, and cyanotic lips and fingernails. This patient is in immediate need of what intervention?
A. Ventilation
B. Oxygenation
C. Airway suctioning
D. Semi-Fowler’s positioning

B. Full speech patterns
Which of the following findings would still be present in a patient suffering from respiratory distress?
A. Cyanotic nail beds
B. Full speech patterns
C. Altered mental status
D. Pulse oximeter of 85 percent

A. Suction the airway
You are managing a patient who was in a fight and was struck in the head with a large pipe. You find the patient to be unresponsive, apneic, bradycardic, bleeding from the head, and bleeding into the airway from oral trauma. Assuming you had all your EMT equipment available to you, what should be the initial action?
A. Suction the airway
B. Immobilize the patient
C. Provide positive pressure ventilation
D. Administer high-flow oxygen at 15 lpm

C. The patient has breath sounds in all lung fields.
Which of the following clinical indications would first appear to indicate that the patient is being adequately ventilated manually with a BVM?
A. The chest wall moves with each breath.
B. The ventilations are at a rate of 12/minute.
C. The patient has breath sounds in all lung fields.
D. The vital signs and pulse oximeter start to normalize.

C. Nonrebreather mask
If the EMT wishes to deliver the highest concentration of oxygen possible to the patient with spontaneous breathing, what oxygenation adjunct should she use?
A. Venturi mask
B. Simple face mask
C. Nonrebreather mask
D. Partial nonrebreather mask

D. Systolic blood pressure of 88 mmHg
You are preparing to treat a patient with CPAP for respiratory distress. Upon assessment, however, you observe the following findings. Which of these would serve as a contraindication for the use of CPAP?
A. Respiratory rate of 22/minute
B. Pulse oximeter reading of 86 percent
C. Heart rate of 112/minute and irregular
D. Systolic blood pressure of 88 mmHg

C. Someone with wheezing from an asthma attack
Which of the following patients has a ventilatory status that would most benefit from a prescribed inhaler?
A. A patient with right lower lobe pneumonia
B. A patient with dyspnea following a heart attack
C. Someone with wheezing from an asthma attack
D. An apneic patient following a traumatic brain injury

C. Cystic fibrosis
What childhood respiratory disease is characterized by the production of thick mucus from the airways that may lead to respiratory distress?
A. Asthma
B. Emphysema
C. Cystic fibrosis
D. Chronic bronchitis

A. Pulmonary edema
A patient complaining of respiratory distress is also found to have chest pressure, crackles to the chest with auscultation, distended neck veins, and edema of the lower extremities. This clinical picture best fits what field diagnosis?
A. Pulmonary edema
B. Acute asthma attack
C. Myocardial infarction
D. Spontaneous pneumothorax

B. Lungs
If a patient has suffered a myocardial infarction to the right ventricle, blood flow to what structure may become impaired initially?
A. Aorta
B. Lungs
C. Brain
D. Pulmonary veins

D. Chest pain
The best known and most common finding associated with a patient suffering an acute coronary syndrome is what?
A. Nausea
B. Dyspnea
C. Sweating
D. Chest pain

A. A heart rate of 48/min
A patient experiencing bradycardia would have what finding?
A. A heart rate of 48/min
B. A pulse rhythm that is irregular
C. A respiratory rate that is 6/min
D. A pulse pressure less than 20 mmHg

B. Elderly females
Of the following segments of the population, which is most likely to experience atypical presentations of a myocardial infarction?
A. Asian males
B. Elderly females
C. Younger African-Americans
D. American Indian descendants

A. Apply oxygen
EMTs are treating a patient suspected of experiencing a myocardial infarction. Of the following options, which one should the EMT perform first (assuming all could be done immediately)?
A. Apply oxygen
B. Administer nitroglycerin
C. Administer baby aspirin
D. Attach the AED and hit “analyze”

B. Erectile dysfunction
The patient’s use of which drug class of medications can be problematic if the EMT wishes to help an MI patient administer a sublingual nitroglycerin?
A. Anti-anginals
B. Erectile dysfunction
C. Metered-dose inhalers
D. Cholesterol-lowering drugs

A. Headache
Which of the following conditions is least likely to be present in a patient suffering a myocardial infarction?
A. Headache
B. Diaphoresis
C. Tachycardia
D. Respiratory distress

C. Allow the patient to take his wife’s nitroglycerin
An elderly male patient is experiencing a myocardial infarction. Which one of the following interventions should the EMT not perform?
A. Administer oxygen
B. Have the patient chew baby aspirin
C. Allow the patient to take his wife’s nitroglycerin
D. Place the patient in a semi-Fowler’s position

B. Asthma
Assuming all of the following patients were experiencing an MI, which patient’s medical history would preclude her from receiving aspirin administration?
A. Stroke
B. Asthma
C. Diabetic
D. Hypertension

D. Myocardial infarction
A patient with coronary artery disease is predisposed to what coronary event?
A. CVA
B. Seizures
C. Tachycardia
D. Myocardial infarction

A. Fibrinolytic
Expedient transport of a patient experiencing an MI to the hospital is recommended so the hospital can administer what kind of medication in the hopes of stopping the MI progression?
A. Fibrinolytic
B. Beta(2) specific
C. Antidysrhythmic
D. Antihypertensive

C. “Nitroglycerin increases blood flow to your heart muscle.”
A patient with cardiac-type complaints asks the EMTs why they are administering nitroglycerin to him. What would be an appropriate response?
A. “Nitroglycerin makes it easier to breathe.”
B. “Nitroglycerin makes the blood pressure drop.”
C. “Nitroglycerin increases blood flow to your heart muscle.
D. “Nitroglycerin causes the heart to beat harder and raise blood pressure.

A. Pulmonary edema
A patient with a history of congestive heart failure is likely to have what other emergency when she deteriorates?
A. Pulmonary edema
B. Hypertensive crisis
C. Cerebral vascular accident
D. Severe headache with visual disturbances

C. Provide four 81-mg baby aspirin
You are caring for a 59-year-old male thought to be experiencing a heart attack. The patient initially described the pain as substernal and dull in characteristics. The patient now, however, is becoming more and more lethargic and difficult to wake up. You also note the patient’s heart rate is becoming irregular. Vitals are blood pressure of 112/92, heart rate is 82 and irregular, and respirations are 20. During the ongoing management of this MI, what part of normal MI management should you elect not to perform?
A. Administer oxygen
B. Ready the AED for application
C. Provide four 81-mg baby aspirin
D. Contact ALS for intercept en route to the hospital.

A. Human failure in proper use
What is the most common reason that an AED may shock a patient inappropriately?
A. Human failure in proper use
B. Inappropriate sensing of electrodes on the chest
C. Inability of the machine to charge quick enough to defibrillate V-fib
D. Failure on the AED processor to determine V-fib is either present or absent

C. Ventricular fibrillation
The most common lethal cardiac rhythm a patient experiencing sudden cardiac arrest displays is what?
A. Asystole
B. Atrial fibrillation
C. Ventricular fibrillation
D. Sinus tachycardia with ventricular ectopy

B. Initiate compressions
You arrive on-scene and find a patient who is unresponsive with no palpable pulse and no breathing. Bystanders state the patient has been in this condition for about 7 or 8 minutes. Assuming the EMT has all of his equipment immediately available, what should be done first?
A. Administer oxygen
B. Initiate compressions
C. Provide two positive pressure ventilations
D. Call medical direction to pronounce the patient

B. Asses for a carotid pulse
You are using an AED on an adult patient in cardiac arrest. After one particular analysis the AED indicates “no shock advised.” What is your next immediate action?
A. Remove the AED pads
B. Assess for a carotid pulse
C. Charge the AED capacitators
D. Start compressions immediately

D. Diminished breath sounds
You are assessing a patient who fell from a tree. The patient is conscious and complains primarily of pain to his right arm, although he admits to having dyspnea as well. You note he has an open fracture of the right humerus with moderate bleeding controlled with pressure. He also has diminished breath sounds to the right thorax, a pulse of 92 percent on ambient air, and a narrowing pulse pressure. Of the above findings, which is so severe that it would categorize the patient as a “high priority”?
A. Pain to the right leg
B. Open humerus fracture
C. Narrowing pulse pressure
D. Diminished breath sounds

A. Focus only on the injury site
A patient has injured the index finger of her left hand while trying to drive a nail with a hammer, accidentally hitting her finger instead. Other than the soft-tissue trauma and skeletal deformity, no other signs of trauma are present. How should the secondary assessment proceed on this patient?
A. Focus only on the injury site
B. Complete a full head-to-toe assessment
C. There is no need to perform a secondary assessment
D. Assess both arms and hands, but not the torso or lower extremities.

A. Painful
While orienting a new EMT hire to your EMS system, you quiz him over general knowledge and ask him to define what the mnemonic “DCAP-BTLS” stands for. Which of the new EMT’s answers below is incorrect?
A. Painful
B. Swelling
C. Contusions
D. Lacerations

B. Arterial
A teenage patient has sustained soft-tissue trauma while using woodworking tools in shop class. The patient, upon your arrival, has a damp cloth held over his forearm. When he removes this cloth, bright red blood comes spurting out. This would be what kind of bleeding?
A. Venous
B. Arterial
C. Capillary
D. Arteriovenule

D. Hypovolemic
A patient with soft-tissue trauma has lost a significant amount of blood. If this patient enters into a hypoperfusion syndrome (shock), what is the likely etiology?
A. Neurologic
B. Distributive
C. Cardiogenic
D. Hypovolemic

C. Blood pressure
You are managing an elderly patient who has sustained a deep laceration to the leg with bleeding. The patient has the following vitals: blood pressure 102/88, heart rate is 98/min, respirations are 22/min, and the skin is cool and clammy. Which of the above findings best represents a developing shock state?
A. Heart rate
B. Skin findings
C. Blood pressure
D. Respiratory rate

B. Crush injury
Of the following types of soft-tissue trauma, which one could present as either an open soft-tissue injury or a closed soft-tissue injury?
A. Contusion
B. Crush injury
C. Arterial laceration
D. Superficial abrasion

C. Elevate the limb above the heart
You are treating a patient who has sustained a high pressure injection injury to the hand while at his construction job. Prior to transport you have provided oxygen and immobilized the hand. As the EMT, what additional intervention should you provide en route?
A. Apply heat
B. Apply cold
C. Elevate the limb above the heart
D. Lower the limb below the heart

A. Treat for shock
Which of the following interventions should always be provided to any patient suffering from a soft-tissue injury?
A. Treat for shock
B. Extremity immobilization
C. Full spinal immbolization
D. Transport the patient with lights and sirens

A. Nasal cannula at 6 lpm
A child was running with a broken stick in her hand when she suddenly fell, and the stick she was carrying became impaled through her mouth and into her hard palate. Including proper positioning and stabilization of the object, how should the EMT apply oxygen?
A. Nasal cannula at 6 lpm
B. Simple face mask at 10 lpm
C. Pediatric face mask at 12 lpm
D. Nonrebreather mask at 15 lpm

C. 32 percent
A 32-year-old male patient has sustained a burn to his entire left arm, the front of his chest and abdomen, and half of his right arm. What is the approximate percentage of his burned skin?
A. 18 percent
B. 30 percent
C. 32 percent
D. 36 percent

D. Material such as plastic wrap
During an MCI where your equipment is limited, you are treating a chest trauma patient and you elect to apply an occlusive dressing to an open chest wound. The normal occlusive dressings for this purpose have already been used up. What other type of material could substitute for this purpose?
A. Trauma dressing
B. Sterile burn sheet
C. 4″ x 4″ gauze pads
D. Material such as plastic wrap

B. To maintain good lung expansion
What is the most important clinical reason that a flail segment should be properly stabilized as early as possible?
A. To reduce pain
B. To maintain good lung expansion
C. To prevent a hemothorax from forming
D. To prevent a pneumothorax from forming

C. Commotio cordis
Of the following chest injuries, which one would have the least effect on the quality of breath sounds heard with a stethoscope?
A. Hemothorax
B. Pneumothorax
C. Commotio cordis
D. Traumatic asphyxia

C. Femur
Which of the following isolated fractures would be best treated with a traction splint?
A. Tibia
B. Pelvis
C. Femur
D. Humerus

B. Suction the airway
You are caring for a patient who fell off the back of a dirt bike. The patient has blood in the airway and an obvious fracture to the thigh. His breathing is labored and his mental status is deteriorating rapidly. What treatment should the EMT render first?
A. Apply oxygen
B. Suction the airway
C. Immobilize the fractured femur
D. Treat the patient’s altered mental status

A. Altered mental status
In a trauma patient, what is typically the first finding that is consistent with a brain injury?
A. Altered mental status
B. Systolic hypertension
C. Neuromuscular deficit
D. Slowing of the pulse rate

B. Systolic pressure of 86 mmHg
Which of the following patient’s clinical conditions meets the physiologic criteria for determining if the patient is a high priority?
A. GCS of 15
B. Systolic pressure of 86 mmHg
C. Pupils are 2 to 3 mm in diameter
D. The patient demonstrates weakness to one extremity

D. Call for aeromedical transport to the trauma center
You are caring for a multisystem trauma victim who is unresponsive. You are currently 10 minutes from an urgent care facility, 35 minutes from a small community hospital, and 1.5 hours from a trauma center. Aeromedical transport could be at your location in 10 minutes, with a 20-minute transport time to the trauma center. What is the best way to provide transport?
A. Transport to the urgent care facility
B. Transport to the trauma center by ground
C. Transport to the small community hospital
D. Call for aeromedical transport to the trauma center

C. Conduction
If a patient falls through the ice on a lake into the cold water, the patient will lose the majority of her body heat through what mechanism?
A. Elimination
B. Respiration
C. Conduction
D. Evaporation

C. Initiate active external cooling
You are caring for a male patient who was found sitting in the stands at a baseball game. The patient is disoriented, skin is warm to the touch, the pulse is strong and bounding, sweating is absent, and the skin is reddened. Along with oxygen and moving the patient out of the sun, what other intervention should you perform as the EMT?
A. Attach the AED
B. Provide cool fluids by mouth
C. Initiate active external cooling
D. Allow the patient to drink an electrolyte solution

B. OPQRST
While assessing a medical patient, what mnemonic will assist the EMT with finding out particularities of the chief complaint?
A. AVPU
B. OPQRST
C. SAMPLE
D. DCAP-BTLS

D. Determination of patient priority
Which of the following is not considered to be one of the components of the secondary exam in a responsive patient with a medical emergency?
A. Baseline vital signs
B. Focused physical exam
C. History of the present illness
D. Determination of patient priority

B. Ask the patient which thing is bothering him the most
Upon initial questioning of a patient, the patient is complaining of multiple things: chest pain, dyspnea, and abdominal pain. What should be the next course of action for the EMT?
A. Ask the patient which thing is bothering him the most
B. Ask the patient which thing is bothering him the most
C. Treat the patient as if he has three chief complaints
D. Assume that the chest pain is the chief complaint due to its potential severity

A. An 86-year-old unresponsive diabetic patient
Which of the following medical patients will the EMT perform a rapid physical exam on?
A. An 86-year-old unresponsive diabetic patient
B. A 9-year-old boy with asthma who is scared
C. A 32-year-old male with respiratory distress
D. A 60-year-old psych patient threatening to kill himself

D. Reticular activating system
A patient with an injury or dysfunction to what region of the brain may have problems staying awake or paying attention?
A. Cerebellum
B. Hypothalamus
C. Foramen magnum
D. Reticular activating system

A. Insulin
The patient with diabetes has a pathophysiologic problem with what hormone?
A. Insulin
B. Glucose
C. Dopamine
D. Norepinephrine

C. Hypoglycemia
A diabetic patient takes her insuling medication regularly and watches what she eats. However, one day she is not feeling well and she vomits a few times after breakfast. What medical emergency related to diabetes could this cause?
A. Hypertension
B. Hypotension
C. Hypoglycemia
D. Hyperglycemia

B. Brain
Within the body of the diabetic patient, what organ is most sensitive to lowered levels of glucose?
A. Liver
B. Brain
C. Heart
D. Pancreas

B. 80 mg/dL
A diabetic patient may be considered hypoglycemic if he has an altered mental status and a blood sugar level less than what value?
A. 60 mg/dL
B. 80 mg/dL
C. 100 mg/dL
D. 120 mg/dL

A. Manual airway technique
You are caring for a diabetic patient with low blood sugar. The patient is unresponsive with sonorous airway sounds. He has a rapid pulse and peripheral perfusion is intact. Which of the following interventions is of utmost concern for this patient?
A. Manual airway technique
B. Suctioning out the airway
C. Providing high-flow oxygen
D. Administration of oral glucose

B. Generalized
A patient with what type of seizure is most likely to present as unresponsive?
A. Partial
B. Generalized
C. Focal motor
D. Jacksonian seizures

B. Loosen restrictive clothing and protect the patient from injury
You are caring for a patient who is experiencing a generalized seizure. During the tonic-clonic phase, which of the following management strategies should you employ?
A. Apply high-flow oxygen via nonrebreather mask and position the patient supine
B. Loosen restrictive clothing and protect the patient from injury
C. Apply oral glucose and place the patient in the recovery position
D. Restrain the patient physically to avoid harm and transport immediately

A. Stroke
You are assessing a patient who has confusion, difficulty in speaking, and is unable to get out of bed due to left-sided weakness. The symptoms, the family states, occurred very suddenly. The blood pressure is 210/130, the pulse is 98, and respirations are 18. As an EMT, what should be your most likely field impression for the patient’s condition?
A. Stroke
B. Seizure
C. Hyperglycemia
D. Hypoperfusion

D. TIA
A patient that has recurrent episodes of unilateral weakness, headaches, vision disturbances, or confusion that resolves on its own in a short period of time is likely experiencing what emergency?
A. MI
B. CVA
C. HTN
D. TIA

B. “A drop in blood flow to the brain from a failing heart.”
You are working with a new EMT on your squad. After a call for an MI patient who became unresponsive, the EMT asks you how a heart attack could cause such a thing. Which of the following most correctly answers this question?
A. “Increased blood flow due to tachycardia.”
B. “A drop in blood flow to the brain from a failing heart.”
C. “The heart was extracting too much oxygen from the bloodstream, leaving an insufficient amount for the brain to use.”
D. “The patient’s lungs were failing due to the drop in left ventricular contraction.”

C. The patient woke up seconds after going supine.
You are assessing a known seizure patient that passed out according to bystanders. During your assessment, which clinical clue would help you identify that the patient had a syncopal episode rather than a seizure?
A. The patient bit his tongue.
B. The patient was conscious upon your arrival.
C. The patient woke up seconds after going supine.
D. The patient took his antiseizure medications today.

B. Capillaries start to seep fluid
You have a patient experiencing an anaphylactic reaction. Which chemical mediator function is responsible for the edema to the airway that can rapidly cause the patient’s death?
A. Dilation of blood vessels
B. Capillaries start to seep fluid
C. Bronchoconstriction to the lungs
D. Production of thick mucus in the airways

D. Stridorous breathing
You have a male patient who was stung by a bee 5 minutes ago. Of the various signs and symptoms consistent with an allergic reaction, which one would be the most suggestive of a severe allergic reaction (anaphylaxis)?
A. Tachycardia
B. Rapid onset
C. Bilateral wheezing
D. Stridorous breathing

C. Itching and hives
The administration of epinephrine to an anaphylactic patient will not have any effect on which of the following clinical conditions?
A. Wheezing
B. Airway edema
C. Itching and hives
D. Low blood pressure

B. 0.30 mg
You are completing a PCR on an anaphylactic patient you just transported to the hospital. During transport, you had to assist the patient with the administration of her Epi-Pen. As you complete this task, what adult dose of drug would you document for the epinephrine?
A. 0.15 mg
B. 0.30 mg
C. 0.45 mg
D. 0.60 mg

C. The patient is now able to speak in full sentences.
Following the management of a patient with an acute allergic reaction, you have administered epinephrine, given oxygen, and placed him in a position of comfort. Which of the following findings is most consistent with marked patient improvement?
A. The blood pressure increased 10 mmHg.
B. Bilateral wheezing has diminished greatly.
C. The patient is now able to speak in full sentences.
D. The heart rate decreased from 132 to 108/minute.

B. Ingestion
Of the following mechanisms of entry for a poison to enter the body, which is most likely to occur to the pediatric patient?
A. Injection
B. Ingestion
C. Inhalation
D. Absorption

A. Peritonitis
You are caring for a male patient who is complaining of a severe global abdominal pain. The abdomen is also rigid to the touch. The patient has a history of blunt abdominal trauma from 5 days earlier during a sporting event that he did not seek medical treatment for. What condition is this presentation most consistent with?
A. Peritonitis
B. Pancreatitis
C. Appendicitis
D. Gastroenteritis

B. Speak slow and clear
Which of the following strategies would be most beneficial in establishing a rapport with a physologically disturbed patient who is displaying a behavioral emergency?
A. Avoid eye contact
B. Speak slow and clear
C. Do not listen to her stories
D. Speak to her sternly for clarity

A. Inability to carry oxygen
During the management of a patient known to have sickle cell anemia, the patient is complaining of shortness of breath. This could be due to what change in her red blood cells?
A. Inability to carry oxygen
B. Inability for the heart to pump the sickle-shaped cells
C. Inability for the lungs to take in oxygen and produce carbon dioxide
D. Inability of the red blood cells to use oxygen for energy production

A. The patient is immobilized supine.
While assisting another ambulance with odd-loading a patient at the hospital who is immobilized on a backboard, the EMT states that they can’t maintain the blood pressure in this 30-year-old, near-term, pregnant female. As you quickly scan the patient over, what would alert you to a cause for the hypotension?
A. The patient is immobilized supine.
B. The patient is only on low-flow oxygen.
C. The patient has a traction splint applied to the left leg.
D. The patient has backboard straps across the body tightly.

A. Sleep apnea
You arrive at the scene of a medical patient. You are shown to the patient’s bedroom where the patient is found lying on the bed. You immediately see the patient wearing a CPAP mask. Given this, what is one expected medical diagnosis you’ll probably see in the patient’s history?
A. Sleep apnea
B. Hypertension
C. Severe pneumonia
D. Neuromuscular disorder

D. The patient still has bilateral breath sounds.
You are assessing a patient who is on a home ventilator secondary to a traumatic brain injury years earlier. The family is afraid the ventilator is failing. What is the best assessment parameter to let the EMT know that the ventilator is still working properly?
A. The patient has no cyanosis.
B. The heart rate is less than 100/min.
C. The patient’s pulse ox is over 95 percent.
D. The patient still has bilateral breath sounds.

A. NHTSA
What agency has established national EMS standards as well as an assessment program to achieve these recommended standards?
A. NHTSA
B. NREMT
C. NAEMT
D. NAEMSE

B. Emergency Medical Responder
Which of the following national certification levels is designed for the first person likely to be on the scene of an accident or illness?
A. Advanced Emergency Medical Technician
B. Emergency Medical Responder
C. Emergency Medical Technician
D. Paramedic

B. Off-line medical direction
You are going to help in the self-administration of an MDI in a patient with COPD that has a prescribed inhaler. If the EMT provides this care without first contacting medical direction, what kind of protocols is he employing?
A. On-line medical direction
B. Off-line medical direction
C. Patient-directed medical care
D. Evidence-based medical direction

C. Bloodborne pathogens
Organisms that can cause diseases in humans when in contact with the infected patient’s blood, such as the HIV virus, are known by what name to the EMT?
A. Cross contamination
B. Airborne pathogens
C. Bloodborne pathogens
D. Parasite-host relationship

B. Gloves
You are exiting the ambulance for a patient who has called EMS for recurrent vomiting. At a bare minimum, what type of protective equipment should you employ prior to patient contact?
A. Gown
B. Gloves
C. HEPA mask
D. Eye protection

A. Keep the weight as close to your body as possible when lifting
Which of the following lifting techniques is best for maintaining good body mechanics to avoid potential injury?
A. Keep the weight as close to your body as possible when lifting
B. Use a backboard for moving patients down stairs whenever possible
C. When given the choice, pull a wheeled cot rather than push it
D. Carry, rather than use a wheeled device, for moving patients a distance

A. Emergency
What type of patient move would the EMT employ if the patient was in a building in danger of collapse that would certainly kill the patient?
A. Emergency
B. Non-urgent
C. Delayed
D. Urgent

D. Negligence
An adult patient is suing an EMS provider for not properly immobilizing his leg, leading to a permanent neurological injury. In a court of law, what legal tenet must the patient’s attorney prove in order to show that the EMTs were at fault?
A. Guilt
B. Assault
C. Battery
D. Negligence

B. Mobile phone
During an MCI where there are multiple people injured due to a large industrial building collapse, what type of communication technology may become easily overwhelmed by the sudden usage?
A. Repeaters
B. Mobile phone
C. Mobile radios
D. Base stations

B. 100′ by 100′
You are at the scene of an MVA where a patient is going to be flown to the hospital due to her injuries. You are serving as the landing zone coordinator for your fire department. As you set up the landing zone, what is the minimum recommended size?
A. 50′ by 50′
B. 100′ by 100′
C. 150′ by 150′
D. 200′ by 200′

A. First Responder Awareness
What would be the recommended training level for EMS responders as it relates to responding to hazardous materials incidents?
A. First Responder Awareness
B. First Responder Operations
C. Hazardous Materials Specialist
D. Hazardous Materials Technician

C. Toward the curb side of the road
Although not entirely possible at all MVA accidents, the EMS responders should exit the emergency vehicles to what side?
A. With oncoming traffic
B. Against oncoming traffic
C. Toward the curb side of the road
D. Toward the centerline of the road

C. Biological
You and your partner are attending an in-house continuing education session on terrorism. The presenter has used the term “CBRNE” multiple times, but hasn’t explained it completely. Your partner leans over and asks you what the “B” stands for. What would be your response?
A. Burns
B. Bombs
C. Biological
D. Bionuclear

D. 12 months.
Infancy is defined as the stage of life from birth to:
A. 9 months.
B. 18 months.
C. 15 months.
D. 12 months.

A. Inadequate perfusion
Shock occurs as a result of what?
A. Inadequate perfusion
B. C/Q match
C. V/Q match
D. Adequate perfusion

C. dorsalis pedis
The pulse palpated on the top of the foot is called the _ pulse.
A. phalanx proximalis
B. tibialis posterior
C. dorsalis pedis
D. suprametatarsal

A. In loco parentis
You respond to a middle school for a 12-year-old male patient who has been hit by a car. You get consent to treat the patient from the school principal. What concept allows the principal to speak for the parents?
A. In loco parentis
B. Informed consent
C. Res ipsa loquitar
D. Healthcare proxy

A. edema.
When the body’s water moves from the bloodstream into the interstitial space, it is called:
A. edema.
B. hydrostatic.
C. dehydration.
D. hypovolemia.

A. Low oxygen and high carbon dioxide
What do chemoreceptors found in the body and vascular system measure?
A. Low oxygen and high carbon dioxide
B. High oxygen and low carbon dioxide
C. Low oxygen and low carbon dioxide
D. High oxygen and high carbon dioxide

C. meninges.
The layer of covering that protects the nervous system is called the:
A. skull.
B. spinal column.
C. meninges.
D. cerebrospinal fluid.

A. The chest is a closed space with only one opening, the trachea, to inspire air; the diaphragm contracts down and the intercostal muscles expand the ribs, causing a negative pressure that fills the lungs with air.
Choose the BEST description of the chest’s mechanical functions.
A. The chest is a closed space with only one opening, the trachea, to inspire air; the diaphragm contracts down and the intercostal muscles expand the ribs, causing a negative pressure that fills the lungs with air.
B. The diaphragm contracts, causing the intercostal muscles that are attached to the lungs to also contract, which moves the air into the lungs.
C. With the chest being an airtight space, the act of breathing occurs when the muscles of the diaphragm and intercostal relax, which causes the lungs to expand and air to flow into the lungs.
D. The chest is a closed space, and the pleural space surrounds the lungs; when the diaphragm contracts and the intercostals expand, the chest and lungs move outward and downward, allowing expiration and gas exchange in the lungs.

A. Inhalation
Which of the following respiratory processes requires the active use of muscles?
A. Inhalation
B. Bifurcation
C. Exhalation
D. Gas exchange

B. Expressed consent
Which type of consent must be used by the EMT when seeking to treat a mentally competent adult?
A. Unconditional consent
B. Expressed consent
C. Conscious consent
D. Implied consent

C. Platelets
A clot cannot form without which of the following blood components?
A. Plasma
B. Corpuscles
C. Platelets
D. White blood cells

B. Adolescence
Patients in which age group tend to be very concerned with body image?
A. School age
B. Adolescence
C. Middle adulthood
D. Early adulthood

C. Draw-sheet method
For a patient without a possibility of spinal injury, which of the following carries should be used to move him from a bed to a stretcher?
A. Extremity carry
B. Direct-ground lift
C. Draw-sheet method
D. Extremity lift

D. Pancreas
Which organ secretes insulin after a person eats a large meal?
A. Gallbladder
B. Spleen
C. Liver
D. Pancreas

A. At the groin
In which of the following locations is the femoral pulse palpated?
A. At the groin
B. Behind the knee
C. On the medial side of the ankle, posterior to the tibia
D. On the medial side of the upper arm

A. glucose.
The basic nutrient of the cell and the building block for energy is:
A. glucose.
B. protein.
C. oxygen.
D. water.

D. Esophagus
Which of the following structures allows food to move from the mouth to the stomach?
A. Trachea
B. Vena cava
C. Duodenum
D. Esophagus

A. transport the patient immediately.
You discover a patient with abdominal bruising after arrival to an assault. The patient has an elevated pulse and respiratory rate, and a low blood pressure. Law enforcement is on-scene, and one of the officers asks if you can wait 15 minutes for a crime scene investigator to arrive. He said the CSI needs to photograph the bruising, which will mean valuable evidence if the perpetrator goes to trial. The EMT’s primary obligation is to:
A. transport the patient immediately.
B. use a cell phone camera to take the photos.
C. have the CSI upgrade to a Code 3 response.
D. await the CSI because the person responsible needs to go to jail.

C. Inability to detoxify harmful substances in the bloodstream
Failure of the liver would result in which of the following?
A. Inability to filter out old and damaged blood cells
B. Inability to secrete insulin
C. Inability to detoxify harmful substances in the bloodstream.
D. None of the above

D. Medical Director
Who assumes the ultimate responsibility for patient care rendered by the EMT?
A. EMS supervisor
B. EMT
C. Director of quality assurance
D. Medical Director

A. Transfer of care
Upon arrival at the hospital, the EMT advises hospital personnel of the patient’s condition, observations from the scene, treatment rendered, and other pertinent data to assure continuity of care. This process is known as which of the following?
A. Transfer of care
B. Breach of duty
C. Definitive care
D. End of tour

D. Sympathetic
What is the name of the “fight or flight” response that stimulates blood vessels to constrict and prepares the body for fighting or running?
A. Parasympathetic
B. Neurohumoral
C. Polysympathetic
D. Sympathetic

D. Ethical
You respond to a bus accident with multiple patients. As you arrive at the scene, you are approached by a local politician who states that his wife has been involved. He tells you that if you will take care of her first, he will ensure that you get the promotion you are up for. This becomes what type of decision for you as an EMT?
A. Legal
B. Financial
C. Medical
D. Ethical

A. Tibia
What is the largest bone of the lower leg?
A. Tibia
B. Femur
C. Fibula
D. Patella

D. The Ryan White Comprehensive AIDS Resource Emergency Act
What is the name of the federal act that mandates a procedure by which emergency healthcare providers can find out if they have been exposed to potentially life-threatening diseases while on the job?
A. The Communicable Disease Notification Act
B. The EMS Personnel Health Protection Act of 1991
C. The Occupational Exposure to Bloodborne Pathogens Act
D. The Ryan White Comprehensive AIDS Resource Emergency Act

A. hypersensitivity.
When a person has an exaggerated response to a body invader, the person is said to have:
A. hypersensitivity.
B. antibody reaction.
C. histamine dump.
D. hyposensitivity.

C. Standing, facing forward, with arms at the side, palms forward
Which of the following BEST describes the anatomical position?
A. Supine with arms crossed over the chest and knees slightly bent
B. Standing, facing forward, with arms raised above the head.
C. Standing, facing forward, with arms at the side, palms forward
D. Standing in profile with the hands on the hips

D. Endocrine
Which of the following body systems produces hormones?
A. Cardiovascular
B. Gastrointestinal
C. Exocrine
D. Endocrine

C. Toddler
At which of the following stages is toilet-training most likely to begin?
A. Preschool age
B. Infant
C. Toddler
D. School age

B. Epiglottis
Which of the following structures protects the lower airway from food, liquid, and other substances?
A. Trachea
B. Epiglottis
C. Larynx
D. Chordae tendinae

D. An organism that causes infection and disease
Which of the following descriptions BEST defines the term pathogen?
A. An immunity developed after an exposure
B. A study of the origins of the infection and disease
C. A medication with a harmful effect
D. An organism that causes infection and disease

C. Humerus
What is the long bone of the upper portion of the arm?
A. Scapula
B. Ulna
C. Humerus
D. Radius

D. Cardiac output
What will stroke volume and minute heart rate determine?
A. Contractility
B. Blood pressure
C. Systemic vascular resistance
D. Cardiac output

A. 500 mL
How many milliliters of air are moved during a typical breath?
A. 500 mL
B. 250 mL
C. 150 mL
D. 300 mL

B. A state of inadequate tissue perfusion
Which of the following BEST describes the medical condition of shock?
A. Hypotension
B. A state of inadequate tissue perfusion
C. An extreme emotional reaction to a stressful event
D. Delayed capillary refill

C. Xiphoid process
What is the inferiormost portion of the sternum?
A. Body
B. Sternal notch
C. Xiphoid process
D. Manubrium

C. Pulmonary arteries
What vessel carries deoxygenated blood to the lungs?
A. Vena cava
B. Aorta
C. Pulmonary arteries
D. Pulmonary veins

D. All of the above
Which of the following is a function of the skin?
A. Regulation of body temperature
B. Regulation of salt and water balance
C. Protection from the environment
D. All of the above

A. Coronary arteries that branch off the aorta
The cardiac muscle receives its supply of oxygenated blood by which of the following mechanisms?
A. Coronary arteries that branch off the aorta
B. Coronary veins that branch off the pulmonary vein as it returns oxygenated blood to the heart
C. Absorption of oxygen from the blood returning from the lungs
D. None of the above

D. standing order.
A written authorization for an EMS provider to perform a particular skill in a specific situation is a(n):
A. on-line protocol.
B. standard of care.
C. direct medical order.
D. standing order.

C. Automaticity
Which of the following terms describes the heart muscle’s ability to generate its own electrical impulses?
A. Contractility
B. Conductivity
C. Automaticity
D. Irritability

C. Left ventricle
What is the strongest and most muscular part of the heart?
A. Left atrium
B. Right ventricle
C. Left ventricle
D. Right atrium

D. Heart rate 110/minute, respiratory rate 24/minute, and systolic blood pressure of 80 mmHg
Which of the following would be a normal set of vitals for a 2-year-old male patient?
A. Heart rate 140/minute, respiratory rate 30/minute, and systolic blood pressure 60 mmHg
B. Heart rate 100/minute, respiratory rate 34/minute, and systolic blood pressure of 100 mmHg
C. Heart rate 60/minute, respiratory rate 24/minute, and systolic blood pressure of 90 mmHg
D. Heart rate 110/minute, respiratory rate 24/minute, and systolic blood pressure of 80 mmHg

D. Implied consent
Your patient is a 40-year-old known diabetic who was found unconscious at work by a coworker. What type of consent allows you to treat this patient?
A. Consent for treatment of minor emergencies
B. Consent for mentally incompetent adults
C. Expressed consent
D. Implied consent

D. Ilium
What is the large, wing-shaped bone on either side of the pelvis?
A. Ischium
B. Acetabulum
C. Pubis
D. Ilium

A. The U.S. Department of Transportation
In 1966 the National Highway Safety Act charged which of the following agencies with the development of emergency medical service standards?
A. The U.S. Department of Transportation
B. The U.S. Department of the Interior
C. The U.S. Department of Health Services
D. The U.S. Department of Homeland Security

D. Anterior
What is another term for the frontal aspect of the body?
A. Caudal
B. Dorsal
C. Posterior
D. Anterior

B. Vomiting and diarrhea
What is the most common digestive disorder?
A. Irritable bowel and gastric esophageal disease
B. Vomiting and diarrhea
C. Diarrhea and gastric esophageal disease
D. Bohemia and flatus

D. Arteries, arterioles, capillaries, venules, veins
Which of the following describes the path blood takes as it leaves the heart and then returns?
A. Veins, venules, capillaries, arterioles, arteries
B. Arteries, venules, capillaries, arterioles, veins
C. Veins, arteries, capillaries, venules, arterioles
D. Arteries, arterioles, capillaries, venules, veins

C. Personal protective equipment
What is a common term used to describe the items needed for Standard Precautions or body substance isolation precaution?
A. Harm-reduction strategy
B. Exposure-control plan
C. Personal protective equipment
D. Infection-control plan

B. An EMT transports a patient to the emergency department, leaves the patient in the waiting room, but does not advise the ED staff.
Which of the following situations BEST illustrates the act of abandonment by the EMT?
A. The EMT resuscitates a patient who has a DNR order signed by his physician.
B. An EMT transports a patient to the emergency department, leaves the patient in the waiting room, but does not advise the ED staff.
C. An EMT begins care of a patient, then turns the patient over to a paramedic.
D. An EMT who is off-duty sees a motor vehicle collision with probably injuries but does not stop to help.

C. Duty to act
The EMT’s obligation to provide care to a patient either as a formal or ethical responsibility is known as which of the following?
A. Legal responsibility
B. Scope of practice
C. Duty to act
D. Standard of care

A. Reducing psychological trauma
Which of the following BEST describes the purpose of CISD?
A. Reducing psychological trauma
B. Reenacting the call to confront stressors
C. Bringing to the public’s attention the stressful roles of public safety personnel
D. Identifying individuals who are not emotionally capable of handling EMS roles and responsibilities

D. All of the above
Which of the following situations would require the use of an emergency move?
A. Your patient has a life-threatening condition that requires repositioning.
B. You must reach other, more critically injured patients.
C. The scene is hazardous.
D. All of the above

C. Ensure scene safety
You are approaching the scene of a motor vehicle crash. Which of the following would be the first way to safeguard your well-being as an EMT?
A. Take personal protective equipment (PPE) precautions
B. Request additional resources
C. Ensure scene safety
D. Utilize DOT-approved reflective safety clothing

A. 60
The human body is made up of _ percent water.
A. 60
B. 7.5
C. 17.5
D. 75

B. A patient in shock
The Trendelenburg position would be used to transport which of the following patients?
A. A patient having difficulty breathing
B. A patient in shock
C. A patient with a head injury
D. A patient having difficulty maintaining his airway

A. Scope of practice
Which of the following refers to the set of regulations that defines the legal actions expected and limitations placed on the EMT?
A. Scope of practice
B. Legal standards of practice
C. Protocols and standing orders
D. Professional standards

A. The triage nurse at the emergency department asks about the care you provided to the patient.
In which of the following situations is it legal to share information about treatment you provided to a patient?
A. The triage nurse at the emergency department asks about the care you provided to the patient.
B. The patient gives verbal consent to release information to a friend.
C. The patient’s lawyer requests the information over the phone.
D. You are asked by a coworker who knows the patient.

B. Use your legs to lift.
Which of the following statements regarding body mechanics is true?
A. Use your back to lift.
B. Use your legs to lift.
C. Twist your torso while lifting.
D. Position your feet close together.

A. Mandible
Which of the following is the only movable bone of the face?
A. Mandible
B. Maxilla
C. Zygomatic
D. Manubrium

D. Scapula
What is another name for the shoulder blade?
A. Pharynx
B. Clavicle
C. Patella
D. Scapula

D. All of the above
Which of the following should you do when reaching for something?
A. Keep your back in a locked-in position.
B. Avoid twisting.
C. Avoid reaching more than 15 to 20 inches in front of your body.
D. All of the above

C. Adolescent
The transition from childhood to adulthood is known as which of the following?
A. Transitional
B. Young adult
C. Adolescent
D. Early adulthood

B. Long backboard
Which of the following is used to immobilize a patient with a suspected spinal injury?
A. Portable stretcher
B. Long backboard
C. Basket stretcher
D. Scoop stretcher

D. He possibly has internal injuries and is in shock.
You are called to a kindergarten class where a boy has fallen while climbing on playground equipment. The teacher saw him fall and hit his tummy on a step. He is sobbing and holding his abdomen. As you assess him, you find his skin cool and clammy, his pulse is 132, and his respiratory rate is 28. Why is this boy acting this way?
A. He is really upset and wants his mother.
B. He is embarrassed and is afraid that the other students will make fun of him.
C. He possibly has wet his pants and does not want anyone to know.
D. He possibly has internal injuries and is in shock.

D. bariatric stretchers.
Stretchers that are designed to carry obese patients up to 800 pounds are called:
A. battery-powered stretchers.
B. wheeled stretchers.
C. hydraulic stretchers.
D. bariatric stretchers.

D. anemia.
A condition that is caused by a decreased number of red blood cells is called:
A. polycythemia.
B. erythrocythemia.
C. leukemia.
D. anemia.

D. Brain, spinal cord, and nerves
Which of the following are the components of the nervous system?
A. Brain and autonomic nerves
B. Spinal cord and motor nerves
C. Spinal cord and sensory nerves
D. Brain, spinal cord, and nerves

C. All arteries carry blood away from the heart.
Which of the following statements concerning arteries is true?
A. All arteries carry oxygenated blood.
B. All arteries return blood to the heart.
C. All arteries carry blood away from the heart.
D. All arteries carry deoxygenated blood.

D. Epidermis
Which layer of the skin is exposed to the environment?
A. Adipose tissue
B. Dermis
C. Subcutaneous tissue
D. Epidermis

C. Decreased oxygen delivery to the tissues
What condition of immediate concern results from a loss of red blood cells due to hemorrhage?
A. Decreased ability to fight infection
B. Decrease in pulse rate
C. Decreased oxygen delivery to the tissues
D. Decreased production of carbon dioxide

B. Violation of patient privacy
Sharing information about a patient’s medical history with your neighbor after you hear a call on a radio scanner would constitute which of the following?
A. Slander
B. Violation of patient privacy
C. Breach of confidentiality
D. Libel

C. Radius
What is the bone on the thumb side of the forearm?
A. Fibula
B. Acromion
C. Radius
D. Ulna

D. The patient was in fear of bodily harm at the time of the incident.
Which of the following is NOT required to prove a claim of negligence against an EMT?
A. The EMT had a duty to act.
B. The patient suffered harm as a result of what the EMT did or did not do.
C. The EMT failed to act according to the standard of care.
D. The patient was in fear of bodily harm at the time of the incident.

D. Recovery position
You are treating an unconscious patient who does not have a possibility of spinal injury and who is breathing adequately. Which of the following is the BEST position for transporting the patient?
A. Spune
B. Fowler’s position
C. Semi-Fowler’s position
D. Recovery position

C. Protocols
Which of the following BEST describes a list of steps the EMT should perform while assessing and managing emergency medical situations?
A. Standing orders
B. On-line medical direction
C. Protocols
D. Standard operating procedures

A. 21
A patient breathing in room air should be receiving _ percent oxygen.
A. 21
B. 5
C. 16
D. 100

A. Emergency Medical Technician
What is the minimum level of certification required of ambulance personnel in most areas?
A. Emergency Medical Technician
B. Emergency Medical Responder
C. Advanced EMT
D. Emergency Ambulance Driver

B. Emergency move
What type of move should the EMT use to move a patient who is in a burning car?
A. Non-urgent move
B. Emergency move
C. No attempt to move
D. Urgent move

D. 30; 300
To be effective, a suction unit must be able to generate air flow of _ liters per minute and create a vacuum of _ mmHg.
A. 300; 30
B. 30; 30
C. 300; 300
D. 30; 300

B. remove the cylinder and get a green cylinder.
You are attempting to replace the oxygen cylinder in your truck. After removing the regulator from the old cylinder, removing the old cylinder, and placing the new cylinder in the oxygen compartment, you attempt to connect the regulator. The new cylinder has a yellow stripe around it instead of a green one but was stored with the green cylinders. You are unable to get the regulator to seal properly and it will not turn. You should:
A. attempt to force the regulator onto the cylinder.
B. remove the cylinder and get a green cylinder.
C. replace the oxygen regulator with a new one.
D. put the old cylinder back on the truck.

C. It limits the alveoli’s capability to exchange oxygen and carbon dioxide because the alveoli itself is not working.
Your 68-year-old patient is suffering from chronic obstructive pulmonary disease; this condition can cause gas exchange interruption by what process?
A. It is caused by a long-term disease process that blocks the blood flow to the alveoli due to arteriosclerosis interrupting gas exchange.
B. The bronchioles are constricted by the abundance of thick secretions that reduce the airflow into the alveoli.
C. It limits the alveoli’s capability to exchange oxygen and carbon dioxide because the alveoli itself is not working.
D. The process of inspiration and expiration has grown sluggish due to the advanced age of the patient and the deterioration of the lung tissue.

C. Gloves, mask, and goggles are needed.
When providing airway management and ventilation procedures, which of the following is required for body substance isolation?
A. Gloves, gown, mask, and goggles are needed.
B. Gloves only are needed.
C. Gloves, mask, and goggles are needed.
D. Body substance isolation is not needed unless secretions contain a visible amount of blood.

D. apply a nonrebreather mask giving 15 lpm of oxygen.
Your patient is a 65-year-old male with a history of COPD. He is sitting up and complaining of a severe shortness of breath. You should:
A. suction the airway with a rigid suction catheter.
B. administer 4 lpm of oxygen via nasal cannula.
C. insert a nasal airway and ventilate.
D. apply a nonrebreather mask giving 15 lpm of oxygen.

C. It may be tolerated by many patients with a gag reflex.
Which of the following is an advantage of using a nasopharyngeal airway?
A. It eliminates the need for manual positioning of the patient’s head to keep the airway open.
B. It is ideal for patients with a suspected skull fracture.
C. It may be tolerated by many patients with a gag reflex.
D. All of the above.

A. The bevel should be turned toward the nasal septum.
Which of the following is true concerning the procedure for inserting a nasopharyngeal airway?
A. The bevel should be turned toward the nasal septum.
B. If a water-soluble lubricant is not available, a silicon spray can be substituted.
C. It can only be placed in the right nostril.
D. The length of the device is not as important as it is with oropharyngeal airways.

D. Air moves into the alveoli, blood is transported by the pulmonary capillaries, and diffusion occurs.
Which of the following statements BEST describes the exchange of gas in the alveoli?
A. Blood moves by way of the pulmonary capillaries, air arrives at the alveoli, and osmosis occurs.
B. Blood moves from the left heart to the lungs, air arrives in the alveoli sacks, and diffusion occurs.
C. Air moves into the airway, blood arrives via the pulmonary veins, and osmosis occurs.
D. Air moves into the alveoli, blood is transported by the pulmonary capillaries, and diffusion occurs.

C. distress.
A 16-year-old patient presents with labored breathing and increased respiratory rate, increased heart rate, and leaning forward with his hands on his knees. His skin is pink and his pulse oximetry is 96. This patient is suffering from respiratory:
A. failure.
B. hypoxia.
C. distress.
D. arrest.

B. D tank
Which of the following oxygen cylinders would be the closest to last no longer than 50 minutes flowing at 10 liters per minute?
A. M tank
B. D tank
C. G tank
D. E tank

C. Regulator
Which of the following is necessary to deliver oxygen to patients at a safe pressure?
A. Filter
B. Float ball
C. Regulator
D. Flowmeter

A. Supplement the breaths with high-concentration oxygen through a nonrebreather mask.
A 16-year-old patient presents with labored breathing and audible wheezes, heart rate of 124, respiration 36; he is confused about whether he has taken his asthma medication. What is the treatment for this patient?
A. Supplement the breaths with high-concentration oxygen through a nonrebreather mask.
B. Use a pocket mask, which will provide adequate oxygen to improve the patient’s condition.
C. Ventilate with a bag-valve mask with high oxygen or FROPVD.
D. Give mouth-to-mouth breathing with a nasal cannula, providing the patient with an increase of oxygen.

A. The trachea is easily obstructed by swelling.
Which of the following should be kept in mind when assessing and managing the airway of a pediatric patient?
A. The trachea is easily obstructed by swelling.
B. The tongue is not as likely to obstruct the airway as in an adult.
C. Due to their short necks, pediatric patients require a greater degree of hyperextension to open the airway than do adults.
D. Gastric distention is unlikely.

A. Check your rate of ventilation.
Your patient is a 4-year-old male who was struck by a vehicle and is now unresponsive with an obvious head injury. As you are ventilating him with a bag-valve mask device, you detect increasing resistance to ventilation. Which of the following should you do?
A. Check your rate of ventilation.
B. Stop ventilations for 1 to 2 minutes to allow trapped air to escape from the lungs.
C. Perform a head-tilt, chin-lift maneuver to ensure that the airway is open.
D. Switch to a flow-restricted oxygen-powered ventilation device.

A. Check for a radial pulse.
You have arrived at the scene of a call for a “man down.” As you enter the residence you note that your patient is a male in his mid-60s who is awake but does not seem to acknowledge your presence. He is perspiring profusely, has cyanosis of his ears and lips, and has rapid, shallow respirations. Which of the following should you do first?
A. Check for a radial pulse.
B. Obtain the patient’s medical history.
C. Listen to his lung sounds.
D. Assist ventilations with a bag-valve mask and supplemental oxygen.

B. Nasal flaring
Which of the following is a sign of an inadequate airway?
A. Regular chest movements
B. Nasal flaring
C. Equal expansion of both sides of the chest when patient inhales
D. Typical skin coloration

C. It may be tolerated by many patients with a gag reflex.
Which of the following is an advantage of using a nasopharyngeal airway?
A. It eliminates the need for manual positioning of the patient’s head to keep the airway open.
B. It is ideal for patients with a suspected skull fracture.
C. It may be tolerated by many patients with a gag reflex.
D. All of the above.

D. All of the above
On which of the following types of calls should you bring your portable suction unit to the patient’s side upon arrival on the scene?
A. Motor vehicle collision
B. Cardiac arrest
C. Seizure
D. All of the above

A. 10
When suctioning the airway, suction should never be applied for longer than _ seconds.
A. 10
B. 30
C. 45
D. 60

D. A 24-year-old woman who is breathing 28 times per minute after being in an argument with her husband
Which of the following patients does NOT require the administration of supplemental oxygen?
A. A 60-year-old woman with a history of chronic obstructive pulmonary disease (COPD) who can speak two or three words at a time without a breath
B. A 6-year-old male with a history of asthma whose breath sounds are silent and who is drowsy
C. A 31-year-old male who is unresponsive due to an overdose of narcotics
D. A 24-year-old woman who is breathing 28 times per minute after being in an argument with her husband

B. COPD has changed the stimulus to breathe to the hypoxic drive and high concentration can depress breathing.
What is the extremely rare condition that can cause respiratory depression?
A. Lungs can react unfavorably to oxygen when the concentration is too high for a long period of time and this can depress breathing.
B. COPD has changed the stimulus to breathe to the hypoxic drive and high concentration can depress breathing.
C. High concentration of oxygen can depress breathing when the patient has an allergic reaction from the oxygen.
D. The eyes can develop scar tissue on the retina from a high concentration of oxygen.

B. Nonrebreather mask
Which of the following is the best device to deliver high-concentration oxygen to a breathing patient?
A. Simple face mask
B. Nonrebreather mask
C. Nasal cannula
D. Oropharyngeal airway

C. internal respiration.
The movement of oxygen and carbon dioxide across the cell membranes from the capillaries is called:
A. external respiration.
B. dehydration.
C. internal respiration.
D. oxygenation.

C. alveoli.
The structures within the lung that allow exchange of gases with the bloodstream are known as:
A. bronchioles.
B. pleura.
C. alveoli.
D. carina.

A. apply cricoid pressure.
You are ventilating an adult patient with a bag-valve mask when you notice that his abdomen is getting bigger. You should:
A. apply cricoid pressure.
B. decrease the flow of oxygen.
C. apply pressure to the abdomen.
D. suction the airway.

A. increase the concentration of oxygen.
You are aggressively ventilating an adult patient with a bag-valve mask when you notice that his previously strong pulse is getting weaker. You should:
A. increase the concentration of oxygen.
B. reduce the concentration of oxygen.
C. reduce the volume of the ventilations.
D. begin chest compressions.

C. Turn the patient on her side and remove the airway.
Your patient, in whom you have inserted an oropharyngeal airway, is beginning to regain consciousness and develop a gag reflex. Which of the following is the proper way of managing this situation?
A. Pull the airway out slightly to keep it away from the back of the throat.
B. Use gentle manual pressure to keep the patient from expelling the airway.
C. Turn the patient on her side and remove the airway.
D. Spray a topical anesthetic into the throat to prevent the gag reflex from being stimulated.

D. Using large bore suction tubing without a tip or catheter attached
You are ventilating a cardiac arrest patient when he begins to vomit copious amounts of large pieces of undigested food. Which of the following would be most effective in clearing the airway?
A. Using a rigid pharyngeal suction tip
B. Irrigating the mouth with sterile water to dilute the material before suctioning
C. Using a 14 French suction catheter
D. Using large bore suction tubing without a tip or catheter attached

B. Insert the catheter or tip to the desired depth prior to applying suction.
Which of the following is the correct method of suctioning?
A. Suction intermittently, both while inserting and withdrawing the suction tip or catheter.
B. Insert the catheter or tip to the desired depth prior to applying suction.
C. Begin suctioning as you insert the suction tip or catheter into the mouth.
D. Suction continuously, both while inserting and withdrawing the suction tip or catheter.

D. When the respiratory challenge continues, the systems cannot keep up with the demand, and skin color and mental status change.
When does respiratory distress change to respiratory failure?
A. When the respiratory challenge continues, the systems fail with the demand for oxygen, pupils dilate, and the skin becomes hot and dry.
B. When the patient who is short of breath, with noisy respiration, presents in the tripod position but then suddenly has the condition clear up and return to normal.
C. When the compensatory mechanism is no longer needed and the patient goes into arrest.
D. When the respiratory challenge continues, the systems cannot keep up with the demand, and skin color and mental status change.

D. Pressure-compensated flowmeter
Of the three types of oxygen flowmeters, which one can only be used upright?
A. Bourdon gauge flowmeter
B. Hudson gauge flowmeter
C. Constant flow selector valve
D. Pressure-compensated flowmeter

B. Nasal flaring
Which of the following is a sign of an inadequate airway?
A. Regular chest movements
B. Nasal flaring
C. Equal expansion of both sides of the chest when patient inhales
D. Typical skin coloration

B. Commonly seen as blue or gray skin, deterioration of patient’s mental status like confusion or restlessness
What are the signs of hypoxia?
A. Warm dry skin, with difficulty breathing, and hypertension
B. Commonly seen as blue or gray skin, deterioration of patient’s mental status like confusion or restlessness
C. Disease process that robs the patient of adequate breathing and perfusion
D. Shock caused from the lack of blood flowing to the vital organs like the brain and heart that is irreversible

D. They cannot be used in a patient with a gag reflex.
Which of the following is a disadvantage of oropharyngeal airways?
A. They cannot be used in patients with a suspected skull fracture.
B. They do not come in pediatric sizes.
C. They require the use of a water-soluble lubricant.
D. They cannot be used in a patient with a gag reflex.

D. Inflate the reservoir bag and make sure the bag does not deflate during inspiration.
Before applying a nonrebreather mask, the EMT should take what action?
A. Insert the proper venture to receive the correct oxygen concentration.
B. Connect the mask to a humidified oxygen source and observe for the heart rate to slow.
C. Make sure the oxygen supply has greater than 200 psi in the tank.
D. Inflate the reservoir bag and make sure the bag does not deflate during inspiration.

B. A homeless person of undetermined age found lying unresponsive in an alley with no bystanders
Which of the following patients should NOT have their airway opened using a head-tilt, chin-lift maneuver?
A. A 35-year-old diabetic woman found unresponsive in the driver’s seat of her vehicle in the parking lot of her apartment complex
B. A homeless person of undetermined age found lying unresponsive in an alley with no bystanders
C. A 50-year-old woman who choked on a piece of food while dining in a restaurant and slid out of her chair
D. A 25-year-old man who is still unresponsive after a grand mal seizure

A. insert a nasopharyngeal airway.
Your patient is breathing 4 shallow breaths a minute due to overdosing on his pain medication but he has a palpable radial pulse. The patient gagged while you suctioned his airway. Now that the airway is clear, you should:
A. insert a nasopharyngeal airway.
B. insert an oropharyngeal airway.
C. ventilate the victim in the recovery position.
D. compress the chest to clear the lungs.

B. You should increase the patient’s oxygen flow rate to deliver adequate amounts of oxygen to his tissues. If his respiratory rate decreases, you can assist him with a bag-valve mask device.
Your patient is a 55-year-old man with a history of chronic bronchitis. You have been called to his home today because of an increase in his level of respiratory distress. The patient is on 2 liters per minute of oxygen by nasal cannula at home. Your assessment reveals difficulty speaking due to shortness of breath, leaning forward to breathe, a productive cough, and a respiratory rate of 32 per minute. Which of the following is true concerning the best course of action for this patient?
A. Because increased blood levels of carbon dioxide are the primary stimulus to breathe, you should encourage the patient to rebreathe his exhaled air from a paper bag.
B. You should increase the patient’s oxygen flow rate to deliver adequate amounts of oxygen to his tissues. If his respiratory rate decreases, you can assist him with a bag-valve mask device.
C. You should increase the patient’s oxygen flow rate until his respiratory rate decreases and then resume oxygen administration at 2 liters per minute.
D. You should not increase the patient’s oxygen flow rate because of his likely dependence on a hypoxic drive to stimulate breathing.

D. apply a nonrebreather mask giving 15 lpm of oxygen.
Your patient is a 65-year-old male with a history of COPD. He is sitting up and complaining of a severe shortness of breath. You should:
A. suction the airway with a rigid suction catheter.
B. administer 4 lpm of oxygen via nasal cannula.
C. insert a nasal airway and ventilate.
D. apply a nonrebreather mask giving 15 lpm of oxygen.

A. 200
The safe residual for an oxygen cylinder is _ psi.
A. 200
B. 300
C. 500
D. 1,000

C. With the addition of a second passage, the positive pressure in the chest balances with the atmospheric pressure to create a flail segment that allows for air to be sucked through the wound.
A 28-year-old male has been stabbed in the chest with a hunting knife. As you perform your primary survey, you see that air is escaping from the wound. What is the reason for this?
A. When the mechanics of breathing are disrupted, a negative pressure cannot be created to pull air through the normal air passages and air is sucked through the wound.
B. Chest integrity has caused disruption of the mechanics of breathing over pressuring the plural space, taking the air flow through the wound.
C. With the addition of a second passage, the positive pressure in the chest balances with the atmospheric pressure to create a flail segment that allows for air to be sucked through the wound.
D. The disruption of chest integrity has caused bronchoconstriction to increase, which makes the only pathway for the air through the wound.

C. 6
The oxygen flow rate for a nasal cannula should not exceed _ liters per minute.
A. 8
B. 4
C. 6
D. 2

A. The trachea is easily obstructed by swelling.
Which of the following should be kept in mind when assessing and managing the airway of a pediatric patient?
A. The trachea is easily obstructed by swelling.
B. The tongue is not as likely to obstruct the airway as in an adult.
C. Due to their short necks, pediatric patients require a greater degree of hyperextension to open the airway than do adults.
D. Gastric distention is unlikely.

C. The swelling of the abdominal space causes the diaphragm to be restricted, which will reduce the thorax space.
Why does a patient involved in an auto crash who has major internal abdominal injuries require oxygen to maintain internal respiration?
A. The red blood cells have a reduction of hemoglobin that reduces the amount of oxygen that can be transported.
B. A lack of oxygen in the air decreases the oxygen diffused into the bloodstream, which creates an increase of carbon dioxide.
C. The swelling of the abdominal space causes the diaphragm to be restricted, which will reduce the thorax space.
D. A lack of circulating volume decreases the oxygen and carbon dioxide transport capability.

B. roll him over onto his side to clear the airway.
Your patient is breathing 4 shallow breaths per minute due to overdosing on his pain medication but he has a palpable radial pulse. He vomited prior to your arrival and is choking. You should:
A. insert an oropharyngeal airway and ventilate.
B. roll him over onto his side to clear the airway.
C. perform chest thrusts to clear the lungs.
D. move the patient to the ambulance and suction.

D. High carbon dioxide and low oxygen
The normal stimulus to breathe is stimulated by the chemoreceptors that measure the change of what two gases?
A. Low hydrogen and high carbon monoxide
B. High carbon monoxide and low oxygen
C. High hydrogen and low carbon dioxide
D. High carbon dioxide and low oxygen

B. A flexible gasket
Which of the following is acceptable for maintaining a seal between an oxygen cylinder and regulator?
A. A pop-off valve
B. A flexible gasket
C. A light coating of lubricant
D. Medical grade adhesive tape

A. The lungs may not have time to fill and exchange gas.
Which of the following describes why fast respiration may decrease minute volume?
A. The lungs may not have time to fill and exchange gas.
B. The rate causes turbulence in the trachea that increases friction and decreases the amount of air movement.
C. It is due to the delay in the movement of the intercostal muscles and the pleural space.
D. The rate does not decrease minute volume; it actually increases.

A. Use a bag-valve mask with supplemental oxygen.
Your patient is a motorcyclist who was ejected from his vehicle due to striking a guard rail. The patient is unresponsive to painful stimuli and is breathing six to eight times per minute. Which of the following should you do first?
A. Use a bag-valve mask with supplemental oxygen.
B. Perform a rapid trauma assessment.
C. Apply a cervical collar.
D. Apply a nonrebreather mask with an oxygen flow rate of 15 lpm.

D. hypoxia.
For life to be maintained, a balance of oxygen and carbon dioxide is needed. The condition when oxygen levels are low is called:
A. hypotension.
B. hypercarbia.
C. hyperventilation.
D. hypoxia.

D. ease the head forward a little.
You have performed a head-tilt, chin-lift maneuver on a 17-month-old boy and are attempting to ventilate him with a bag-valve mask. You are experiencing a lot of resistance with each breath and the chest is barely rising. Prior to attempting ventilations again, you should:
A. visually examine the airway.
B. tilt the head back further.
C. perform chest thrusts.
D. ease the head forward a little.

A. carina.
The point at which the trachea divides into the two mainstem bronchi is called the:
A. carina.
B. sternal notch.
C. xiphoid process.
D. hypopharynx.

A. Bronchi
Which of the following structures is found in the lower airway?
A. Bronchi
B. Uvula
C. Pharynx
D. Tonsils

D. Increased effort to breathe, cyanosis, cool clammy skin, altered mental status
What signs and symptoms would indicate inadequate breathing in a patient?
A. Increased effort to breathe, increased depth of respiration, pink dry skin, normal mental status
B. Rapid breathing, pale skin, and a normal mental status
C. Decreased depth of respiration, decreased rate of breathing, hot clammy skin, normal mental status
D. Increased effort to breathe, cyanosis, cool clammy skin, altered mental status

B. Insert the catheter or tip to the desired depth prior to applying suction.
Which of the following is the correct method of suctioning?
A. Suction intermittently, both while inserting and withdrawing the suction tip or catheter.
B. Insert the catheter or tip to the desired depth prior to applying suction.
C. Begin suctioning as you insert the suction tip or catheter into the mouth.
D. Suction continuously, both while inserting and withdrawing the suction tip or catheter.

A. give breaths with a pocket mask.
Your patient was attempting to slide down a railing on a skateboard. He struck a concrete step with his face and is unconscious. You note a clear viscous fluid coming from his nose. He is breathing inadequately and requires ventilation. You should:
A. give breaths with a pocket mask.
B. suction the fluid to clear the airway.
C. insert a nasopharyngeal airway.
D. put the victim in the recovery position.

C. respiration.
The process of air moving in and out of the chest is called:
A. tidal volume.
B. inhalation.
C. respiration.
D. ventilation.

A. 50 percent
What is the percentage of oxygen provided by connecting a high flow of oxygen to the oxygen inlet found on a pocket mask?
A. 50 percent
B. 21 percent
C. 100 percent
D. 16 percent

C. leave the dentures in place.
You are ventilating an 85-year-old male without difficulty. A nurse tells you that the patient has dentures. To ensure a good mask seal, you should:
A. tape the dentures in place.
B. remove the dentures.
C. leave the dentures in place.
D. use an infant mask over the nose.

A. 80 to 90 percent
A nonrebreather mask at 12 to 15 liters per minute can deliver to the patient what percent of oxygen?
A. 80 to 90 percent
B. 16 to 21 percent
C. 24 to 44 percent
D. 90 to 100 percent

D. Green
Which of the following colors identifies an oxygen cylinder?
A. Orange
B. Black
C. Blue
D. Green

A. Use a nasal cannula instead.
You are transporting a 44-yearr-old female with chest pain and sudden respiratory distress. She is agitated, anxious, and refuses to have a nonrebreather mask applied. Which of the following is the best option?
A. Use a nasal cannula instead.
B. Have her breathe into a paper bag to control her hyperventilation.
C. Do not make further attempts to administer oxygen as it will only agitate the patient further.
D. Consult with medical control about restraining the patient.

Leave a Comment

Scroll to Top